判断推理经典题

2022-07-12 12:53:15   第一文档网     [ 字体: ] [ 阅读: ] [ 文档下载 ]
说明:文章内容仅供预览,部分内容可能不全。下载后的文档,内容与下面显示的完全一致。下载之前请确认下面内容是否您想要的,是否完整无缺。下载word有问题请添加QQ:admin处理,感谢您的支持与谅解。点击这里给我发消息

#第一文档网# 导语】以下是®第一文档网的小编为您整理的《判断推理经典题》,欢迎阅读!
推理,判断,经典



01. 一个研究者发现,按照身体重量的比例,孩子比成人吃更多的碳水化合物,孩子同时也比成人运动得多。研究者假设,碳水化合物的消耗与不同程度的锻炼相联系的卡路里的需求成正比变化。

下列哪一个,假如正确,最反对研究者的假说?

A. 在政府为公众运动项目中平均每人花费更多的国家,平均每人吃的碳水化合物更多。 B. 不参加有组织运动中的孩子比参加有组织运动的孩子倾向于较少的碳水化合物。 C. 吃增加量的碳水化合物是长跑运动员准备长距离奔跑的一个通常的战术。 D. 身体生长时期比其他情况下需要相对更多的碳水化合物。

E. 尽管碳水化合物是维持身体健康所必不可少的,吃更多碳水化合物的人并不一定更健康。 02. 去年,埃塞俄比亚从世界银行得到了25亿美元的贷款,它的国民生产总值增长了5%今年,埃塞俄比亚向世界银行提出两倍于去年的贷款要求,它的领导人并因此期待今年的国民生产总值将增加10%但专家认为,即使上述贷款要求得到满足,埃塞俄比亚领导人的期待也很可能落空。

以下各项如果正确,都将支持专家们的意见,除了:

A. 去年该国5%GNP增长率主要得益于农业大丰收,而这又主要是难得的风调雨顺所致。 B. 埃塞俄比亚的经济还未强到足以吸收每年30亿美元以上的外来资金。 C. 埃塞俄比亚不具备足够的重工业基础以支持每年6%以上GNP增长率。

D. 对发展中国家来说,外来资金对国民生产总值增长的促进作用怎么估计也不过分。 E. 外来资金对国民生产总值增长的促进作用在很大程度上依赖于国际国内环境

03.在如今信息时代,计算机越来越普及,越来越在众多领域发挥重要作用。在这种情况下,教会学生有效地使用计算机是很重要的。因此,学校应该给全体学生开设计算机程序设计课,以使全体学生都会编写计算机程序。

以下哪项,如果为真,最可能削弱上述结论?

A.只有那些能够有效使用计算机的人,才擅长计算机程序设计。 B.只有精通计算机程序设计的人,才能有效地使用计算机。 C.不少能有效使用计算机的人,并不会编写计算机程序。 D.有些学校教授的计算机编程课比其他学校更为实用。 E.绝大多数能编程的人,都能熟练使用计算机。 04. 晶晶的父母正在分析孩子为什么会发生肥胖。 父亲说:“我看孩子的肥胖是由于她不爱运动引起的。 母亲说:“我听说只有大量脂肪堆积在体内才会引起肥胖。所以我看孩子的肥胖还是因为她吃的脂肪太多引起的,和运动不运动没有关系。 下列哪项可以说明母亲对父亲的反驳是不对的? A 吃过多的含高脂肪的食物确实可以导致肥胖。

B 现在的孩子普遍营养过剩,又不只是他们一个孩子的问题。 C 现在的孩子确实没有原来的孩子喜欢运动了。 D 不爱运动就容易导致脂肪大量堆积在体内。

E 因为孩子肥胖以后,运动起来会很累,所以一般越胖的孩子越不爱运动。

05. 比较古树上的生长年轮使科学家从一片木片上测定用作木材的树被砍伐时的年代。因此,通过分析残存古代建筑上的木材的生长年轮,考古学家能够精确测定那些建筑物被建造的年代。

下列哪一个是上文所基于的假设?

A. 在古代建筑中所用的木材来自非常老的树。


B. 用于古代建筑的木材,在被用于建筑之前,不会在一段无法确定的时间被闲置不用。 C. 在某一确定年代中砍伐的任何树木的生长年轮与那一年被砍伐的其他树的生长年轮相同。

D. 幸存到现在的最古老的建筑是由最持久的木材建成的。

E. 古代的建筑师在一特定建筑物的建造中不会使用超过一种类型的木材。

06. 美国交通部公布的统计数字表明,接近80%的交通死亡事故具有以下特点:第一,车速低于每小时50英里;第二,出事地点在距肇事者家25英里之内。因此,如果你在美国驾车时速高于50英里并且超出你家方圆25英里之外,那么,你是较为安全的。 以下哪项,如果是真的,最为有力地削弱了上述论证的结论? A.在导致死亡的交通事故肇事者中,75%是年轻人。

B.在因车速超过法定时速而被捕的人中80%是酒后驾车者。

C.全国每年的交通死亡事故中,50%发生在6个周末,这6个周末被认为是交通高风险周末,因为它们包括全国的节日。

D.交通部的统计数字的来源是各种警察局的报告。 E一天中只有凌晨一时至三时,才允许汽车超过时速50英里或者超出司机家方圆25英里。 07. 去年春节我们只用了2天时间就销售福利彩票400万元。今年春天我们准备用10天的时间来销售彩票,估计销售2000万元是不成问题的。 以下哪项最能支持上述推理?

A 在前年举办的春季体育彩票的销售活动中,5天的时间就销售彩票1000万元。 B 体育彩票由于奖品采取奖金的方式发放,一直很受摸彩人的青睐。

C 我国的体育事业发展很快,这是和全国人民的支持分不开的。我们买体育彩票也是对体育事业的支持。

D 在每年卖体育彩票的过程中,我们发现每天的销售量是差不多的 E 我国经济一直处于良性发展状态,人们的生活水平有了显著提高。

08. 在布各兰,价值420万美元的大学奖学金津贴去年无人问津,因为许多奖学金项目没有吸引到合乎项目标准的申请者。这在一个每年有成千上万名有前途的学生付不起学费的国家是一种惊人的资金浪费。因此这些奖学金的标准应当被修改。 下列哪一个,在布各兰假如正确,最支持上面的结论?

A. 许多奖学金无人领取并不是因为它们的合格标准太严格了,而是因为这些奖学金项目很少宣布它们的存在。

B. 因为通货膨胀,某些奖学金项目所提供的津贴现在看起来比他们当初提供的时候不那么对潜在申请者有吸引力了。

C. 去年,大量的奖学金项目发放了所有的可获得的奖学金津贴,大部分项目也有非常严格的合格标准。

D. 某些奖学金项目仅仅从在某一确定的专业中学习的学生那里接受申请,但是布各兰的大学不再提供那些奖学金项目所指定的领域的专业。

E. 一些奖学金项目已经发现在没有进行冗长并且昂贵的法律程序的情况下不可能修改他们的合格标准。

09. 过敏性肠胃炎一直被认为是由食物过敏引起的。但是,如果我们让患者停止食用那些已经证明会引起过敏性肠胃炎的食物,可他们的过敏性肠胃炎并没有停止。因此,显然存在别的某种原因引起过敏性肠胃炎。

下列哪项如果是真的,最能削弱上面的结论?

A. 诱发许多患者过敏性肠胃炎的往往是他们最喜欢吃的食物。 B. 许多不患过敏性肠胃炎的人同样有食物过敏反应。


C. 很少有食物过敏会引起像过敏性肠胃炎那样严重的症状。

D. 许多过敏性肠胃炎患者同时患有神经官能症,表现为多疑、不安等。

E. 许多普通食物在食用几天后才会诱发过敏性肠胃炎,因此,不易观察患者的过敏反应和其食用的食物之间的关系。

10. 在德士飞镇,新的环境行动党在1988年赢得了七人镇议会中的两个席位。在1992的选举中,它失去了这两个席位,尽管该党派保护环境的政纲实质上保持不变。该党派命运的变化展示在德士飞镇,对环境的关心在1988年至1992年间明显下降了。 下列哪一个,假如正确,最反对上面的论述?

A. 1988年到1992年间,德士飞镇合法选民的数目增加了,但实际投票的比例没有增加。 B. 1988年到1992年间,德士飞镇的最主要的政治党派修改了其政纲,采纳了一个很强的环境保护主义者的立场。

C. 1992年德士飞镇竞选候选人的党派与1988年选举中的党派是相同的。 D. 1992年,环境行动党所赢得的票数比1988年下降了。

E. 1988年至1992年间,一些有益于环境的方法被镇议会采纳,但没有产生有说服力的结果。

11. 20世纪初,由于滥捕滥猎,海豹的数量已降到了几十只,濒临灭绝的边缘。在最近几十年,由于各国政府的保护,海豹的数量又迅速增加了。然而,由于它们通过广泛的近交进行自我繁殖,显示出遗传的惊人一致性,这是其他哺乳动物所没有的,因此它们面临绝迹危险性要比其他种类大得多。

以下哪项最可能是其他哺乳动物比海豹绝迹的危险更小的原因? A.其他种类的哺乳动物数目多,因此死亡一些并不太紧要。

B.其他种类的哺乳动物经过一代代的经验积累,已经增加了对危险的了解。 C.猎人很容易辨认出其他哺乳动物的雌雄或老少。

D.在其他种类的哺乳动物中,一些成员因遗传原因比其他成员更能抵抗疾病或灾害。 E.因为其他种类的哺乳动物没有被当作濒临灭绝的动物而加以保护,所以保留了小心、警惕的习惯。

12. 某私立大学准备分批送教师去国外培训。有人提出反对意见,因为这需要一大笔培训经费,会减少学校收入,削弱学校的竞争力。 以下哪一项,最能削弱反对者的意见?

A. 教师具有较高的水平,可以吸引更多的学生,从而增加学校收入。 B. 教师送到国外培训,可以使学校有更好的声誉。 C. 学校的资金比较宽裕,完全可以支付这笔培训经费。

D. 教师到国外培训后,会以对某私立大学感激的心情努力工作。 E. 把教师送到国外培训,可以和外国大学建立合作关系

13. 通常人们不认为美国是一个鹦鹉爱好者的国度,然而在一个被选择的可比较国家的鹦鹉拥有者的统计中,美国被排在第二位,每100人中有11个鹦鹉拥有者。从中可以得到如下结论:美国人比大多数国家的人更有可能拥有鹦鹉。 下列哪一个知识在评价上述结论的准确性上最有用? A. 美国鹦鹉的数目。

B. 美国鹦鹉拥有者的数目。

C. 统计中排在第一位的国家每100人中鹦鹉拥有者的数目。

D. 与其他宠物鸟拥有者的数目相比较,美国鹦鹉拥有者的数目。 E. 统计中未被包括的国家每百人中鹦鹉拥有者的数目。

14. 按照惯例,北华工商管理学院每年一度的特等奖学金获得者的名额仅3名,奖金数额


高达10000元。1998年,该校董事会关于评定本年度特等奖学金确定了如下的原则:只有连续3年获得过三等奖以上的奖学金并且本年度各门学科终考成绩平均得分在90分以上的,方可获该项特等奖学金。但评选的结果表明,校董事会确定的上述原则并没有得到严格的贯彻。

以下哪项如果为真,最能支持上述结论?

A. 1998年度北华工商管理学院学生的各科终考成绩的平均得分都在90分以下。 B. 校董事会的上述评选原则受到了各方的批评。 C. 张三为1997年入学,他以本年度各科终考平均得分95分的优异成绩获1998年度特等奖学金。

D. 李四连续三年获得三等奖以上的奖学金并且本年度各学科终考成绩平均得分在90分以上,但没有获得本年度的特等奖学金。

E. 1998年度北华工商管理学院实际评定的特等奖学金获得者的名额为6名,每名的奖金数额下降为5000元。

15. 直到1984年,惟有阿斯匹林和爱思纷占据利润丰富、不需处方购买的止痛药市场。然而,在1984年布洛芬被期望占据所有非处方购买的止痛药市场销售的15%。以这个基础,商业专家预测1984年阿斯匹林和爱思纷在销售上要下降相应的15% 上文最后一句的预测基于下列哪一个假设?

A. 大多数消费者宁愿使用布洛芬而不是阿斯匹林和爱思纷。

B. 阿斯匹林、爱思纷和布洛芬都缓解头疼和肌肉痛,但是阿斯匹林和布洛芬也可以导致胃部疼痛。

C. 1984年以前,布洛芬仅是凭处方购买才可获得的药。 D. 制造和销售阿斯匹林和爱思纷的公司不制造和销售布洛芬。 E. 布洛芬的引入不增加非处方止痛药市场的总销售。

16. 在一个小学校的图书馆中,科技书和文艺书一直是最受学生欢迎的两种图书,最近借科技书的同学越来越多,所以借文艺书的同学有所减少。 以上说法要假设下面哪个前提?

A 因为最近在高科技活动月活动,所以科技书很受欢迎。

B 同学们的课程很多,课外作业也很多,所以没有时间看文艺书。 C 借文艺书的同学和借科技书的同学的总和是基本不变的。

D 喜欢读书的同学往往各类图书都喜欢读,不喜欢读书的同学什么书都不爱读。 E 只有高年级的同学到图书馆借书,低年级的同学认字太少,一般不借书。 17. 美国实业界有一种观点对政府干预持绝对否定的态度,认为政府干预增加了实业的经营成本,减弱了有效竞争。他们举出像货运、航空、电讯这样一些实业,政府的放松或放弃干预明显地增加了经济效益。这种观点忽略了另外一些实业部门,例如金融业,其中政府的干预和规范是不可缺少的。如果没有政府的干预,美国就不可能渡过30年代的金融危机。 以下哪项是以上反驳所使用的方法? A.对论敌的动机提出质疑。

B.指出论敌的论证所使用的概念前后不一致。

C.指出论敌的基本假设是一种主观臆造,没有客观依据。

D.指出论敌所举的实例不具有否定政府干预必要性的典型意义。 E.举出反例来驳斥论敌的一般性结论。

18. 为了产生种子,植物必须首先开花。两种龙蒿植物,俄罗斯龙蒿和法国龙蒿,看起来非常类似。俄罗斯龙蒿开花,而法国龙蒿不开花。然而俄罗斯龙蒿的叶子,缺乏那种使法国龙蒿成为理想烹饪香草的独特气味。


假如上面的信息是正确的,下列哪一个可以基于此可靠地推导出来? A. 作为装饰性的植物,法国龙蒿比俄罗斯龙蒿更理想。 B. 俄罗斯龙蒿的花可能是没有气味的。

C. 在以标签“龙蒿”的袋子中出售的种子而生长出来的植物不是法国龙蒿。 D. 除了俄罗斯龙蒿和法国龙蒿以外没有其他种类的龙蒿。 E. 叶子有味 道的花园植物通常不开花。

19. 专家们从16世纪意大利艺术家所画的一幅画中的一个人物上移去了一层18世纪的红色颜料,揭示出下面有一层绿色颜料。因为绿色颜料追溯到16世纪,这个人物在1563年被完成时一定是绿色,而不是红色。

下列哪一个,假如正确,最反对上面的论述? A. 专家们被委任重现当该画被完成时的颜色。 B. X射线揭示在人物的绿颜色下还有一层颜色。 C. 化学分析被用于决定红颜色和绿颜色的年代。

D. 18世纪被加上的红色是修复17世纪晚斯所造成的损害的一个补救。 E. 画中另一个人物袍子上的红色追溯到16世纪。

20. 两位大妈听说有帮助困难学生完成学业的致远学校。正在议论一个邻居的孩子王小伍能否进这所学校读书。 张大妈说:“王家的人均收入每月不足200元,这在北京来说,真够困难的,我看王小伍能进致远学校读书。 李大妈说:“我看不一定,听说王小伍学习成绩不太好,上次期末考试他有三门功课不及格呢。

李大妈的看法如果成立,必须有以下哪项假设? A. 致远学校只招收本地区的学生。 B. 做为致远学校的学生要品学兼优。

C. 只有家庭经济困难的学生才能进入致远学校学习。 D. 作为致远学校的学生不一定非得家庭经济困难。

E. 仅仅家庭经济困难,还不够进入致远学校学习的条件。 21. 尽管海贝岛上出售许多牌子的汽油,那儿的汽油公司从海贝海港的仅有的储油罐那里得到他们所销售的所有精炼汽油,那儿的储油罐也总是补充相同质量的汽油。因此,海贝岛上出售的汽油牌子可能在名称和价钱上不同,但他们在质量上是相同的。 上面的结论基于下列哪一个假设?

A. 消费者通常意识不到他们所购买的汽油质量的变化,除非汽油公司宣布那些变化。 B. 当油船运汽油到海贝岛的海港,海贝岛上的储油罐总是与上一次运送一样接受相同量的汽油。

C. 海贝岛上出售的许多牌子的汽油在价钱上有很大的区别。

D. 假如海贝岛上任一家汽油公司在出售前改变汽油的质量,其他汽油公司在出售前也使用导致相同汽油质量变化的方法。

E. 海贝岛上汽油储油罐足够大来满足海贝岛上不同汽油公司的需求。

22. 在现在法国的里尔河峡谷的一个洞穴中,大约于15000年前创作的一组画描绘了许多不同的动物。被描绘的一个动物看起来像查茹(一种稀有的羚羊)

下列哪一个,假如正确,最支持假说:在画类似查茹的动物时,那位洞穴艺术家画的是他或她所知道的动物?

A. 在相近时期的洞穴岩画中有许多虚构动物的展现。

B. 考古学家在峡谷北部发现了查茹化石,经检测,这些查茹存在于大约16000年前。


C. 含有类似查茹动物图形的洞穴也含有有特色的植物图形的展现。 D. 相同地区较老的洞穴中不含类似于查茹的动物的展现。 E. 画中动物的角比成熟的查茹的角长。

23. 随着莫罗湖沿岸工业的迅速发展,出现大量工业排污,莫罗湖水质日趋恶化,多种鱼类已经大量死亡,渔业生产受到灾难性影响。但由于莫罗鱼有适应污染水质的生存能力,因此,莫罗湖沿岸的莫罗鱼类加工业将不会受到严重影响。 以下哪项,如果是真的,将最严重地削弱上述论证?

A. 只要污染不再加重,个别鱼类也可能会逐渐适应污染水质。

B. 科学研究尚未弄清楚莫罗鱼类之所以能适应污染水质的生理机制。 C. 政府已经在着手治理莫罗湖的污染问题了。

D. 作为幼莫罗鱼主要食物来源的水生藻类无法在污染的水质中继续存活。

E. 莫罗湖地区的渔业资金、人力向莫罗鱼捕捞与加工业集中,竞争将日趋激烈。

24. 为了保护环境,政府下令停止审批、建造小造纸厂,已有的小造纸厂也必须在规定期限之内关、停、并、转。也就是说,今后造纸厂只有达到一定规模,才有可能得到政府有并部门的审批。

制定这一政策最主要的原因是:

A.大企业领导层的综合素质高,环保意识强。

B.企业的生产规模越大,污水处理费用摊到单位产品相对较低,企业可以承受。 C.大企业的技术力量雄厚,有能力处理污染问题。

D.大企业的资金实力雄厚,可以承受污水处理的费用。 E.大企业的厂址选择较为科学,可以减少污水的损害程度。

25. 假如一个人增加每天吃饭的次数,而且假如所吃食物的量没有显著增加,那么这个人的胆固醇含量显著下降。然而,大多数每日增加就餐次数的人也多吃更多的食物。 假如上面的话是正确的,下列哪一个最被上文所支持?

A. 对于大多数人,胆固醇的含量不被每天吃的食物的量所影响。 B. 对于大多数人,每顿饭吃的食物的量由吃饭的时间来决定。

C. 对于大多数人,增加每天吃饭的次数将不能导致胆固醇水平的显著下降。 D. 对于大多数人,每天吃饭的总量不被每天吃饭的顿数影响。

E. 对于大多数人,增加每天吃饭的顿数将导致吃的食物的种类显著变化。 答案:

01.D 02.D 03.C 04.D 05.B 06.E 07.D 08.D 09.E 10.B 11.D 12.A 13.E 14.C 15.E 16.C 17.E 18.C 19.B 20.E 21.D 22.B 23.D 24.B 25.C

01.环境科学家:在过去的10年中,政府对保护湿地的投资确实增加了6倍,而同时需要这样保护的土地面积只增加了两倍(尽管这些区域在10年前已经很大了)。即使把通货膨胀考虑进去,今天的资金数额也至少是10年前的3倍。虽然如此,目前政府对保护湿地的投资仍是不够的,政府的投资应该进一步增加。

下面哪一点,如果正确,最有助于使环境科学家的结论与引用的证据相一致?

A.负责管理湿地保护资金的政府机构在过去的10年中一直管理不当且运行效率较低。 B.在过去的10年中,那些被政府雇来保护湿地的科学家的薪水的增长比率高于通货膨胀的比率。

C.过去10年的研究使今天的科学有在潮湿土地遭到严重破坏的危险之前就把它们定为需要保护的对象。

D.今天,有更多的像科学家和非科学家的人在为保护包括湿地在内的自然资源而工作。 E.不像今天,10年以前对保护湿地的投资几乎是不存在的。


02.某餐馆发生一起谋杀案,经调查:

第一,谋杀或者用的是叉,或者用的是刀,二者必居其一。 第二,谋杀时间或者在午夜12点,或者在凌晨4点。 第三,谋杀者或者是甲,或者是乙,二者必居其一。 如果以上断定是真的,那么以下哪项也一定是真的?

(1)死者不是甲用叉在午夜12点谋杀的,因此,死者是乙用刀子在凌晨4点谋杀的。 (2)死者是甲用叉在凌晨4点谋杀的,因此,死者不是乙用叉在凌晨4点谋杀的。

(3)谋杀的时间是午夜12点,但不是甲用叉子谋杀的,因此,一定是乙用刀子谋杀的。 A.(l) B.(2) C.(3)

D.(1)(2)(3) E.(2)(3)

03.引自一篇报纸社论:许多有海都会洛因瘾的人最终试图戒毒,这主要基于两个原因:维持吸毒的开支和害怕被捕。如果海洛因被合法化且可以廉价取得,正像一些人所鼓吹的,那么这两个原因都不适用了。

以上考虑可最好地用于下列哪一论述?

A.使海洛因销售合法化可能导致其价格下降。

B.使用海洛因瘾的人容易获得戒毒治疗可能鼓励许多有海洛因瘾的人努力戒毒。 C.使海洛因销售合法化可能增加有海洛因瘾的人为买毒品而造成的犯罪。 D.使海洛因能够合法且廉价地获得,将使那些对海洛因上瘾的人不大可能试图戒掉他们的毒瘾。

E.降低对吸海洛因的个人惩罚的力度,不会增加新的染海洛因瘾的人数。

04.有人向某制衣厂经理提出一项建议:在机器上换上大号的缝纫线团,这样就可不必经常停机换线团,有利于减少劳动力成本。 这一建议预设了以下哪项?

A.大号缝纫线团不如小号的结实。

B.该厂实行的是计时工资制,不是计件工资制。 C.缝纫机器不必定期停机保养检修。

D.操作工人在工作期间不允许离开机器。

E.加快生产速度有利于提高该厂生产的衣服的质量。 05.根据古代记录,S市政府对基本商品征收的第一种税是对在S市出售的每一罐食用油征税两个生丁。税务纪录显示,尽管人口数量保持稳定且税法执行有力,食用油的税收额在税法生效的头两年中还是显著下降了。下列哪一项,如果正确,最有助于解释在S市油税收入的下降?

A.在税法实施后的10年,S市的平均家庭收入稳定增加。

B.在食用油税实行后的两年,S市政府开始在许多其他基本商品上征税。 C.S市,食用油罐传统上被用作结婚礼物,在税法实施后,食用油的礼物 增多了。

D.S市的商品,在税法实施后开始用比以前更大的罐子售油。

E.很少S市的家庭在加税后开始生产他们自己的食用油。

06.每年,一个消费者机构将所有国内航空公司按照在过去一年里飞机起降准时的表现排名,它所使用的惟一标准是每个航空公司晚点不超过15分钟的航班的比率。该机构不将因机械故障造成的延误计算在内,但去年因机械故障造成延误的航班的比率在所有国内


航空公司中大致相同,这一事实说明

下列哪项能最合逻辑地完成以上论述?

A.将因机构故障造成的延误包括在航空公司起降准时表现排名的计算之内,如果有影响的话,那么它对去年的排名只有极小的影响。

B.如果把机构故障造成的延误包括在准时表现的排名中,航空公司可能努力工作来减少延误。

C.该机构的排名没有给予消费者关于一个航空公司与另一家航空公司相比较关于其晚点的比率的准确信息。

D.去年有最好的准时表现记录的航空公司同时也有最大数目的机械原因造成的晚点。 E.去年准时表现对于所有国内航空公司大约相同。

07.某银行被窃,甲、乙、丙、了四人涉嫌被拘审。侦破结果表明,罪犯就是其中的某一个人。 甲说:“是丙渝的。 乙说:“我没偷。 内说:‘我也没偷。 丁说;‘如果乙没有偷,那么就是我偷的。

现已查明,其中只有一个说假话。从上述条件可以确定以下哪项成立? A.甲份。 B.乙份。 C.丙偷。 D.丁偷

E.推不出何人偷。

08.永久型赛马场的休闲用骑乘每年都要拆卸一次,供独立顾问们进行安全检查。流动型赛马场每个月迁移一次,所以可以在长达几年的时间里逃过安全检查网及独立检查,因此,在流动型赛马场骑马比在永久型赛马场骑马更加危险。

下列哪一项,如果对于流动型赛马场而言是正确的,最能削弱上面论述?

A.在每次迁移前,管理员们都拆卸其骑乘,检查并修复潜在的危险源,如磨损的滚珠轴承。

B.它们的经理们拥有的用于安全方面及维护骑乘的资金要少于永久型赛马场的经理们。 C.由于它们可用迁徙以寻找新的顾客,建立安全方面的良好信誉对于他们而言不是特别重要。

D.在它们迁移时,赛马场无法接收到来自它们的骑乘生产商的设备回收通知。 E.骑乘的管理员们经常忽视骑乘管理的操作指南。

09.每次核聚变都会发射出中子。为了检验一项关于太阳内部核聚变频繁程度的假设,物理学家们计算了在核假设正确的条件下,每年可能产生的中子数。他们再从这一点出发,计算出在地球某一特定地点应该经过的中子数。事实上,点数到的经过该地点的中子数要比预计的少得多,看起来这一事实证明了该假设是错误的,除了。 下列哪一项,如果正确,最能从逻辑上将上段补充完整?

A.理学家们应用了另一种方法来估计可能到达该地点的中子数,结果验证了他们最初的估计。

B.关于太阳核聚变反应频率还存在着其他几种竞争性假设。 C.太阳内部没有足够的能量来破坏它释放出的中子。

D.用来点数中子的方法仅发现了约不足10%的通过该地区的中子。 E.其他星球核聚变反应所出发的中子也到达了地球。




10.小张约小李第二天去商场,小李说:“如果明天不下雨,我去爬山。”第二天,天下起了毛毛细雨,小张以为小李不会去爬山了,就去小李的宿舍找他,谁知小李仍然去爬山了。待两人又见面时,小张责怪小李食言,既然天下雨了,为什么还去爬山;小李却说,他并没有食言,是小张的推论不合逻辑。

对于两人的争论,下面哪项论断是合适的? A.小张和小李的这个争论是没有意义的。 B.小张的推论不合逻辑。

C.两个人对毛毛细雨的理解不同。 D.由于小李食言,引起了这场争论。

E.由于小李的表达不够明确,引起了这场争论

11.欧洲的舞蹈批评家:美国芭蕾水平的提高是由于现在有更多的欧洲人在美国教授芭蕾。在美国的芭蕾教师中,在欧洲出生并接受训练的教师比例上升了,我知道这一点是因为在我去年去纽约时,我所遇见的欧洲来的芭蕾教师——在欧洲出生并接受训练——比从前要多。 下列哪一项指出了该舞蹈批评家在推理中所使用的一项有疑问的假设? A.该论述忽视了一种可能,即美国的一些芭蕾教师可能出生在欧洲但却是在美国接受的训练。

B.该论述假设该批评家在其去年去纽约时遇见的教师群在这类教师中具有典型代表性。 C.该论述假设美国的芭蕾教学水平比欧洲的要高。

D.没有考虑导致美国舞蹈家思想状态水平上升的其他可能原因。

E.该论述假设在欧洲出生并受识破的舞蹈家一般比在美国出生并受识破的舞蹈家天赋更高。

12.为保护海边建筑免遭海洋风暴的袭击,海洋度假地在海滩和建筑之间建起了巨大的防海墙。这些防海墙不仅遮住了一些建筑物的海景,而且使海岸本身也变窄了。这是因为在风暴从水的一边对沙子进行侵蚀的时候,沙子不再向内陆扩展。

如果上述信息正确,那些从其出发,下列哪一项得到了最有力的支持? A.由于海洋风暴的猛烈程度不断加深,必须在海洋和海边财产之间建立起更多的高大防海墙。

B.即使是在海滩被人类滥用着的时候,它们对于许多使用它们的野生物种的生存来说依然是必不可少的。

C.用来保护海边建筑的防海墙如果要保护那些建筑,它们自己最终不会被风暴破坏,不需要昂贵的维修和更新。

D.为以后的世代保留下海滩应该是海岸管理的首要目标。

E.对于一个想要维护自己海滩疗养地功能的海边社区来说,通过建筑防海墙来保护海边建筑的努力,从长远来看,其作用是适得其反的。

13.甲、乙、丙三人讨论“不劳动者不得食”这一原则所包含的意义。 甲说:“不劳动者不得食,意味着得食者可以不劳动。 乙说:“不劳动者不得食,意味着得食者必须是劳动者。 丙说:“不劳动者不得食,意味着得食者可能是劳动者。 以下哪项结论是正确的?

A.甲的意见正确,乙和丙的意见不正确。 B.乙和丙的意见正确,甲的意见不正确。 C.甲和丙的意见正确,乙的意见不正确。 D.乙的意见正确,甲和丙的意见不正确。


E.丙的意见正确,甲和己的意见不正确。 14.加拿大人现在越来越多地加入“境外购物”,即越过国界到价格较低的地方购物。加拿大以外的价格要低很多,很大一部分原因是支付给加拿大社会服务体系的商品和服务税不再适用。

根据以上的信息,下面哪个说法最可以被支持? A.如果境外购物的上升趋势继续保持在较高的水平,并且政府支付给加拿大社会服务体系的金额不变,估计加拿大的商品和服务税税率就会上升。 B.如果加拿大对从境外购买的商品征收较多的关税,另一个方向上别国也会相应地对从加拿大购买的商品征收关税,从而损害加拿大的商业。

C.加拿大政府支付给为加拿大人提供社会服务的人的资金数额一直在增加。 D.同样品牌的商品,加拿大顾客在境外和在国内都可以买到。

E.境外购物所购商品在购物者越过边界进入加拿大境内时,要交纳加拿大规定的税收。 15.某国政府决策者面临的一个头痛的问题就是所谓的“别在我家门口”综合症。例如,尽管民意测验一次又一次地显示公众大多数都赞成建造新的监狱,但是,当决策者正式宣布计划要在某地建造一新的监狱时,总遭到附近居民的抗议,并且抗议者往往总有办法使计划搁浅。以下哪项也属于上面所说的“别在我家门口”综合症?

A.某家长主张,感染了艾滋病毒的孩子不能允许入公共学校。当知道一个感染了艾滋病毒的孩子进入了他孩子的学校,他立即办理了自己孩子的退学手续。 B.某政客主张所有政府官员必须履行个人财产公开登记,他自己递交了一份虚假的财产登记表。

C.某教授主张宗教团体有义务从事慈善事业,但自己拒绝捐款资助索马里饥民。 D.某汽车商主张国际汽车自由贸易,以有利于各国经济,但要求本国政府限制外国制造的汽车进口。

E.军事战略家认为核战争足以毁灭人类,但主张本国保持足够的核能力以抵御外部可能的核袭击。

16.左撇子的人比右撇子的人更经常患有免疫功能失调症,比如过敏。但是左撇子往往在完成由大脑右半球控制的任务上比右撇子具有优势,并且大多数人的数学推理能力都受到大脑右半球的强烈影响。

如果以上的信息正确,它最能支持下面哪个假设?

A.大多数患有过敏或其他免疫功能失调症的人是左撇子而非右撇子。 B.大多数左撇子的数学家患有某种过敏症。

C.数学推理能力强于平均水平的人中,左撇子的人的比例,要高于数学推理能力弱于平均水平的人中的左撇子比例。

D.如果一位左撇子患有过敏症,他很可能擅长数学

E.比起左撇子的人或者数学推理能力不寻常地好的人所占的比例来讲,患有过敏等免疫功能失调症的人的比例要高一些。

17.M公司的最新产品成本是如此之低,以至于公司不大可能在出售产品时不增加公司通常允许赚取的成本加价:潜在的客户可能完全不能相信这么便宜的东西会真好使。但M公司的信誉是建立在仅包括合理的边际利润的公平价格基础上的。 以上的论述,如果正确,最强有力地支持了下面哪个? A.M公司在试图为其最新产品定价、使价格能在不损害公司信誉的前提下促进销售时会遇到困难。

B.尽管售出的每件产品利润很小,但通过大规模的销售,M公司仍取得了巨大的年利润。 C.M公司在为其最新产品计算生产成本时犯了计算错误。


D.M公司的最新产品将要执行的任务是其他制造成本更低的设备也能胜任的。 E.M公司的生产程序的设计和该公司制造的产品一样具有新颖之处。

18.报上登出了国内20家大医院的名单,名单按它们在近3年中病人死亡率的高低排序。家指出不能把名单排列的倾序作为评价这些医院的医疗水平的一个标准。 以下各项,如果是真的,都能作论据支持专家的结论,除了

A.20家医院中,有5家依靠国家资助从国外进口了多项先进、大型和配套的医疗设备,其余的都没有。

B.有些医院,留病人住院的时间长,病人死亡率因此就较高;有些医院,往往较早地动员患绝症而救治无望的病人出院,病人死亡率因此就较低。 C.20家医院中,有2家老人医院和3家儿童医院。 D.20家医院中,有2家是肿瘤医院。

E.有些医院不具备特种手术和特别护理条件,碰到相关的病人就转院了事。

19.大学图书管理员:三年以前,非学生读者使用本图书馆是免费的。后来,因为我们的预算减少了,所以我们要求他们每年支付100美元的费用。然而,仍然约有150名非学生读者使用了图书馆而没有缴费。因此,如果我们雇用一名警卫来辨认非学生读者并令其缴费,么我们就可以获得经济收益。

下列哪一项对于评价图书管理员的结论是最为重要的? (A)每年使用图书馆的学生人数。 (B)今年图书馆的预算。

(C)图书馆是否安装了昂贵的计算机分类系统。 (D)三年前图书馆的预算降低了多少。

(E)图书馆雇用一名警卫每年的成本是多少。 20-21题基于以下题干:

某仓库被窃。经过侦破,查明作某的人部是甲、乙、丙、丁四个人中的一个人。审讯中,四个人的口供如下: 甲:“仓库被窃的那一天,我在别的城市,因此我是不可能作案的。 乙:“丁就是罪犯。 丙:“乙是盗窃仓库的罪犯,因为我亲眼看见他那一天进过仓库。 丁:“乙是有意陷害我。

20.现假定这四个人的口供中,只有一个人讲的是真话。那么 A.甲是盗窃仓库的罪犯。 B.乙是盗窃仓库的罪犯。 C.丙是盗窃仓库的罪犯。 D.丁是盗窃仓库的罪犯。

E.甲、乙、丙、丁都不是盗窃仓库的罪犯。

21.现假定这四个人的口供中,只有一个人讲的是假话。那么 A.甲是盗窃仓库的罪犯。 B.乙是盗窃仓库的罪犯。 C.丙是盗窃仓库的罪犯。 D.丁是盗窃仓库的罪犯。

E.甲、乙、丙、丁都不是盗窃仓库的罪犯。

22.要成为一名成功的商业经理需具备一定的天赋,商业课程可以帮助人们解决管理问题,但这种课程仅能帮助那些具有管理天赋的人,这些人应该通过商业课程来获得一些方法,果这些管理问题恰巧发生时,他们就可以很好地应用这些方法。


如果上文论述正确,从其出发,下列哪一项也一定正确?

A.那些在解决管理问题方面受益于商业课程的人也具有管理天赋。

B.那些在解决管理问题方面已经很有办法的人不可能从商业课程中受益。 C.大多数成功地解决了管理问题的方法是从商业课程中学到的。

D.缺乏管理天赋的人与那些具有管理天赋的人相比,更有可能去学习商业课程。 E.那些没有学过商业课程的人在管理问题发生时无法解决这些问题。

23.需要一种特殊棉纱带的惟一目的是把某些外科伤口包扎10天——这些伤口需要包扎的最多天数。纽太是这种棉纱带的一个新品牌。纽太的销售人员声称纽太会有助于伤口愈合,为纽太的粘附能力是现在使用的棉纱带的2倍。

下面哪种说法,如果正确,会对纽太的销售人员的声明提出最严重的疑问? A.大多数外科伤口大约需要10天愈合。

B.大多数外科棉纱带是由医院和诊所而非私人外科医生购买的。

C.现在使用的棉纱带的粘附能力远超过包扎伤口10天所需的足够的粘附能力。 D.纽太和现在使用的棉纱带都不能很好地粘附未经洁净的皮肤。

E.纽太对已涂上一种特别化学药剂的皮肤的粘附能力仅为现在使用的棉纱带对那种皮肤的粘附能力的一半好。

24.一项研究发现,1970年调查的孩子中有70%曾经有过牙洞,而在1985年的调查中,仅有50%的孩子子曾经有过牙洞。研究者们得出结论,在19701985年这段时间内,孩子们中的牙病比率降低了。

下列哪一项,如果为真,最能削减弱研究者们上面得出的结论? A.牙洞是孩子们可能得的最普通的一种牙病。 B.被调查的孩子来自不同收入背景的家庭。

C.被调查的孩子是从那些与这些研究者们进行合作的老师的学生中选取的。

D.1970年以来,发现牙洞的技术水平得到了突飞猛进的提高。E.平均来说,1985年调查的孩子要比1970年调查的孩子的年龄要小。 25.通过检查甲虫化石,一研究小组对英国在过去2.2万年内的气温提出了到目前为止最为详尽的描述。该研究小组对现存的生物化石进行挑选,并确定了它们的日期。当发现在同一地方发现的几种生物的个体属于同一时间段时,现存的甲虫类生物的已知忍受温度就可以被用来决定那个地方在那段时间内的夏季的最高温度。 研究者的论述过程依赖于下面哪一条假设?

A.甲虫忍耐温暖天气的能力比忍耐寒冷天气的能力强。 B.在同一地方发现的不同物种的化石属于不同的时期。 C.确定甲虫早期的方法比确定其他生物日期的方法准确。

D.一个地方某个时期的实际最高夏季气温与在那个地方那段时间发现的每种甲虫类生物的平均最高可忍受气温相等。

E.在过去的2.2万年的时间内,甲虫类生物的可忍受气温没有明显变化。 参考答案:

01.E 02.B 03.D 04.B 05.D 06.A 07.D 08.A 09.D 10.B 11.B 12.E 13.D 14.A 15.D 16.C 17.A 18.A 19.E 20.A 21.B 22.A 23.C 24.E 25.E

1.一天,营长在全营会上讲话说:不想当将军的士兵不是好士兵。士兵小张问:那您的意思是说我们都可以成为将军了。营长讲话的意思是: A.想当将军的士兵就一定是好兵。 B.除非想当将军,否则不是个好士兵。


C.坏士兵是不想当将军的。 D.坏士兵也是想当将军的。

E.不想当将军的土兵,也可以是一个好士兵。

2.一些人对某法官在针对妇女的性别歧视的案件中的客观性表示质疑。但是记录显示,在这种案件中有60%的情况下该法官做出了有利于妇女的判决。这项记录证明,在针对妇女的性别歧视的案件中,该法官并没有歧视妇女。

上述论述是有缺陷的,因为它忽略了一种可能性,即

A.由该法官审理的大量案件是由对妇女的性别歧视的指控引起的。 B.许多法官发现在对妇女性别歧视的案件中很难做到客观公正。 C.在不牵涉性别歧视的案件中该法官对女被告或原告有偏见。

D.送到该法官所在法庭的针对妇女的性别歧视案件多数是从低级法院上诉来的。

E.有证据显示该法官在关于针对妇女的性别歧视案件中,妇女应该赢得超过60%的案件。 3.快速周转时间是一项策略,通过对生产组织的设计来消除生产中的瓶颈和延迟。它不仅加速生产,而且确保质量,理由是除非所有的工作第一次就被全部正确地完成,否则瓶颈和延迟不能被消除。

以上做出的关于质量的论述是建立在一个值得怀疑的假设的基础上,即 A.生产一种产品的过程中的任何缺陷都会造成瓶颈或延迟,而在快速周转生产线中可以阻止这种情况发生。

B.快速周转时间的策略要求从根本上重新思考产品的设计。

C.该组织的首要目标是生产一种质量无与伦比的产品,而不是为股东创造利润。 D.通过节省生产周期中每一个组成过程的时间可以成功地实施6J陕速周转时间。 E.快速周转时间是一个迄今尚未在任何一个工厂实施过的商业战略。 4.两个基于不同原理的检测系统,每一个都能测出全都有瑕疵的产品,但也会错误地抛弃3的无瑕疵的产品。假设两个系统错误抛弃的产品没有重叠,且如果同时运行也不会相互干扰,则使用两个系统并仅抛弃那些两个系统都认为有瑕疵的产品可以避免所有的错误抛弃。 下列哪一项最精确地描述了上面论述中的推理过程?

A.该推理是结论性的,如果支持结论的陈述正确,那么结论就不可能错。

B.该推理很强,但不是结论性的,如果支持结论的陈述正确,那么就为结论提供了很好的根据,尽管附加的信息可能会削弱该论述。

C.该推理很弱,支持结论的陈述,尽管与结论有关,但陈述本身最多只为该结论提供了不充分的根据。

D.该推理有缺陷,因为该结论也只是对支持该结论的一项证据的重新阐释而已。

E.该推理有缺陷,因为该论述把一个因素是某一事件发生的必要条件的证据当作了该事件发生的充分条件的证据

5.毫无疑问,向没有核武器的国家出售钚是违反国际法的,但如果美国不这样做,其他国家的公司会这样做。

以下哪一项同以上论述在逻辑结构方面最为相似?

A.毫无疑问,同绑架者谈判是违反警察部门的政策的。但如果警察想阻止生命损失,他们在某些情况下必须同绑架者谈判。

B.毫无疑问,拒绝登记服兵役是违法的。但在美国有一项历史很久、可以做到尽责地拒绝在军队服役的传统。

C.毫无疑问,一个政府官员参与一项有明显利益冲突的交易是违法的。但如果将事实调查得更清楚一些,就可以很清楚地看到实际上在被告方面没有利益冲突。

D.毫无疑问,夜间潜入别人的住宅是违法的。但如果被告不先这样做,总会有别的某个人做


夜盗潜入该房子。

E.毫无疑问,该公司的政策禁止管理者在没有两次书面警告的情况下将雇员解雇,但有很多管理者不遵守这项政策。

6.Simon:我们仍不知道机器是否能够思考。计算机能够执行非常复杂的任务但是却缺乏人类智慧的灵活特性。Roberta:我们不需要更复杂的计算机来弄清机器是否能够思考,我们人类就是机器,而我们可以思考。

RobertaSimon的回应基于对哪个词的重新解释? A.计算机B.知道C.机器D.复杂的E.思考

7.飞机制造商:我反对你把我们的X-387型喷气机描述为危险的。商业使用的X-387飞机从未坠毁,也未曾有过严重的功能失调。

7航空调度员:X-387飞机的问题并不在于其自身,而在于发动起来时会引起空气湍流,给附近的飞行器造成危险的环境。

航空调度员通过下面哪一项对制造者做出了回答?

A.把制造商的论断特征描述为来自主观兴趣,而不是来自于对事实的客观评价。 B.把注意力集中于这个事实,制造商对危险的阐释太狭隘了。 C.引用一些制造商把它们当作与争论问题无关而明显忽略的证据。 D.引用统计证据以反驳制造商的断言。

E.向制造商对最近空难数量的了解程度提出质疑。

8,皇帝:大海另一边的敌国几个世纪以来一直骚扰我们,我想征服它并且一劳永逸地消除这种骚扰。你能给我什么建议?

海军上将:如果你穿过大海,一个强大的帝国将会衰落。 皇帝:那样的话,准备部队。今天晚上我们就出海。 在下面选项中,对皇帝决定人侵的最强有力的批评是 A.必定导致那个皇帝的失败。

B.基于不是关于军队强弱的客观事实的观点。 C.与海军上将的陈述相冲突。

D.没有充分考虑海军上将的建议的可能的意义。 E.对解决即将发生的问题来说是一个无效的策略。 9.不管你使用哪种牙膏,经常刷牙将降低你牙齿腐烂的可能性。科学家得出结论:当刷牙时,你通过去除牙齿与牙龈上所形成的牙菌斑薄片减少牙齿腐烂。因此你可以不用加氟牙膏,要认真刷牙就能告别蛀牙。

下面哪一项是对上面论述推理的批评? A.用加氟牙膏刷牙表明可以降低牙齿腐烂。

B.刷牙将减少牙齿腐烂的事实并没有表明氟无价值。 C.几乎没有人通过刷牙可以充分地消除牙菌斑。

D.在绝大多数的时间内,人们的牙齿上都有牙菌斑。 E.科学家关于氟的说法是错误的。

10.地区磁场发生磁暴的周期经常与太阳黑子的周期一致。随着太阳黑子数目的增加,磁暴的强度增大。当太阳黑子的数目减少时,磁暴的强度降低。所以科学家推测,太阳黑子的出现可能是磁暴的原因。

下面哪一项与上面所使用的推理方法相同?

A.闪电愈强,雷声就愈大;反之,闪电愈弱,雷声就愈小。由此可见,闪电是雷声的原因。 B.在一定限度内,密植可以提高粮食产量。因为,农作物的密度大一点,产量就高一些;密度再大一点,产量可以再高一些。


C.通过温度和压力的同时变化观测气体体积与两者之间的关系。

D.冬至时,气温很低,白昼最短,随着气温升高,白昼变长;夏至时,气温很高,白昼最长。所以气温高低是白昼长短的原因。

E.集中优势兵力,各个歼灭敌人,是战略和战术的一条基本原则。

11.1908年,清朝3岁的溥仪继皇帝位称宣统皇帝,接受文武百官的朝贺。钟鼓齐鸣,三呼万岁,把宣统皇帝吓得直哭。抱着宣统皇帝的摄政王安慰小皇帝说:快完了,快完了。后来,清王朝于1911年被辛亥革命推翻。清朝的遗老遗少怪罪摄政王说,就是他在登基大典上说快完了,所以把大清朝的江山给葬送了。

以下的哪一项与清朝的遗老遗少的推理方法相似?

A.这个码头坍塌,固然与建筑的质量有关,但与今年潮水过大也有一定的关系。 B.这座大桥被冲垮了,完全是由于百年未遇的洪水的缘故。 C.兴达公司如此兴旺发达,完全是这个公司的名字取得好。 D.暂时没有攻克这个难关,是由于我们掌握的资料还不完全。 E.只要真理在我们手里,就没有什么困难可以阻止我们取得胜利。

12.研究在不同环境下分别养大的同卵双胞胎的研究者发现,每一对这种双胞胎在性格上、医疗记录和生活经历上都相似。这些研究者认为这些结果证实了这种假设:遗传在决定人的性格和生活经历时比环境起更大的作用。

以下哪项如果为真,将最严重地削弱上述假设?

A.一对在一起抚养的同卵双胞胎,经适当的测试显示出非常接近的价值观。 B.一对不在一起扶养的同卵双胞胎,他们在攻击性和其他个人特征上明显不同。 C.一对在一起抚养的姐弟,他们有相似的性格和生活经历。 D.一对母女,虽然性情不同,但从事同样的职业。

E.一对在一起抚养的双胞胎,有着相似的个性特征但有不同的价值观。 13.一项每年进行的全国性的调查表明,在过去30年里高中的高年级学生对非法药品的使用呈持续而明显的下降。

要想从上面描述的调查结果得出结论认为20岁以下的人对非法药物的使用正在下降,下面哪项,如果正确,能提供最多的支持?

A.高中的高年级学生们使用非法药品的水平的变化很少与20岁以下的其他人使用非法药品的水平的变化相符。 B.在过去,高中的高年级学生一直是最可能使用非法药品和最可能大量使用非法药品的人群。

C.高中的高年级学生使用非法药品的比例与所有的20岁以下的使用非法药品的人的比例非常相似。

D.这项调查显示的下降趋势是特别针对20岁以下的人进行的药物教育计划的结果。 E.比起使用非法药品的人数中承认出售过非法食品的人数下降得更快。

14.较普遍的假设认为,当一个慷慨的赞助人捐赠了有潜力的展品时,博物馆得到了财政上的支持。但是在事实上,捐赠物品需要储藏空间,那不会是免费的,还需要非常昂贵的日常维护。所以,这些赠品加剧了而非减轻了博物馆财政资源的需求。 下面哪个,如果正确,最严重地削弱了以上的论证? A.为了妥善地安排赠品,博物馆明智的做法是至少把一些捐赠品进行展览而不是仅仅储存起来。

B.最可能向博物馆捐赠值钱的物品的人也是最可能向博物馆捐赠现金的人 C.博物馆不能因为展品有潜力而收藏,因为这么做会提高维护费用。

D.赞助人希望博物馆能把捐赠物保持在其财产中而不是出售它们来筹集现金


E.赞助人给博物馆捐赠的物品常常如此重要以至于博物馆被迫在必要时通过购买来把其加人到收藏中。

15.大多数地理学家相信,石油是埋在古代海洋下面的有机物所产生的碳氧化合物的化学变物。与之不同,有人假设石油实际上产生于在地球内分离的其他复杂的碳氢化合物上面的细菌活动。众所周知,这些碳氢化合物的规模超过了被埋有机物的规模。所以,我们的石油储备要多于大多数地理学家所认为的。

下面哪个,如果正确,对以上关于他们的石油储备的论证提供了最强有力的支持? A.大多数地理学家对地球的石油储备持乐观态度。

B.大多数地理学家已经对以往发现石油储备进行了精确的化学分析。 C.古代的海洋在许多地方被埋到地里了,那里的化石非常丰富。

D.已经在石油储备中发现的惟一细菌很可能是从地面污染物中通过钻井渗透下来的。 E.化学变化减少了来自有机物质被埋碳氢化合物的规模,减少的比例与细菌活动对其他复杂碳氢化合物的规模的减少比例大致相同。

16.从比例上讲,被诊断患有大脑紊乱精神分裂的人中,出生在冬季月份的人比出生在一年中其他时间的人多。最近的一项研究显示,其原因可能是一些母亲在一年中最冷的几个月中的营养不良,在这一时期,人们最难买到或买得起多种新鲜食品。 下面哪个,如果正确,能帮助支持上述的结论?

A.几年以来,精神分裂症的发病率并未显示出与经济萧条的程度有关。

B.大部分精神分裂症中染病的脑部区域是在母亲怀孕期的最后一个月发育的。 C.冬季的自杀率明显高于其他季节。

D.新鲜食品中的营养与冷藏食品中的营养对脑部发育产生的效果相同。 E.相当一部分被调查研究的病人有精神分裂症的家族病史。

17尽管象牙交易已被国际协议宣布为非法行为,但是,一些钢琴制造者,仍使用象牙来覆盖钢琴键,这些象牙通常通过非法手段获得。最近,专家们发明了一种合成象牙,不像早期的象牙替代物,这种合成象牙受到了全世界范围内音乐会钢琴家的好评。但是因为钢琴制造者从来不是象牙的主要消费者,所以合成象牙的发展可能对抑制为获得最自然的象牙而捕杀大象的活动没什么帮助。

下面哪一项,如果正确,最有助于加强上述论述? A.大多数会弹钢琴,但不是音乐会钢琴家的人也可以轻易地区分新的合成象牙和较次的象牙替代物。

B.新型的合成象牙被生产出来,这种象牙的颜色、表面质地可以与任何一种具有商业用途的自然象牙的质地相似

C.其他自然产物,如骨头和乌龟壳被证明不是自然象牙在钢琴键上的替代物。

D.自然象牙最普遍的应用是在装饰性雕刻品方面。这些雕刻品不但因为它们的工艺质量,而且因为它们的材料的真实性而被珍藏。

E.生产新型象牙的费用要比生产科学家们以前开发的任何象牙替代品的费用低得多。 18.有些地理学家认为如果世界上未勘探过的地区的石油与已勘探过地区的石油一样普遍,那么我们现在对地下石油储备的估算,必须要乘以万倍了。由此我们可以得出结论,假定未来的石油消费加速增长,我们至少可以再满足整个世界5个世纪的石油需求。 为了得出上述结论,作者必须假设下面哪个? A.人们有可能在未开发地区再发现石油。 B.石油的消费速度不会迅速增长。

C.在至少500年,内,石油仍将是重要的能量来源。 D.世界人口将实现并保持零增长。


E.新技术将使石油的发现和钻井比以前更可行。

19.去年,美国政府研究基金的总预算里只有3%是扶持社会和行为学科的。因此,今年在这些项目上资金的锐减,与其说是经济压力的原因,不如说是社会观念的原因。 下面哪一个是上文的结论所依据的假设?

A.分配给社会行为学科的研究资金不足以完成必需的工作。 B.社会行为学科与物理学生物学具有同等价值。 C.目前资金的缩减将中止社会行为学科的研究。

D.在美国,政府拨款是研究工作所需资金的首要来源。

E.3%是政府用于科研工作的总体预算中微不足道的一部分。

20.宇宙中的大多数物质都被假定是暗的,即见不到的。研究已经表示,如果很多星系群中的星系组成物质是可见的恒星,那么相对于其他星系组成物质的移动速度就会快得多。这一研究表明星系在相当大量的不可见物质的引力作用下运动。 下面哪个是上文依据的假设?

A.运动星系速度的测量是极不可靠的。 B.重力的作用形式没有被很好地理解。

C.对于上述星系中可见恒星的总的质量的估计是有一定把握的。 D.宇宙中不可见物质的大体组成已经被确定了。 E.上面提到的星系都毫无例外地向彼此移动。

21.3月,300名大学生在华盛顿抗议削减学生贷款基金的提案,另外有35万大学生在3期间涌向佛罗里达的阳光海滩度春假。因为在佛罗里达晒太阳的人数要多一些,所以他们比在华盛顿抗议的学生更能代表当今的学生,因此国会无需注意抗议学生的呼吁。 上面的论证进行了下面哪个假定?

A.在佛罗里达度假的学生不反对国会削减学生贷款基金提案。

B.在佛罗里达度假的学生在削减学生贷款基金提议问题上与大多数美国公民意见一致。 C.在华盛顿抗议的学生比在佛罗里达度假的学生更关心其学业。

D.既没去华盛顿抗议、也没有去佛罗里达度假的学生对政府的教育政策漠不关心 E.影响议会关于某个政治问题的观点的最好方法是华盛顿与其选举出来的代表交流。

22.有些末日论者警告说,天气形势长期转暖或转冷的趋势都将大量减少谷物产量。但是,比较乐观的报告指出,即使平均气温的这种变化真的发生,我们可以预期谷物产量不会有太大变化,因为几乎没有迹象表明降雨量会改变。此外,对大多数庄稼来说,气候导致的产量变化将被年产量的波动和科技因素引起的产量增加而掩盖。 下面哪项是上文提到的较乐观报告所基于的假设? A.天气形势长期的变化无法被准确地预测。

B.谷物的生产高度依赖于科技因素,以至于不论气候条件如何,产量提高的可能性都不大。 C.降雨量的变化趋势比温度变化趋势更难孤立地去考虑。 D.长期的转暖或转冷趋势如果伴随着降雨形势的变化,其对谷物产量的破坏比没有降雨形势的变化时更大。

E.长期转冷趋势比长期转暖趋势对谷物产量的潜在破坏更严重。 23.20世纪70年代出现了大学毕业生的过度供给,过度的供给使大学毕业生的平均年收入降到了比只持有高中文凭的工人仅高18%的水平。到了20世纪80年代,大学毕生的平均年收入比只持有高中文凭的工人高43%,尽管20世纪?0年代到80年代后期大学毕业生的供给量没有下降。

下面哪个,如果在20世纪80年代后期是正确的,最好地调解了上述明显的分歧? A.经济放慢了,从而使对大学毕业生的需求减少了。


B.高中教育的质量提高了。

C.20世纪70年代相比,更多的高中为它们的学生提供了职业指导计划。 D.至少受过一种大学水平教育的人所占的比例上升了。

E.20年来第一次出现了仅有高中文凭的求职者的过度供给。 24.下面哪个,如果正确,最好地完成了以下的论证:

对两个国家的居民平均生活水平的比较可以反映居民获取产品和服务的相对情况。以一国货币表示的其居民的平均收人的可靠数字可以轻易得到,但从这些数字中很难得到平均生活水平的准确比较。因为

A.通常没有数字来比较为购买一定数量的产品和服务需要花费多少两种不同货币。 B.同样工作的工资水平,因依赖于文化和纯粹的经济因素而在各国之间相差很大。 C.这些数字必须用一国的国民总收入除以其人口来计算。

D.获取产品和服务的相对情况只是决定生活质量的若干相关因素中的一个。 E.一国居民的财富以及生活水平与他们的收入紧密联系着。

25.1975年以来,美国的麻疹等传统儿童疾病的发病率已经有了显著的下降。这一下降的同时伴随着儿童中间Peterson病——一种迄今为止罕见的病毒感染——发病率的上升。但是,很少有成年人被这种疾病侵袭。

下面哪个,如果正确,最能帮助解释儿童中间Peterson病发病率的上升? A.遗传因素部分决定了一个人易受导致Peterson病的病毒感染的程度。

B.传统儿童疾病的减少和与之相随的Peterson病的增加在其他任何国家没有发现。 C.得过麻疹的儿童形成了对导致Peterson病的病毒的免疫力。

D.儿童时期没有得过麻疹的人到成年时可能得麻疹,在这种情况下,疾病的后果一般会更加严重。

E.那些得了Peterson病的人得水痘的危险增加了。 参考答案

1.D 2.C 3.D 4.D 5.A 6.E 7.A 8.D 9.A 10.E 11.D 12.B 13.E 14.E 15.A 16.C 17.C 18.C 19.B 20.C 21.E 22.D 23.B 24.E 25.C

01.临海市消费者协会公布的牛奶比较试验报告中说:从整个抽查的情况看,牛奶的质量基本上是合格的,国产和合资厂生产的品牌,不仅大部分指标符合国际的要求,而且价格上有很大的优势,但也有某些品牌个别指标存在问题。另外,在抽检中没有发现明显的掺假现象,只是个别品牌的牛奶加水过量。

以下各项中有哪一项不能从上面的内容中经过合理推断得出?

A. 国产品牌的牛奶中仍然有一些品牌的个别指标不符合国际要求。 B. 合资厂品牌牛奶的价格优势比起国产的牛奶品牌来更有竞争力。 C. 在抽查中,个别品牌牛奶的加水过量算不上明显的掺假现象。 D. 总体来看,国产和合资的品牌的质量已经越来越接近国际要求。 E. 消费者对于临海市销售的不同品牌的牛奶基本可以买起来放心。

02-03为了避免戊型肝炎的流行,血液中心将立即开始筛选所有的献血者。尽管新的筛选测试估计将从献血者中筛选掉5%却仍然会放过三分之二的戊型肝炎携带者。因此,大约10%的献血者将仍然提供带有戊型肝炎病菌的血液。 02.上面的推论基于以下哪项假设?

A.在大量的案例中,携带戊型肝炎的献血者并不会发病。

B.对被戊型肝炎测试筛选下来的献血者数字的估计是被低估了的。 C.在大量的案例中,携带戊型肝炎病菌的献血者并没有携带其它常规测试能够筛选出来的相关病菌。


D.在想要献血的人中,戊型肝炎患者的比例比在整个人口中的比例要低。

E.仍然提供带有戊型肝炎病菌血液的献血者的献血次数要比整个献血者献血的平均次数低。 03.根据上文我们能够合乎逻辑地推出以下哪项可能的后果? A. 因为输血而传染上戊型肝炎的病例会比以前增加。 B. 血液中心可能供应的血液量似乎要下降。 C. 病人为自己用而献血可能会大大减少。 D. 初次献血者的人数可能会上升。

E. 血液中心对血液的需求量似乎要急剧上升。

04.为了吸引新的消费者,从竞争对手中将顾客夺过来,客隆超级市场开始对在本店一次购50元或50元以上的顾客折扣优惠。客隆的总经理认为,这个折扣的措施取得了巨大的成功,因为根据收银员的统计,自从措施开始实施后,50元或50元以上的买卖增加了30% 以下各项如果为真,哪项最能削弱上述的推论? A.在该打折措施开始实施后,大部分50元或超过50元的买卖是那些从没有到过客隆超级市场的人们所为,并且他们的平均帐单是高于50元的。

B.在该打折措施开始实施后,大部分50元或超过50元的买卖是那些老顾客,他们的平均帐单比他们以往的记录要高。

C.打折开始后改到客隆来购物的人们几乎每一次都花不止50元钱。

D.那些打折后仍然只买50元以下的商品的顾客,可能永远都不会被这种折扣措施所吸引。 E.大部分在打折阶段购买了50元或50元以上商品的顾客,在打折结束后还会到客隆超级市场来购物。

05.学校里最近成立了“工商管理同学会”,所有的MBA在校学生都加入了这个组织。小贺参加了工商管理同学会,所以他一定是一名在校的MBA学生。 以下哪项最好地指出了上述论证过程的逻辑错误?

A.小贺参加工商管理同学会是因为他希望参加而不是因为他的身份。 B.工商管理同学会的成员中也有已经毕业的MBA C.小贺符合参加工商管理同学会的全部条件。

D.工商管理同学会的成员一旦加入,不能够随意退出。

E.小贺对工商管理这个专业非常喜爱,是工商管理同学会的发起人之一。

06.持续的室内人造光照对患有先天性心脏病的小白鼠的健康很有好处。将连续接受室内人造光照的一组心脏病小白鼠与另一组交替接受室内人造光照和黑暗环境的类似的心脏病小白鼠进行对比试验,前者比后者的存活期平均要长25%

上面所描述的这种研究方法提醒我们可以在以下那种问题中也进行尝试?

A. 是否可以观察一下产业工人是否在日光下和人造光下的工作效果是否不同? B. 有没有植物需要特定的黑暗环境来生长? C. 深海鱼是否能在完全黑暗的环境中生存? D. 小白鼠得的是什么样的先天性的疾病?

E. 是否医院里可以用持续的人造光来改善对病人的治疗?

07.心理研究表明,在大学里,足球、篮球运动员比起非接触性的运动(如游泳)的运动员更容易变得充满敌意和进攻性。但是研究者的结论——身体接触性的运动比起非接触性运动更鼓励和熏陶参加者的敌意和进攻性--不能站住脚。说不定足球和篮球的运动员们一开始就比游泳运动员更多一些敌意和进攻性,这谁又敢保证呢?

下面各项如果为真,哪项能够最有力地支持这些心理研究者的结论?

A.足球和篮球的选手们在一个赛季里会变得更有敌意和进攻性,然后在非赛季中并不增加,只是保持那个水平。游泳运动员在赛季中并没有这样的在进攻性上的增强。


B.足球和篮球运动员在测试进攻性和敌意的试验开始时都有些害怕,而游泳运动员却不会。 C.同样的心理学研究表明篮球、足球运动员对于合作和团队精神有更多的理解;而游泳运动员只是关心个人的竞争对手。

D.这个研究在设计时就包括了身体接触性运动和非接触性运动这两类的参加者。 E.在中国,更多的有暴力倾向的球迷出现在足球场而不是游泳场地。 08-09学校的领导中有一些是足球迷。

学校行政管理主管准备把学校的足球场改建为一个科贸写字楼,以改善学校收入状况。 所有的足球迷都反对将学校的足球场改建成什么科贸写字楼。 08.如果以上各句陈述均为真,则下列哪项也必为真? A.所有的学校领导都是学校行政管理主管。 B.有的学校领导不是学校行政管理主管。 C.有的学校行政管理主管是足球迷。

D.并不是所有的学校行政管理主管都是学校领导。 E.有的足球迷是学校行政管理主管。

09.如果作为上面陈述的补充,明确以下条件:所有的学校领导都是足球迷,那么下列哪项一定不可能是真的?

A. 有的学校领导不是学校行政管理主管。 B. 学校行政管理主管中有的是学校的领导。 C. 并不是所有的足球迷都是学校的领导。

D. 所有的学校领导都反对将学校的足球场改建为科贸写字楼。 E. 有的足球迷不是学校的行政管理主管。

10.小朱和小李在讨论莎士比亚作品真正的作者问题。 小朱:那些被认为是没有受过良好教育的农村土包子莎士比亚所写的戏剧和诗歌其实出自伊丽莎白一世女王之手。只有伊丽莎白一世女王才有完成这些作品的学识和才智。

小李:你的话我不同意。如果伊丽莎白一世写了哈姆雷特和麦克白这样的戏剧的话,她将会很快被认为是历史上最伟大的作家,而事实上她却没有获得这样的声誉。 小李认为以下哪项是不可能的?

A. 伊丽莎白一世的写作生涯能够成为一个秘密。

B. 没有受过教育的人也可能写出象哈姆雷特和麦克白这样的戏剧。 C. 伊丽莎白一世具有获得伟大的文学成就所必需的天赋。

D. 教育和才智作为有创造力的作家必需具备的条件,是密不可分的 E. 伊丽莎白一世时期的女性不可能成为伟大的作家

11.为了减肥,安女士在去年夏秋之交开始严格按照规定服用“瘦身”牌减肥药,但经过整4个月的疗程,她的体重反而又增加了4公斤,由此可见,“瘦身”牌减肥药可能是完全无效的。

下列哪项,如果为真,则最能削弱上述结论? A. 她服用的“瘦身”减肥药过于昂贵了。

B. 她服用的“瘦身”减肥药是国家奖的获奖产品。

C. 她服用的“瘦身”减肥药经过多次试用,疗效一直很好。 D. 她服用的“瘦身”减肥药是中外合资企业生产的。

E. 如果不服用“瘦身”减肥药,她的体重在这一段会增加10公斤。

12.斯巴达克人为提高后代的战斗力,对新生的婴儿进行身体素质测试。他们把出生的婴儿放在水中、野地里等恶劣的自然环境下来检验其生命力的强弱,抛弃那些病弱的、经不起考验的婴儿,留下那些健壮的婴儿。然而,这一方法并没有使斯巴达克人的城邦长治久安,反


而先于敌对的雅典而衰落。

根据上文的情况,以下哪项作为斯巴达克人选拔人才最终失败的原因最为合适? A. 新生婴儿身体素质好的,未必长大成人后一定是战斗力更强的。

B. 生理素质只是人全面素质的一部分,国家的安定需要人的综合素质的提高。 C. 人才是选拔出来了,但是人数太少,所以难逃失败的厄运。 D. 雅典采取的人才选拔办法比斯巴达克人可能更为严酷。

E. 孩子的父母对与这种残酷的淘汰心存不满,所以不愿建设和保卫自己的城邦。 13.在日渐发达的社会里,肥胖已成为一种恼人的“社会病”。女人爱苗条,男人爱潇洒,可臃肿的身材,特别是大腹便便的肚子,让人不胜痛苦。在这种情况下,寻找肥胖的原因,探索减肥的良方,几成一股“社会潮流”。目前,某社会机构公布了一项长期社会调查的结果,调查显示:在婚后的13年里,妇女们平均增长了13公斤,男人们平均增长了20公斤。这一机构得出结论:婚姻能使人变胖。

以下哪项,如果为真,最可能对上述结论提出质疑?

A.如果调查时间取3年或者31年,被调查者的体重会远远小于以上增长幅度。 B.在婚后的13年里,被调查的男人中,有一些体重增长不到20公斤。

C.与被调查者年纪相仿的独身者,妇女们13年里平均增长了16公斤,男人们平均增长了25公斤。

D.在婚后的13年里,被调查的男人中有一些体重明显下降了。 E.有些婚后体重增长特快的“重量级妇女”,不愿意参与这类调查。

14.某大都市最近公布了一组汽车交通事故的调查显示,在受重伤的司机和前座乘客中,有80%在事故发生时未系安全带。这说明,系上安全带,就能使司机和前座乘客极大地减少在发生事故时受严重伤害的危险。

上述推断要成立,以下哪项必须是真的?

A.在被调查的所有司机和前座乘客中,20%以上在发生事故时是系着安全带的。

B该大都市中的市民中,在驾车或在前座乘车时习惯于系上安全带的人数,远远超过20% C.调查中发现,因汽车事故受重伤者中,司机和前座乘客要比后座乘客多。 D.一半以上被调查的司机和前座乘客在事故发生时未系安全带。

E.大多数在该大都市交通警察局记录在案的汽车交通事故都不是严重伤亡事故。

15.某法院审理一起盗窃案件,某村的甲、乙、丙三人作为嫌疑犯被押上法庭。审问开始了。法官先问甲:“你是怎样作案的?由于甲说的是方言,法官听不懂。于是,法官就问乙和丙:“刚才甲是如何回答我的问题的?”乙说:“甲的意思是,他并不是盗窃犯。”丙说:“甲刚才招供了,他承认自己是盗窃犯。”法官听完了乙和丙的话之后,马上做出判断:释放乙,逮捕丙入狱。事实证明法官的判断是正确的。 法官做出准确判断最不可能依据的假定是什么?

A.初审时,在没有胁迫的情况下,说真话的不会是盗窃犯,而说假话的是盗窃犯。 B.初审时,在没有胁迫的情况下,甲是不可能招供的。

C.初审时,在没有胁迫的情况下,甲不论是否是盗窃犯,他总会回答说:我不是盗窃犯。 D.据某村村民反映,丙以前曾多次盗窃人家的财物。 E.丙在转述甲的回答中说了假话

16.近年来,以营利性为目的的渔民在南大西洋的河虾捕获量的总重量上急剧下降。这个下降主要是由于日益增多的以娱乐性为目的的渔民的竞争,他们能够在河虾成熟的河湾处捕捉到小的河虾。

下面哪一个管制规则将最可能有助于增加以营利性为目的的渔民的河虾捕获量? (A)要求以营利性为目的渔民在河湾处钓鱼。


(B)限制以营利性为目的的渔民每季效游的总次数。

(C)要求以娱乐性为目的的渔民在钓鱼时使用大网眼的网。

(D)对以娱乐性为目的的渔民被准许捕获的鱼的大小规定一个上限。 (E)允许以娱乐性为目的的渔民移出河湾进入南大西洋。

17.在一项试验中,一半的人摄取了大量的人们常吃的人造糖,另一半人没有吃糖。后来发现,吃糖的一半比没有吃糖的一半认知能力低。这一结果的决定因素是糖中的一种主要成分--氨基酸。

下列哪个选项,如果为真,能最有力地支持糖中的某种成分决定了上述试验结果: A.大多数吃糖的人并不象被试验的人吃那么多糖。

B.上述结论中起确定因素的氨基酸存在于所有蛋白质,为了营养必须摄取一定量的氨基酸。 C.试验人员摄取糖的数量是政府规定的安全范围内的。 D.两组人的认知能力在试验前是相当的。

E.试验的第二部分只针对了吃糖的试验者,而未对没吃糖的试验者做试验。 18.下列哪个选项,如果为真,能够最有力地帮助解释上述观察结果: A.政府有关人造糖的分析表明,出售的糖是相对纯净的。

B.血液中氨基酸含量高使得一种使头脑正常运转的物质发生化合反应。 C.由于糖是一种食品添加剂,对糖的不良反应很少受到消费者的重视。

D.氨基酸是糖中的一种成分,但单独氨基酸本身也是人们每日不可缺少的。 E.作为试验品的人并不知道他属于哪一组,吃了糖还是没有。

19.那些尚未生育的成年母鼠在和幼鼠一起生活七天以后也表现出来母亲的行为方式。这一时间段可以通过使母鼠丧失味觉或使幼鼠不能产生体味来缩短。 下列哪个选项可以作为上述结论的最优假设: A.成年母鼠的嗅觉比幼鼠的嗅觉更灵敏。 B.幼鼠离开生母以后体味的排放量会增加。

C.生育后的成年母鼠比尚未生育的成年母鼠更易受嗅觉暗示的影响。

D.生育后的成年母鼠对那些不是它所生的幼鼠表现出母亲的行为方式比那些尚未生育的母鼠表现出的更快。

E.培养母鼠对非它所生的幼鼠扮演母亲角色的兴趣受到幼鼠体味的阻碍。

20.最近一次战争里在重战区中执行任务的医疗人员,即使是那些身体未受伤害的,现在比在该战争不太激烈的战斗中执行任务的医疗人员收入低而离婚率高,在衡量整体幸福程度的心理状误解测验中得分也较低。这一证据表明即使是那些激烈的战争环境下没有受到身体创伤的人,也会受到负面影响。

下面哪个,如果正确,最强有力地支持了以上得出的结论?

(A)重战区的医疗人员和其他战区的医疗人员相比,服役前所接受的学校教育明显比较少。 (B)重战区的医疗人员比其他战区的医疗人员刚入伍时年轻。

(C)重战区医疗人员的父母和其他战区医疗人员的父母,在收入、离婚率和整体幸福程度方面没有什么显著差别。

(D)那些在重战区服务的医疗人员和建筑工人在收入、离婚率和整体幸福程度等方面非常相似。

(E)早期战争中的重战区服务的医疗人员在收入、离婚率和整体幸福程度等方面,和其他在该战争中服役的医疗人员没有表现出太大差别。

21.目前有很多顾客非常关心由于包装上的浪费对生态环境造成的影响。也许正是这种关心使商场大量地购进紧密包装的产品。这些产品被包装在很小的盒子里,用很少的塑料。在运输中占的空间很小。


以下哪项如果为真,能够最有效地削弱上述解释?

A.很少有顾客认为紧密包装的产品比一般包装的产品节省了包装材料 B.紧密包装产品采用的包装材料与一般包装材料一样可以重新利用

C.那些用过后可以重新利用的紧密包装产品都印有怎样重新加工的明确的说明

D.在超级商场和杂货店的有限的货架上销售紧密包装的产品可以增加商店的销售额 E.由于人们杜绝包装浪费的压力,原来用来包装CD的纸盒子已经绝迹。 22.不入虎穴,焉得虎子。

如果上述断定是真的,那么下述哪项断定必定是真的? I. 除非入虎穴,否则不得虎子。 II. 若已得虎子,则必定已入虎穴。 III. 若未得虎子,则必定未入虎穴。 A. I B. II C. III D. III E. IIII

23.某个体户严重违反了经营条例,执法人员向他宣布:“要么罚款,要么停业,二者必居其一”他说:“我不同意”如果他坚持自己的意见的话,以下哪项是他在逻辑上必须同意的: A. 罚款但不停业。 B. 停业但不罚款。 C. 既罚款又停业。

D. 既不罚款也不停业。

E. 如果既不罚款又不停业办不到的话,就必须接受既罚款又停业。

24.某大学进行论文比赛,得第一名的只能有一人。究竟谁得第一名?在对甲、乙、丙、丁、戊、己六个参赛者进行名次预测时,四人作了如下的预测: 甲:取得第一名的要么是我,要么是乙。 乙:取得第一名的要么是甲,要么是丙。 丙:如果不是戊取得第一名,就一定是己。 丁:第一名决不会是甲。

比赛结束后,发现只有一个人的预测是正确的。请问:谁得第一名? A. B. C. D. E. 25.以下哪个选项可以最好地完成下文: 在最近的一个有关北海环境危机的会议上,许多参与国赞成无论每个地区的环境是否只受某一地区排放物的影响,应对排放物的质量进行统一控制。当然,为了避免过度的限制,必须说明:

A.采用的任何统一管理都要做到不拖延实施。 B.作为控制范围内的内容必须真正会引起污染。 C.赞成统一管理的国家是大量排放的国家。

D.任何一种被限制的排放物必须是造成目前北海的污染的。 E.北海的环境污染现象是可以扭转的。 参考答案:

01.B 02.C 03.B 04.B 05.B 6.E 07.A 08.B 09.B 10.A 11.E 12.B 13.C 14.B 15.D 16.C 17.D 18.B 19.E 20.C 21.D 22.D 23.E 24.D 25.B

01.希望小学组织学生植树,参加者平均植树11棵。男生平均植树15棵,女生平均植树7


棵。

从上文可推出以下哪个结论? A. 每个女生植的树都比男生少。 B. 一定有些女生植的树都比男生多。

C. 参加植树的女生人数比参加植树的男生人数多。 D. 参加植树的女生人数与参加植树的男生人数相等。 E. 参加植树的女生人数比参加植树的男生人数少。

02.雄性巢鸟总是精心地构建、装饰它的巢。根据研究人员的判断,居住在不同地区的同一种巢鸟构建出的鸟巢风格迥异,他们总结到鸟巢的建造模式是应环境的需要,而不是遗传的本能。

下列哪个选项,如果为真,可以最有效地加强上述结论:

A.进行的广泛研究表明一个地区的鸟巢建造模式的共同点多于不同点。

B.幼年的雄性巢鸟并不会建巢,在年幼的时候,它们自己按地区模式建巢之前,它们只是看年长的雄性巢鸟建巢。

C.有一种巢鸟建的巢与绝大多数巢鸟建的巢比起来,没有塔状物,也没有装饰。 D.这种巢鸟只生活在纽硅尼和澳大利亚,显然它们没有机会相互接触。 E.众所周知,一些歌鸟的有地方特色的歌并不是天生会唱的而是学来的。

03.想当优秀运动员的小学生都上业余体校,小玲上了业余体校,她一定是想当优秀运动员。 以下哪项如果为真,则最能支持上述推断? A.所有上业余体校的小学生,都想当运动员。 B.所有优秀运动员都上过业余体校。

C.只有业余体校的优秀学生,才能成为优秀运动员。 D.只有想当优秀运动员的小学生,才上业余体校。 E.有些优秀运动员是业余体校学生。

04.世界高水平的电影演员每年接拍片约都不超过3个,因此,如果左元每年接拍片约4个,他就不是一名世界高水平的电影演员。 以下哪项的推理方法与上文相同?

A.好的学生每天都不开夜车,如果刘勇每天都不开夜车,那么他就是个好学生。

B.当好的领导离开自己的工作岗位后,他手下的员工能够干得象他在时一样好。所以,如果一个总经理调走后公司变成了一盘散沙,那么这个总经理不是一个好的领导。

C.如果一个企业在淡季的时候还能够保持良好的销售势头,那么在旺季时就会有一个丰收。我们公司今年淡季不淡,所以旺季会更旺。

D.移动电话的话费一般比普通电话要贵。如果移动电话和普通电话都在身边时,我们选择了普通电话,那么就是节约的做法。

E.准确的闹钟在10点钟会敲10下,如果现在这个闹钟在10点钟时敲了10下,那么它就是一个准确的闹钟。

05多毛哺乳动物的侏儒个体与正常多毛哺乳动物的个体比较,在体形上,二者长度的比例差距要大于二者在牙齿长度比例上的差距。最近,挖掘出一具多毛哺乳动物成年侏儒的骨骼片断,发现其牙齿长度是正常成年多毛哺乳动物的3/4 以上陈述最有力地支持了以下哪项?

A.成年侏儒多毛动物体形长度不到正常多毛哺乳动物体形长度的3/4

B.最近发现的侏儒多毛哺乳动物的牙齿和正常多毛哺乳动物的牙齿一样大。 C.大多数成年侏儒多毛哺乳动物的牙齿数目与正常多毛哺乳动物一样多。

D.成年侏儒多毛哺乳动物的牙齿数目不超过正常多毛哺乳动物牙齿数目的3/4


E.成年侏儒多毛哺乳动物总数不超过正常多毛哺乳动物总数的3/4 06.不可能所有的花都结果。

以下哪项的意思最接近上述断定? A.可能所有的花都不结果。 B.可能有的花不结果。 C.可能有的花结果。

D.必然所有的花都不结果。 E.必然有的花不结果。

07.在发生空难时,乘客必须迅速离开飞机,因为事故后飞机经常会泄漏有毒气体,并迅速发生爆炸。为了防止乘客中毒,安全部门要求航空公司必须为乘客准备防毒气面罩。 以下哪项如果为真,能够有效说明安全部门的要求不是必要的? A.实验表明,戴面罩延缓了乘客逃离飞机的速度。 B.航空公司不愿意负担昂贵的防毒气面罩费用。

C.虽然面罩能够防止乘客中毒,但并不能防止毒气泄漏。 D.一些经常坐飞机的人不再注意飞机起飞前的安全说明。

E.在很多空难中,很多本来能够到达紧急出口的乘客,由于中毒昏迷而不能逃出机舱。 08.在消化过程中分解为糖的食物是人体血液内葡萄糖的来源,饮用咖啡后,在消化过程中并不能分解为糖。然而,有时人饮用不加糖和奶的咖啡后,也会引起血液葡萄糖的大量增加。 以下哪项帮助解释咖啡对血液葡萄糖水平的作用?

A.人们常常在服食某些食物后饮食咖啡,这些食物在消化过程中可迅速地分解为葡萄糖。 B.饮用超过两杯咖啡会增加人体压力,人体对压力的反应包括释放储存的葡萄糖进入血液。 C.服食含少量细糖食物的人比服食含大量细糖食物的人的血葡萄糖水平要高。 D.对许多人而言,服食一块巧克力饼干与饮用一杯咖啡有相同的刺激作用。

E.办公室的工作人员通常比体力劳动者饮用更多的咖啡,并有更高的血液葡萄糖水平。 09.人生之路,考试重重。面临即将到来的考试的巨大压力,很多人吃不好、睡不着、难以精神抖擞地全力备考。“双百全优”中药冲剂,集中国传统医学之精华,镇静您的精神,焕发您的精力,去除长时间学习的疲倦,让您吃好睡好,以崭新的面貌走上考场。 以下哪一项在上述的广告宣传中没有作为“双百全优”中药冲剂的功效特性? A.夺取优异成绩。 B.稳定精神状态。 C.帮助保持食欲。 D.恢复身体疲劳。 E.加强睡眠休息。

10.当且仅当竞争对手甲退出投标时,对手乙就会报一个较高的价位,我方也才不会在第一轮就被淘汰。当且仅当我方在第一轮竞争中就被淘汰之后,第一轮轮空的强有力对手丙才会在第二轮报价。当且仅当对手丙报价时,才会出现强劲对手丁不战退出的情况,或者出现另一个对手戊报一个相当的高价的情况,或者两种情况都出现。 如果已知的信息是丁退出了第二轮竞争,下面哪项一定是事实? A.我方通过了第一轮的竞争。 B.竞争对手乙首轮报价过高。 C.竞争对手甲没有退出投标。 D.竞争对手丙第二轮没报价。 E.竞争对手戊报了一个高价。

11.有国风、江夏、金力三家投资公司,一家总部在北京,一家总部在上海,一家总部在深


圳。三家公司中,一家主营基金,一家主营外汇,还有一家主要在证券二级市场炒做。其中,国风公司不经营基金,江夏公司不在证券二级市场上炒做;主营基金的总部不在上海,主要在证券二级市场炒做的公司总部在北京;江夏公司总部不在深圳。 根据上述事实,请尽可能准确地判断国风公司的主要经营方向: A.主营基金。 B.主营外汇。

C.主要在证券二级市场炒做。

D.外汇或是在证券二级市场炒做。 E.已知信息不足,无法判断。

12.交通事故大都是因为车速太快,当遇到问题司机或者根本来不及刹车,或者刹不住车。有实验表明,在时速为100公里时,两车的安全距离为100米。因此,一个了解这一实验结果而又不愿出事故的司机,在时速100公里时,其距前车的距离不会小于100米。 下列哪项为真,最能削弱上述论点? A.很少有人开车能达到时速100公里。

B.当两车相距100米时,肯定会有车从后面超过来加在中间。 C.路上的车辆太多,无法保持适当距离。

D.中国的车况、路况很少能达到时速100公里。 E.有时人们很少注意自己开车的速度。

13.新星机械厂最近一段时间经营状况的改观雄辩地证明了,一个精明的企业领导人能够怎样有效地扭亏为赢,挽救企业的命运。新星机械厂新聘请的外籍总经理史怀特先生到任后,机械厂在半年之内由原来的亏损100万元,一跃转变为盈利35万元。 上述论断最主要的漏洞在于:

A.木器厂的成功可能是暂时现象,也许明年又亏损了。 B.盈利与亏损缺乏严格的界定,概念不清,无法断言。 C.假定外籍总经理的上任是机械厂发生转变的唯一原因。

D.聘请外籍管理者仅仅是个短期解决问题的办法,长期还要依靠中国人自己的管理者。 E.新任的总经理的措施也许会对后面的继任者带来不利的影响。

14.因为气候的变化,同时又因为有许多原先种植番薯的农民改种其它作物,过去三年来,我国番薯的产量一直低于销售量,番薯一直处于相对短缺状态,价格也一涨再涨。 由上文最可能推断出的结论是: A.番薯只有大量种植才有利可图。

B.过去三年中因为种番薯而使我国农民获利很大。 C.气候一旦恢复正常,我国番薯的产量又会上升。 D.过去三年中我国番薯消费量异常的高。 E.过去三年中我国番薯的剩余储备减少了。 15.甲、乙两个人之间发生了以下一段对话:

甲:国营公司X三年前被民营公司Y兼并,现在盈利状况很好,这说明私有制企业的效益要比国有企业的好。

乙:不对,仔细分析X公司的有关经营资料可以发现,在X公司被Y公司兼并前,X公司调整了领导班子,从那以后不久X公司就开始盈利了。

以下哪项指出了甲的推理中的一个破绽,而乙在反驳甲时又抓住了这个破绽? A.X公司的盈利可能是暂时的。

B.甲所认为的原因可能发生在甲陈述的结果之后。 C.X公司的盈利并没有提出准确的数据支持。


D.甲仅仅根据一个公司的资料就片面地得出了“私有制企业的效益要比国有企业的好”这样的一般性结论,未免过于草率。

E.国有企业不等于国营公司,民营公司不等于私有企业。

16.医生甲、乙、丙可治病LMNO。医生甲只能治其中的一种病;医生乙不能治M病;O病不能由医生丙治;L病和O病由同一个医生治;每种病只有一个医生能治。 根据以上对事实的陈述,以下哪个判断为假? A.医生甲不能治O病。

B.医生乙只能治其中一种病。 C.医生丙不能治L病。

D.根据以上事实无法判断出哪位医生能治M病。 E.根据以上事实无法判断出哪位医生能治N病。

17.凡是金属都具有可塑性,铁具有可塑性,所以铁是金属。 下列哪项陈述与上面陈述的推理结构最相近?

A.凡是恒星都是自身发光的,金星不是恒星,所以金星自身不发光。

B.油漆三小时内不会干,这种涂料在三小时内干了,所以这种涂料不是油漆。

C.所有不了解实际情况的人都攻击我扩大经营规模的决策,你攻击我,所以你不了解实际的情况。

D.凡是公民都必须遵纪守法,我们是公民,所以我们也必须遵纪守法。 E.凡是真理都是经过实践检验的,“内行管理外行”是真理,所以“内行管理外行”是经过实践检验的。

18.有一位研究者在分析了从古到今的案例和统计资料后认为:在体育运动方面的才能,黄种人不比其它人种的差。之所以黄种人的体育运动才能未被充分地发掘出来,是因为体育运动对黄种人来说一直不是一种很有意义而得到充分重视的事情。 以下哪一项是该研究者上述断言的一个隐含假定? A.黄种人在体育运动领域没有其它人种的成绩突出。

B.黄种人倾向于从事他们认为很有意义而加以充分重视的事情。 C.黄种人与其它人种体育运动方面具有相当的才能。 D.体育运动方面的才能不如其它方面的才能更重要。 E.黄种人在体育运动方面的才能需要很好地进行挖掘。

19.地球之外有没有生命是科学家长期探索的课题。1996年美国航天局研究人员对火星陨石的研究中,正式提出了表明火星上36亿年前存在生命的证据,并向全世界的科学家提出挑战,欢迎他们证明这一论点是错误的。科学界对这一问题反映不一。以下是一些专家的论述。 在下面的几点意见中,哪个是对美国航天局的挑战?

A.这是能证明地球外生命的最令人深思和浮想联翩的事情。

B.德国一研究员说,36亿年前在太阳系中有众多的陨石,很难确切断定哪一块真正来自火星。

C.对陨石上取下一小片金色样品进行的化学显微和组织检查表明,36亿年前这里有过原始生命、微生物生命的存在。

D.如果已经发现36亿年前火星上有生命的存在,我不会特别感到意外。 E.我们不能排除这种可能性:生命从火星来到了我们所在的这颗行星。

20.经过一段时间的研究,人们发现商场室内温度略高所导致的顾客与售货员之间的争吵要比温度略低时多。因此,如果我们能控制商场室内温度,买卖之间的争吵就能得到控制。 上述推论隐含了以下哪项假设?

.室内温度与买卖争吵之间的关系是纯属巧合的。


.室内温度与买卖争吵之间的关系是有因果关系的。 .室内温度与买卖争吵之间的关系是可以被利用的。 A.III B.IIIII C.IIIIII D.I E.II

21.在最近公布的市场调查报告中,我们发现江东市的居民在婚纱摄影上的平均支出比前两年提高了60%而与此同时,艺术影楼的数量只比前两年增加了10%这说明该市婚纱摄影的市场还远没有达到饱和,大有进入的余地。

以下答案若均属实,则哪一个最可能削弱上述结论? A.该市居民对婚纱摄影的兴趣是最近一年刚刚兴起的。 B.前两年该市大部分艺术影楼的收入状况都非常差。 C.该市近两年居民的平均生活水平上升了30%

D.该市这两年开始进入了结婚高峰期,适龄青年多,结婚比例高。 E.目前该市的大部分艺术影楼的服务水平还不是很高。 22.人类对糖的渴望曾经是有益的,它吸引着人喜爱吃更健康的食品(例如成熟的水果)然而,现在的糖是精制糖,而精制糖对健康是不利的。因此,对糖的渴望将是对人体无益的。 以下哪项强化了上述论述?

A.某些食物生吃不利于健康,煮熟吃对人的健康是有利的。 B.某些渴望吃糖的人宁可吃煮熟的水果,而不吃饼干。

C.以前人不用味觉就分不开哪些食品是有利于健康的,哪些食品是不利于健康的。 D.非精制食品并不比精制食品更利于健康。

E.渴望吃糖的人更可能吃含精制糖的食品,而较少吃含天然糖份(如水果)的食品。

23.由于每一层次的员工都不愿意在上级管理者眼里与坏消息有所关联,因此基层出现的严重问题在沿管理层次逐级上报时总是被淡化或是掩盖。所以,位于最高层次上的总经理对基层出现的真实问题的了解,要比他的下级们少得多。 以上结论是建立在以下哪个假设基础之上的呢?

A. 管理层级中,较高层次的管理者解决问题的能力要比低层管理者解决问题的能力强。 B. 仍然有一些员工更关注的是事实,而不是他们在上级管理者心目中的印象。 C. 位于最高层次的总经理只能从直接下级处了解基层问题,而没有别的渠道。 D. 在哪一层发生的管理问题应有哪一层的管理人员去加以解决。

E. 向上级如实汇报基层情况的员工应当受到来自高层管理者的特别嘉奖。

24.市场上生产的PC机有大约200种品牌,而我们进的货只局限于八种最流行的品牌。我们计划通过增加十种最好销的品牌来增加销量。

下列哪个选项,如果为真,可以最有力地指出上述计划的弱点:

A.三种最流行的PC机品牌功能相似,其中没有哪种品牌在各方面都有优势。 B.七种最流行的品牌几乎构成了所有PC机的销量。

C.随着PC机的用户水平越来越高,他们更倾向于购买并不知名的品牌。

D.不流行的品牌往往给零售商带来较少的利润,因为为了吸引消费者必须采取价格折扣。 E.最知名的品牌的PC机销售量不如非名牌PC机,是由于非名牌PC机与其功能相似且价格低。

25.电脑尽管在许多方面具有很强的功能,但有实验表明,在运算加减乘除时,操作电脑还比不上使用算盘来得更快、更准确。

以下哪项如果为真,能够使上述实验的说服力大为削弱? A.实验中操作电脑与使用算盘的人必定是不同的人。 B.使用算盘的人可以运用口诀。

C.使用算盘的时候能够借助于心算,而操作电脑者却不能。


D.用电脑作运算,不在乎数字的大小,而算盘则不然。实验中运算的只是较小的数字。 E.使用电脑时需要键入诸如“+-×÷”等运算符。 参考答案:

01.D 02.B 03.D 04.B 05.A 06.E 07.A 08.B 09.A 10.C 11.C 12.B 13.C 14.E 15.B 16.B 17.C 18.B 19.B 20.B 21.B 22.E 23.C 24.B 25.D

01.粮食可以在收割前在期货市场进行交易。如果预测谷物产量不足,谷物期货价格就会上升;如果预测谷物丰收,谷物期货价格就会下降。今天早上,气象学家们预测从明天开始谷物产区里会有非常需要的降雨。因为充分的潮湿对目前谷物的存活非常重要,所以今天的谷物期货价格会大幅下降。

下面哪个,如果正确,最严重地削弱了以上的观点?

A.在关键的授粉阶段没有接受足够潮湿的谷物不会取得丰收。 B.本季度谷物期货价格的波动比上季度更加剧烈。

C.气象学家们预测的明天的降雨估计很可能会延伸到谷物产区以外。 D.农业专家们今天宣布,一种已经毁坏一些谷物作物的病菌在生长季节结束前会更广泛地传播。

E.许多在谷物期货市场交易的人很少实际拥有他们所交易的谷物。 02.在一次选举中,统计显示,有人投了所有候选人的赞成票。 如果统计是真实的,那么下列哪项也必定是真实的? A.对每个候选人来说,都有选民投了他的赞成票。 B对所有候选人都投赞成票的不止一人。 C.有人没有投所有候选人的赞成票。 D.不可能所有的候选人都当选。 E.所有的候选人都可以当选。

03.针对某种溃疡最常用的一种疗法可在6个月内将44%的患者的溃疡完全治愈。针对这种溃疡的一种新疗法在6个月的试验中使治疗的80%的溃疡取得了明显改善,61%的溃疡得到了痊愈。由于该试验只治疗了那些病情比较严重的溃疡,因此这种新法显然在疗效方面比最常用的疗法更显著。

对下列哪一项的回答最能有效地对上文论述做出评价? (A)这两种疗法使用的方法有何不同?

(B)这两种疗法的使用成本是否存在很大差别?

(C)6个月中以最常用疗法治疗的该种溃疡的患者中,有多大比例取得了明显康复? (D)这种溃疡如果不进行治疗的话,病情显著恶化的速度有多快?

(E)在参加6个月的新疗法试验的患者中,有多大比例的人对康复的比例不满意?

04.由风险资本家融资的初创公司比通过其他渠道融资的公司失败率要低。所以,与诸如企业家个人素质、战略规划质量或公司管理结构等因素相比,融资渠道在初创公司的成功上是更为重要的原因。

下面哪个,如果正确,最严重地削弱了以上的结论?

A.风险资本家对初创公司财务需要的变化比其他融资渠道更为敏感。 B.在公司的长期成功方面,初创公司的战略规划比起企业家个人素质来说是一个相对不重要的因素

C.所有的新公司中有超过一半在5年内倒闭了。

D.一般来讲,初创公司的管理结构不如发展中的公司正式。

E.风险资本家在决定为初创公司提供资金时依据以下因素,如企业家素质和公司的战略规划质量。


05.高塔公司是一家占用几栋办公楼的公司,它现在考虑在它所有的建筑内都安装节能灯泡,这种新灯泡与目前正在使用的传统灯泡发出同样多的光,而所需的电量仅是传统灯泡的一半。这种新灯泡的寿命也大大加长,因此通过在旧灯泡坏掉的时候换上这种新灯泡,塔公司可以大大地降低其总体照明的成本。

下列哪一项,如果正确,最能支持上面论述?

(A)如果广泛地采用这种新灯泡,这是非常可能的,那么新灯泡的产量就会大大增加,从而使其价格与那些传统灯泡相当。

(B)向高塔公司提供电力的公共事业公司向其最大的客户们提供折扣。 (C)高塔公司最近签订了一份合同,要再占用一栋小的办公楼。

(D)高塔公司发起了一项运动,鼓励其员工每次在离开房间时关灯。

(E)生产这种新灯泡的公司对灯泡中使用的革新技术取得了专利,因此它享有生产新灯泡的独家权利。

06.社会撰写人:因为青少年缺乏基本的开车技巧,所以应给予青少年的驾驶执照附加限制。尽管19岁和再小一点的司机只占注册司机的7%但是他们却是超过14%的交通死亡事故的肇事者。

下面每一项,如果正确,都能削弱青少年缺乏基本的开车技巧的论述,除了: A.与其他人开的车相比,青少年开的车较旧,且稳定性也差。

B.青少年司机和他们的乘客使用座带和肩带的可能性不如其他人的大。C.青少年司机平均每年开车的距离超过其他司机的两倍。

D.青少年引起的交通事故比其他人引起的交通事故严重。 E.青少年开车时的乘客人数很有可能比一般的司机多。

07.某一市长曾建议向进城的私人车辆每天收取五美元的费用,宣称这种费用的征收将缓解该城市的交通拥挤状况。该市长解释说,由于该费用比许多附近站点乘坐环线公共汽车的费用要高,许多人会由自己驾驶汽车转为乘坐公共汽车。

以下哪一项陈述,如果是正确的,为证明该市长的推理是有缺陷的提供了最好的证据? (A)汽油价格的预期上升将提高进城的私人车辆的成本。

(B)停车费用已经使大多数开私人车辆进城的人觉得比乘坐公共汽车昂贵很多了。 (C)目前乘坐公共汽车的人多数没有自己的私家车。

(D)许多反对该市长计划的通勤者指出他们宁愿忍受交通拥挤,也不愿付每天五美元的费用。

(E)在一般的工作日,居住在城区里的人拥有和驾驶的私人车辆占到了该城整个交通流量的20%

08.国会削减社会福利费看来会损害穷人的利益,其实不会。因为社会福利预

算削减的同时,税收也削减。因此,每个人手中的钱将变多,而不是变少.以下哪项能动摇上述论证?

(l)穷人正在敦促国会提高社会福利预算。

(2)穷人本来几乎不纳税或者纳很少的税,因此,税收削减对他们来说没有多大好处。 (3)穷人因税收削减所得到的好处,补偿不了因福利费削减带来的损失。 A.仅仅(1) B.仅仅(2) C.仅仅(3)

D.仅仅(2)(3) E.(1)(2)(3)

09.反核活动家:关闭这个核电站是反核的胜利,它同时也体现了核工业很迟才肯承认


它们不能安全运作这样的发电站的事实。

核电站经理:它并不体现这样的事情。从非核资源可得到便宜的电力,加上强制性的安全检查和安全维修,使继续运作变得不经济。因此,不是出于安全方面的考虑,而是出于经济方面的考虑,才下令关闭了这个核电站。 经理的论证的推论是有缺陷的,因为该论证

A.没有承认即使这家核电站不是出于安全方面的原因则被关闭,电力公司现在也可能会认为核电站是不安全的。

B.忽略了那些可以利用的便宜电力资源本身也可能存在安全问题的可能性。

C.把关闭这个核电站对公众来说体现了什么的问题错认为是经理的关闭理由是什么的问题。

D.把电力工业对待核安全的态度与反核活动家的观点相抵触的态度的观点作为它的一个前提。

E.把由于需要采取安全预防措施而引起的一些费用的上升看作是纯粹的经济上的因素。 10.赵明、钱红、孙杰三人被北京大学、清华大学和北京师范大学录取。他们分别被哪个学校录取的,同学们作了如下的猜测:

同学甲猜:赵明被清华大学录取,孙杰被北京师范大学录取。 同学乙猜:赵明被北京师范大学录取,钱红被清华大学录取。 同学丙猜:赵明被北京大学录取,孙杰被清华大学录取。 结果,同学们的猜测各对了一半。 那么,他们的录取情况是

A.赵明、钱红、孙杰分别被北京大学、清华大学和北京师范大学录取。 B.赵明、钱红、孙杰分别被清华大学、北京师范大学和北京大学录取。 C.赵明、钱红、孙杰分别被北京师范大学、清华大学和北京大学录取。 D.赵明、钱红、孙杰分别被北京大学、北京师范大学和清华大学录取。 E.赵明、钱红、孙杰分别被清华大学、北京大学和北京师范大学录取。

11.电学工程师已反复重申,最好的晶体管扩音机与最好的电子管扩音机在通常测量评价扩音机的音乐再现质量方面的性能是一样的。因此那些坚持认为录制的音乐在最好的电子管扩音机里播放时要比在最好的晶体管扩音机里播放时听起来好的音乐爱好者,一定是在想象他们声称的听到的质量上的差异。

下面哪一点,如果正确,最能严重削弱上述辩论?

A.许多人仅凭耳听不能区分正在播放的音乐是在好的晶体管扩音机里播放还是在好的电子管扩音机里播放。

B.电子管扩音机的音乐再现质量的变化范围要比晶体管扩音机的大。 C.有些重要的决定音乐听起来怎么样的特性不能被测量出来。

D.当放出相同的音量时,晶体管扩音机比电子管扩音机的体积小,用电少且产生的热量少。

E.在试验室里通常测定的用以评价扩音机的音乐再现质量的特性方面,有些电子管扩音机明显地比晶体管扩音机好。

12.室外音乐会的组织者宣布,明天的音乐会将如期举行,除非预报了坏天气或预售票卖得太少了。如果音乐会被取消,将给已买了票的人退款。尽管预售票已卖得足够多,但仍有一些已买了票的人已经得到了退款,这一定是因为预报了坏天气的缘故。 下列哪一项是该论述中含有的推理错误?

(A)该推理认为如果一个原因自身足以导致其一结果,那么导致这个结果的原因只能是它。


(B)该推理将已知需要两个前提条件才能成立的结论建立在仅与这两个条件中的一个有关系的论据基础之上。

(C)该推理仍解释说其中一事件是由另一事件引起的,即使这两件事都是由第三件未知的事件引起的。

(D)该推理把缺少某一事件会发生的一项条件的证据当作了该事件不会发生的结论性证据。

(E)试图证明该结论的证据实际上削弱了该结论。

13.如果小赵去旅游,那么小钱、小孙和小李将一起去。 如果上述断定是真的,那么,以下哪项也是真的?

A.如果小赵没去旅游,那么小钱、小孙、小李三人中至少有一人没去。 B.如果小赵没去旅游,那么小钱、小孙、小李三人都没去。 C.如果小钱、小孙、小李都去旅游,那么小赵也去 D.如果小李没去旅游,那么小钱和小孙不会都去。 E.如果小孙没去旅游,那么小赵和小李不会都去。

14.甲:最近,我被一家航空公司的某一航班拒绝了一个我已经确认过的预定座位,因为这家航空公司超额预定了那个航班。因此,我被迫乘下一班可乘的航班,该航班两个小时后才起飞,我错过了一个非常重要的商业会议。即使我预定的那个航班在最后一分钟因为天气原因而被取消,航空公司也应该因没能让我乘坐那个航班而给我赔偿。 乙:从道义上来说,航空公司没有给你赔偿的责任,即使你没被拒绝乘坐早一点的航班,无论如何你都会错过你的商业会议。

下面哪一条原则,如果正确,能证明乙对甲的反应,即从道义上讲航空公司有责任赔偿那些在某一航班上确认了预定座位而又被拒绝乘坐该航班的乘客是合理的?

A.如果迫使乘坐晚一点航班的惟一原因是航空公司已超额预定了那次航班。 B.只有当乘客被迫乘坐晚一点的航班的原因不是因为天气恶劣而取消了该航班。

C.只有当航空公司没有超额预定最初的那次航班,乘客也没有被迫乘坐晚一点的航班。 D.即使乘客被迫乘坐晚一点的航班的惟一原因是航空公司因为天气不好而取消了最初的那次航班。

E.即使在航空公司没有超额预定最初的那次航班的情况下,乘客仍被迫乘坐晚一点的航班。

15.软饮料制造商:我们的新型儿童软饮料力比咖增加了钙的含量。由于钙对形成健康的骨骼非常重要,所以经常饮用力比咖会使孩子更加健康。

消费者代表:但力比咖中同时含有大量的糖份,经常食用大量的糖是不利于健康的,其是对孩子。

在对软饮料制造商的回应中,消费者代表做了下列哪一项? (A)对制造商宣称的钙元素在儿童饮食中的营养价值提出质疑。

(B)争论说如果对制造商引用的证据加以正确地考虑,会得出完全相反的结论。 (C)暗示产品制造商通常对该产品的营养价值毫不关心。

(D)怀疑某种物质是否在适度食用时有利于健康,而过度食用时则对健康有害。 (E)举出其他事实以向制造商所做的结论提出质疑。

16.有确凿的证据显示,偏头痛(严重的周期性头痛)不是由于心理上的原因引起的,而是完全由生理上的原因所致。然而,数项研究结果表明那些因为偏头痛受到专业化治疗的人患有标准心理尺度的焦虑症的比率比那些没经专业治疗的偏头痛患者的高。 下面哪一点,如果正确,最能有助于解决上面论述中的明显矛盾? A.那些患有偏头痛的人,倾向于有患偏头痛的亲戚。


B.那些患偏头痛的人,在情绪紧张时经常头痛。

C.那些患有标准心理尺度的焦虑症且发作率较高的人追求专业治疗的可能性要比那些在同样尺度上发作率较低的人大。

D.在许多有关偏头痛起因的研究中,大多数认为偏头痛是由像焦虑这样的心理因素引起的研究已被广泛宣传。

E.不管他们的医生认为偏头痛的起因是心理方面的,还是生理方面的,大多数患有偏头痛且追求专业治疗的人在他们停止患有偏头痛后仍坚持治疗。

17.小王、小李、小张准备去爬山。天气预报说,今天可能下雨。围绕天气预报,三个人争论起来。 小王。“今天可能下雨,那并不排斥今天也可能不下雨,我们还是去爬山吧。 小李:“今天可能下雨,那就表明今天要下雨,我们还是不去爬山了吧。 小张:“今天可能下雨,只是表明今天不下雨不具有必然性去不去爬山由你们决定。 对天气预报的理解,三个人中 A.小王和小张正确,小李不正确。 B.小王正确,小李和小张不正确。 C.小李正确,小王和小张不正确。 D.小张正确,小王和小李不正确。 E.小李和小张正确,小王不正确。

18.如果一项投资不能产生利润,那么以投资为基础的减轻赋税就是毫无用处的。任何一位担心新资产不会赚钱的公司经理都不会因减轻公司本来就不欠的税款的允诺而得到安慰。

下面哪项是从上文得出的最可靠的推论?

(A)阻止效益不佳的投资的最有效的方法是对可以产生利润的投资减轻税赋。 (B)公司经理在决定他们认为可以盈利的投资时,可能会不考虑税款问题。 (C)对新投资减轻税款的承诺本身不会刺激新投资。

(D)公司经理把税款问题的重要性看得越小,他就越可能正确地预测投资的有利性。 (E)公司投资决策的一个关键因素可能是公司经理对感知到的商业状况的心理反应。 19.一块石头被石匠修整后,曝露于自然环境中时,一层泥土和其他的矿物便逐渐地开始在刚修整过的石头的表面聚集。这层泥土和矿物被称作岩石覆盖层。在一安迪斯纪念碑的石头的覆盖层下面,发现了被埋藏一千多年的有机物质。因为那些有机物质肯定是在石头被修理后不久就生长到它上面的,也就是说,那个纪念碑是在1492年欧洲人到达美洲之前很早建造的。

下面哪一点,如果正确,能最严重地削弱上述论述? A.岩石覆盖层自身就含有有机物质。

B.在安迪斯,1492年前后重新使用古人修理过的石头的现象非常普遍。 C.安迪斯纪念碑与在西亚古代遗址发现的纪念碑极为相似。 D.最早的关于安迪斯纪念碑的书面资料始于1778年。

E.贮存在干燥和封闭地方的修理过的石头表现,倘若能形成岩石覆盖层的话,形成的速度也会非常地慢。

20.根据医学资料记载,全球癌症的发病率20世纪下半叶比上半叶增长了近10倍,成为威胁人类生命的第一杀手。这说明,20世纪下半叶以高科技为标志的经济迅猛发展所造成的全球性生态失衡是诱发癌症的重要原因。

以下各项,如果是真的,都能削弱上述论证,除了

A.人类的平均寿命,20世纪初约为30岁,20世纪中叶约为4O岁,目前约为65岁,


症发病率高的发达国家的人均寿命普遍超过70岁。

B.20世纪上半叶,人类经历了两次世界大战,大量的青壮年人口死于战争;而20世纪下半叶,世界基本处于和平发展时期。

C.科技极大地提高了医疗诊断的准确率和这种准确的医疗诊断在世界范围的覆盖率。 D.科技极大地提高了人类预防、早期发现和诊治癌症的能力,有效地延长着癌症病人的生命时间。

E.从世界范围来看,医学资料的覆盖面和保存完好率,20世纪上半叶大约分别只有20世纪下半叶的50%70%

21.一份关于酸雨的报告总结道,“加拿大的大多数森林没有被酸雨损害。”这份报告的批评者坚持认为这一结论必须改变为,“加拿大的大多数森林没有显示出明显的被酸雨损害的症状,如不正常的落叶、生长速度的减慢或者更高的死亡率。 下面哪个,如果正确,为批评者坚持要改变报告结论提供了逻辑上最强有力的正当理由? A.加拿大的一些森林正在被酸雨损害。 B.酸雨可能正在造成症状尚未明显的损害。

C.报告没有把酸雨对加拿大森林的损害与酸雨对其他国家森林的损害进行比较。 D.过去的15年内,加拿大所有森林都下过酸雨。 E.酸雨造成的损害程度在不同森林之间具有差异。

22.近期土地价格的下跌已经使许多在房地产上大量投资的机构受到了损害。去年,在这次价格下跌尚未开始的时候,一所地方大学为其资产增加了200英亩的土地。当然,这所大学并未购买这块土地,而是作为馈赠接受下来的。所以价格下降并没有影响到该大学。 下面哪个,如果正确,对以上的结论提出了最严重的质疑? A.去年给予这所大学的200亩土地与该所大学处于同一社区。 B.与房地产馈赠相比,这所大学经常接受更多的资金损赠。 C.这所大学所处地区目前的土地价格要高于全国的平均水平。

D.去年,这所大学预算用来进行翻修的资金包括今年出售一些土地的预期收入。 E.去年,这所大学没有交纳学校建筑物所占土地的地产税,相反却付费补偿地方政府所提供的服务。

23.在某餐馆中,所有的菜或者属于川菜系或者属于粤菜系,张先生点的菜中有川菜,因此,张先生点的菜中没有粤菜。 以下哪项最能增强上述论证?

A.餐馆规定,点粤菜就不能点川菜,反之亦然。

B.餐馆规定,如果点了川菜,可以不点粤菜,但点了粤菜,一定也要点川菜。 C.张先生是四川人,只喜欢川菜。 D.张先生是广东人,但不喜欢粤菜。 E.张先生是四川人,最不喜欢粤菜。

24.飞机制造商:我反对你把我们的X-387型喷气机描述为危险的。商业使用的X-387机从未坠毁,也未曾有过严重的功能失调。

航空调度员:X-387飞机的问题并不在于其自身,而在于发动起来时会引起空气湍流,给附近的飞行器造成危险的环境。

航空调度员通过下面哪一个对制造者做出了回答?

A.把制造商的论断特征描述为来自主观兴趣,而不是来自于对事实的客观评价。 B.把注意力集中于这个事实:制造商对“危险”的阐释太狭隘了。 C.引用一些制造商把它们当作与争论问题无关而明显忽略的证据。 D.引用统计证据以反驳制造商的断言。


E.向制造商对最近空难数量的了解程度提出质疑。

25.某些媒分子通过使环绕肺气管的骨肉细胞收缩来抵御有毒气体对肺部的损害。这使得肺部部分封闭起来。当这些媒分子被不必要的激活时,对某些无害的像花粉或家庭粉尘作出反应,就出现了哮喘病。

有一项计划是开发一种药物通过阻碍接收由上文所说的媒分子发出的信息来防止哮喘病的发生。以下哪一项,如果是正确的,将指出这项计划的最严重的缺陷? (A)研究人员仍不知身体是如何产生这种引发哮喘病的媒分子的。 (B)研究人员仍不知是什么使一个人的媒分子比其他人的更易激活。

(C)很多年内无法获得这样的药物,因为开发和生产这种药物都需要很长的时间。 (D)这样的药物无法区分由花粉和家庭粉尘引发的信息与由有毒气体引发的信息。 (E)这样的药物只能是预防性的,一旦得上哮喘,它无法减轻哮喘的程度。 参考答案:

01.D 02.A 03.C 04.E 05.A 06.D 07.B 08.D 09.E 10.A 11.C 12.A 13.E 14.C 15.E 16.C 17.A 18.C 19.B 20.B 21.B 22.D 23.A 24.B 25.D 01.

去年电话机行业的销售额大幅度上升。为了利用这一形势,某电话公司准备扩大本公司型号的电话机生产量,同时继续从事已经家喻户晓的广告宣传工作。

以下哪项为真,则最有可能使得该公司采用以上计划时不能增加销售额? A.虽然去年生产的产品全部售出,但该公司的市场占有率是下降的。 B.该公司准备发运给零售商的电话机的库存数去年有轻微下降。 C.广告使得该公司生产的电话机的品牌广为人知,但很少有人知道这个牌子的电话机是该公司生产的。

D.该公司的电话机是去年销售额增加的三种品牌之一。 E.尽管零售价格有所上升,该公司的销售额去年是下降的。 02.

有一种高效降血脂药物目前只能从一种叫“菠苴”树的树皮中提取。这种树在我国非常稀少,而制取一公斤药物需要剥取1000株“菠苴”树的树皮。因此,这种药物在我国的大批量持续生产,将不可避免地导致“菠苴”树种在我国的灭绝。 以下哪项如果为真,最能削弱上述结论? A.我国已制定法律严格禁止砍伐“菠苴”

B.“菠苴”可以通过剪枝插栽的方式由人工栽培。 C.在我国,“菠苴”大多生长在人迹稀少,交通不便的边远地区。

D.由“菠苴”树皮制取的药物须由中央卫生部门的批准才能交付医生使用。

E.非洲某国有丰富的“菠苴”树资源,我国每年可以从该国购得10000株“菠苴”树皮作为药物原料。 03.

历史证明,民族兴旺、国家发展的关键因素是国民素质的提高。因此,实现我国宏伟发展目标的关键措施是进一步增加教育投入。 上述断定基于以下哪项假设?

(1)教育事业的发展是提高国民素质的主要条件。 (2)增加教育投入是发展教育事业的重要条件。 (3)我国目前的教育投入不能适应发展教育的需要。 A.仅(1) B.仅(2)


C.仅(3)

D.仅(1)(3) E(1)(2)(3) 04.

几年前的一个司法法令的目标是鼓励电话业的竞争,竞争可能导致消费者的节省。现在白天打的长途电话比法令公布前便宜了,但是居民的长途电话的平均花费已经增加了百分之二十五。

下列哪一个,假如正确,最好地解释了居民长途电话费更高了? A.企业打的长途电话比居民打的多。

B.电话公司把他们的服务延伸到计算数据和数据处理领域。 C.大多数居民打电话的时间是夜晚,夜晚的电话费已经增加了。 D.竞争的加剧已经导致电话公司为新技术的发展扩大预算。

E.电话公司在电话费率变化付之实践之前必须得到管理机构的批准。 05.

从一组调查数据中,我们发现这样一个有趣的现象。从1980年至今,我国农民的人均收入和化妆品的销售额几乎是在同步增长。 下述哪项最能合理地解释这个数据结果呢?

A.改革开放以前,农民的温饱问题都解决不了,哪里还有闲钱去买化妆品呢。现在农民的日子好过了,他们把多挣的钱都用来购买化妆品。

B.虽然实行了计划生育政策,但由于我国农民的基数大,农民人口的增长率仍然很快,人口多,用的化妆品自然就多。

C.改革开放以来,我国人民的生活水平有了很大提高,所以农民的收入和化妆品的销售额都有了较大增加。

D.这一时期里,化妆品的广告越来越多,对化妆品这个行业的发展起到了推动作用。 E.化妆品的价格上涨幅度很大,导致化妆品的销售额增加。 06.

需求规律告诉我们:如果某种产品价格上涨,那么对它的需求就会减少,价格下跌,需求就会增加。但是,1988年,当大豆色拉油的价格上涨时,对它的需求却大大增加了。 如果上述情况属实,以下哪项能最好地解释上述理论和实际的差别? A.因为大豆色拉油味道不错,许多人都喜欢它。 B.大豆色拉油便于储存,比较方便。

C.大豆色拉油的制作工艺比较复杂,一般家庭不会做,只得去买。

D.因为制作大豆色拉油的大豆等的零售价格上涨,所以大豆色拉油的价格自然要上涨。 E.因为纯正花生油等其他食用油的零售价比大豆色拉油上涨得更快,所以相对于其他食用油,大豆色拉油的价格还是比较便宜的。 07.

在一次产品评比会上,某彩电生产企业生产的21厘米彩电获得了金奖。该企业的总经理觉得很受鼓舞,当即决定:加大投入,把21厘米彩电的产量扩大一倍。 以下哪项最不能质疑这位总经理的决策?

A.生产21厘米彩电所需的彩色玻壳已经很难买到了。 B21厘米彩电已经面临被淘汰的局面。 C21厘米彩电已经在市场上供大于求了。

D.此次评比会不是权威部门举办的,很多彩电生产企业没有参加。

E.该企业生产的21厘米彩电经过了几次价格大战,出厂价格已经低于成本价。


08.

地理和历史的证据显示美国东部的土也 震与加州相比震中强度上相同,但影响的面积更大。相同震级的土也 震在美国东部打击的区域是加州的100倍。

下列哪一个,假如正确,最帮助解释上面所描述的被影响区域的面积的不同? A.美国东部的建筑物比加州的建筑物更老,因此在大土也 震中易受更大的破坏。

B.加州的地壳,与东部相比较,当土也 震波从震中向外传递时,有更丰富的吸收土也 震波的断层。

C.发生在美国的一些最强并且影响范围最大的土也 震是以美国东部的一些地方为中心发生的。

D.因为发生在美国东部的主要土也 震没有加州那么频繁有规律,难以预测什么时候下一次土也 震可能发生。

E.加州土也 震的原因比东部土也 震的原因更容易理解。 09.

去年6月下旬天气奇热,但京西大学的师生却无法利用学校游泳池消暑,因为京西大学游泳池要到暑期才开放,而暑期则开始于7月上旬。因此,今后为了避免这一问题,京西大学校方应该把游泳池开放的时间定在从6月下旬开始。 上述论证预设了哪项?

A.游泳是消暑的最好形式。 B.京西大学的师生人数太多。

C.6月下旬京西大学游泳池尚未具备开放的条件。 D.京西大学的游泳池总是要开放到暑期结束

E.去年6月下旬的炎热天气对每年同期的气候来说是很典型的。 10.

在过去10年中,登山设备有了一些改进。这些改进使这项运动对于有经验的登山者而言更安全并且更有娱乐性。然而,尽管有这些改进,登山的受伤率在过去10年还是增加了。 假如上面的话都是正确的,下列哪一个,假如正确,最好地解释了他们的明显差别? A.许多登山者,陷入一种虚假的安全感,使用新设备尝试一些他们没有能力做到的高级动作。 B.一些登山受伤的事故是由未预见的天气条件导致的。

C.登山运动,尽管是一种危险的行动,通常不会给有经验的登山者带来伤害。 D.在过去10年,登山技术和登山设备都有一些提高。 E.尽管登山受伤率已经增加了,登山死亡率没有变化。 11.

名牌进入千家万户,家庭电器化、电脑化和“洋货化”已经成为时尚。但同时,人们也发现,这是一个美丽的陷阱:流行是短暂的,掏不完的钱带给消费者的是新的遗憾。聪明的消费者逐渐醒悟:买家电还是要量力而行。

以下哪项,如果以真,最能构成对上述看法的质疑?

A.家电的流行在一定时期内代表了更为先进的技术水平。

B.购买时尚家电所费甚多,财力的消耗提醒我们注意自己的家底。

C30%购买豪华家电的居民并不具备相应经济实力,但却有紧跟消费浪潮的心理。 D.投资电脑获得知识、技能回报,带给消费者的是比遗憾更多的成功和自信。

E.家电价格不断下调,用旧家电换购新家电只需要支付很小的价格补差就可以更新换代。 12.

因为冷冻食品的过程消耗能量,许多人把他们的电冰箱保持半空,仅仅使用它们冷冻已冷冻好的食品,然而半空的电冰箱比全满的电冰箱消耗更多的能量。


下列哪一个,假如正确,最有助于解释上面所描述的明显不同?

A.电冰箱中一定量的空气保持在零度以下比相同量的冷冻食物消耗更多的能量。 B.冰箱的门开的次数越多,就需要更多的能量来保持电冰箱的正常温度。

C.当未冷冻食品被放在电冰箱中,电冰箱内部一定量的空气的平均温度暂时升高。 D.一个通常保持半空电冰箱的人应该使用小型电冰箱来削减能量花费。

E.只有当冷冻空气在电冰箱内部冷冻室中能自由循环时,一个电冰箱才能有效操作。 13.

某国的一份公文如下:“我国应立法限制使用石油制品生产塑料。我国对燃料的需要远远甚于对塑料的需要;而且,我国对进口石油越来越依赖,一旦战争爆发,石油进口被阻断,我国将陷入危机。如果减少制造塑料和石油制品用量,我们就能在独立自主和国家安全上迈出显著的一步。

以下哪项如果为真,最能反驳上述论证?

A.事实上,该国用于制造塑料的石油消耗只占全部石油消耗非常小的比例。 B.某些塑料制品,如飞机、汽车上的部件,对国防有很重要的作用。 C.战争时期,敌国会极力攻占石油供给国地区。 D.新研制的塑料制造方法能有效减少石油用量。

E.利用核能代替石油的研究受到了国际核条约的约束而减慢。 14.

20年前,任一公司的执行官在重新选择公司总部时主要关心的是土地的成本。今天一个执行官计划重设总部时的主要关心的东西更广泛了,经常包括当地学校和住房的质量。

假如上面的信息是正确的,下列哪一个最好地解释了上面所描述的执行官关心方面的变化? A.20年前高质量的住房和学校像今天一样难以发现。

B.某些地区房地产和学校税停止增加,现允许许多人购买房屋。 C.公司执行官在做决定时总是考虑替换方法将怎样影响公司的利润。

D.一个近来人员缺乏的问题迫使公司找到尽可能多的方法来吸引新的雇员。 E.在过去的20年中,一些地区比其他地区土地的价值变化少。 15.

为了减少天然气使用中的浪费,某区政府将出台一项天然气调价措施;对每个用户,包括民用户和工业用户,分别规定月消费限额;不超过限额的,按平价收费;超过限额的,按累进高价收费。该项调价措施的论证报告估计,实施调价后,全区天然气的月消耗量至少可以节10%

为了使上述论证报告及其所作的估计成立,以下哪项是必须假设的? (1)天然气价格偏低是造成该区天然气使用中存在浪费现象的重要原因。 (2)该区目前天然气消费量的至少10%是浪费。

(3)该区至少有10%的天然气用户浪费使用天然气。

(4)天然气价格上调的幅度足以对浪费使用天然气的用户产生经济压力。 A(1)(2)(3)(4)

B(1)(2)(3)(4)都不是必须假设的。 C.仅(1)(2)(4) D.仅(1)(2)(3) E.仅(2)(3)(4) 16.

在一次世界范围内的股市剧跌这后的余波中,T国宣称该国经历的相当严重的股市下跌是由于在下跌之前不久,该国许多行业经历了过快的非国有化过程。


以下哪一项,如果能被执行,将最有助于对T国股票市场严重下跌原因的评估? (A)计算在下跌期间T国个人交易商的平均损失。

(B)利用经济学理论预测T国下一次下跌最有可能的时间。

(C)T国下跌过程中最糟糕的那段时间抛出的股票总数同刚刚下跌前T国抛出的股票总数相比。

(D)T国下跌的严重程度同那些其他经济条件与T国相似,但没有经历最近的非国有化过程的国家下跌的严重程度相比。

(E)把这次下跌对T国货币购买力的长期影响下下跌对T国货币购买力的即时的、更严重的短期影相比。 17.

甲:尽管本地区几年来中学招生人数持续下降,但是小学招生人数却在大幅增加。因此,地区校务委员会提议建造一所新的小学。

乙:另一个方案可以是将一些中学教室临时改为小学学生教室。 下列哪一个,假如正确,最有助于支持乙的方案?

A.一些中学教室不能被改造为适合小学学生用的教室。 B.建造一个中学的成本比造一个小学的成本高。

C.虽然出生率未提高,送孩子去本地区中学的家庭数目显著增多。 D.中学气氛可能危及小学学生的安全和自信。

E.即使在该地区中学人数开始下降以前,有几个中学的教室很少被使用。 18.

甲:R岛上的大学理科毕业生比以往任何时候都更愿意接受在其他领域的永久性工作。这表明了R岛上的科研人员的报酬不高。

乙:不,并非如此。这些毕业生不在科学界工作的原因很简单,因为在R岛上没有那么多的科学工作可供这些毕业生去做。

下列哪一项,假如正确,将最反对甲的观点?

A.不在科学领域工作的理科学位的大学毕业生目前所得薪水比做科研人员的薪水低。 B.获得理科学位的大学生比以前少了。

C.在过去十年,科学界工作机会数目稳定增长。

D.相当数量的理工学位大学毕业生在校时做过低酬工作。

E.每年均有些应届理工学位大学毕业生接受非科学领域的永久工作。 19.

如果一个家庭的人均收入超过所在地区75%的家庭的水平,则称为该地区的富裕户。近10年来,闽江地区富裕户的数量一直在稳步增长。 如果以上断定为真,则以下哪项也必定为真?

A.近10年来,闽江地区的贫富分化现象越来越严重。 B.近10年来,闽江地区的非富裕户的数量不断增长。

C.闽江地区每一个富裕户的人均收入都超过整个地区的人均收入。 D.每一个富裕户的家庭收入都大于任一非富裕户的家庭收入。 E.近10年来,闽江地区富裕户的人均收入一直在稳步增长。 20. 顾问:某受欢迎的杂志每年都要公布一个以若干项标准评判的综合统计分数排名的美国大学名录。然而,这些总分通常不应被学生作为决定申请哪所大学的依据。 下列哪一个,假如正确,最有助于证明顾问建议的合理性? A.大多数购买列出排名的杂志的人是不准备进大学的人。


B.杂志中排名最高的学校将这一事实作为广告以吸引学生。 C.该排名逐年很少变化。

D.某具体标准对任何两个学生的意义可能因他们不同的需要而不同。

E.一些对其所在大学满意的大学学生在他们做出选择去哪所大学读书的决定前考虑的该杂志的排名。 21.

在某纺织企业的一次干部会议上,关于是否让一部分职工先下岗的问题有两种不同意见。方认为:让一部分职工下岗可以减轻企业的经济负担,增加企业的竞争力。

另一方认为:企业职工下岗后,企业要承担政府部分分配的下岗职工保险费。结果还是羊毛出在羊身上,不解决问题。

下面哪项最有力地反驳了反对职工下岗者的意见? A.下岗职工再就业是很困难的。

B.很多没有下岗的企业竞争力越来越弱。

C.下岗职工中很少有为了领取下岗补贴而选择下岗的。

D.政府征收下岗职工的保险费对所有企业都是一视同仁的。 E.用于交纳保险的费用在短期内比给下岗职工的工资高。 22.

总统之所以任命乔治为副总统,保守党人之所以对这项任命感到高兴,是因为他们确信,项任命如果能被上议院通过的话,最高法院中的政治平衡至少在数年内会持续出现有利于右翼势力的倾斜。乔治之所以遭到自由党人的激烈反对是因为他们确信,保守党人是右翼势力。 从上述的议论中,可以推出以下哪项结论? A.乔治是保守党人。

B.如果保守党人不支持对乔治的提名,自由党人将不反对乔治。 C.目前在最高法院中左翼势力占有利地位。

D.如果自由党人不反对乔治,保守党人将不支持对乔治的提名。 E.自由党人确信乔治将作出有利于保守党人的决策。 23.

计算机程序的特别之处在于,它是唯一受专利权和版权保护的产品。专利权保护的是一种发明的创意,而版权保护的是这种创意的表述,但是为了获得两方面的保护,这种创意和它的表述必须得到严格的划分。

根据以上陈述,可以推出如下哪项结论?

A.计算机程序的创意和它的表述可以区分开来。

B.任何计算机程序的设计者都是这一个程序创意的发明者。

C.受版权保护的大部分产品都是对某种受专利权保护的创意的表述。 D.很少有发明家既是专利权的所有者,又是版权的所有者。 E.一个获得了专利权的计算机程序,很容易就可获得版权。 24.

甲:政府在卡瑟纳省通过鼓励创造工作机会、减少失业率的计划已经失败了,因为在该计划实施一年以后失业率仍没有变化。

乙:但是在计划开始前的三年中,卡瑟纳的失业率是上升的,因此该计划是有帮助的。 下列哪一个,假如正确,最强地反对乙对甲论述的反对? A.政府由一些专门研究失业问题的经济学家提供建议。 B.卡瑟纳省的失业率在历史上比整个国家总体失业率高。

C.当前的政府以很大优势当选的原因是许下了为卡瑟纳省减少失业率的诺言。


D.在政府计划开始的时间里,大量卡瑟纳的居民离开该省去其他地方寻找工作。 E.卡瑟纳省的失业率在现任政府当政以前一直相对稳定。 25.

某先生每天早晨的工作内容和工作程序如下,醒来后立即在煤气灶上烧水(10分钟即开)开煤气灶后就开始穿衣服(5分钟即完)然后出门去取牛奶(来回共5分钟)回来后就烧牛奶(10分钟即开),同时开始洗漱(5分钟即毕),最后是喝奶(5分钟喝完)。又知道,他家中只有一个灶头(热源),他又必须在745分离开家,否则将迟到。

如果我们忽略像打开煤气灶等其他占用时间极少的工作,你同意下列哪种观点?

A.该先生上班前有抽一支香烟(5分钟抽完)的习惯,如果他715分醒来并起床,他就没有时间抽香烟了。

B.如果前一天已经取来牛奶,则该先生就可以晚起来5发钟,上班仍不会迟到。 C.如果前一天已经烧好水,则该先生就可以晚起来10分钟,上班仍不会迟到。

D.如果前一天既烧好水,又取来了牛奶,则该先生就可以晚起来15分钟,上班仍不会迟到。 E.只要他每天715分起来,他不但有时间抽香烟(5分钟抽完),而且上班也不会迟到 参考答案:

01.E 02.B 03.E 04.C 05.C 06.E 07.C 08.B 09.E 10.A 11.E 12.A 13.A 14.D 15.C 16.D 17.E 18.A 19.B 20.D 21.D 22.E 23.A 24.D 25.E

01.消费者在选择空调的时候非常重视维修的方便程度。X空调生产厂就在本市提供方便、时的售后维修服务;Y空调生产厂家远在日本,没有在本市设立维修网点。所以,本市消费者一定会选X空调,而不是Y空调。 下列哪项如果为真,能削弱上述的结论? .Y空调的价格比X高。

.X空调维修方便,但是需要维修的次数比Y空调多几倍。 .Y空调是世界名牌,性能享誉全球。 A.仅Ⅰ。 B.仅Ⅱ。 C.仅Ⅲ。

D.仅Ⅰ和Ⅲ。 E.仅Ⅱ和Ⅲ。

02.无酒精饮料的制造商:我们给孩子们新的无酒精饮料蕊波增加了钙质,喝蕊波将帮助孩子健康。

消费者代表:但是蕊波同时含有大量的糖,并且通常认为摄入大量的糖是不健康的,特别是对于孩子。

在对无洒精饮料制造商的反应中,消费者代表做了下列哪一点? A.挑战制造者关于孩子饮食中钙的营养价值的声明。

B.对制造者引用证据的争论,当正确考虑后,导致一个与制造者所得结论相反的结论。 C.暗示产品的制造者通常是不关心那种产品的营养成分的。

D.问题是在适量摄入时属于健康的物质,在过量摄入时是否不健康。 E.提出使制造者结论陷入疑问的附加事实。 03.目前有人设计了一种安装在汽车中的气袋,它能够直接在撞车事故发生时迅速自动膨胀,减轻乘员在碰撞时承受的压力,从而避免伤亡。设计者认为气袋应该像安全带一样,安装在每一辆新汽车里面。有人对此持反对意见,认为:安装气袋设施将增加新汽车的成本,使得国产汽车在竞争时处于不利地位,因为外国政府并没有要求它们的汽车制造商也给新汽车安装气袋。


以下哪项,如果为真,最能削弱上述反对意见的论证?

A.大多数接受调查的汽车司机都认为气袋并没有比安全带增加更多的效能。 B.在十分之九的汽车事故中,使用安全带都极大程度上减少了伤亡。 C.根据对气袋的选择测试,其质量合格率几乎达到100% D.要求国产汽车必须安装气袋的规定同样适用于进口汽车。 E.气袋的研制已经引起了越来越多的国家的注意。

04.许多月来,研究者将一个小光标置于韦特——一种生长在新西兰不会飞的昆虫的背上,使研究者第一次可以对这种昆虫的夜间活动做全面的观测。那么,既然韦特只是在夜间寻食,研究者的观察也就将显著地提高对韦特正常捕食习惯的了解。 下列哪一个是上面论述所基于的假设?

A.研究者仅仅对观察韦特的捕食习惯感兴趣,所以不观察其他类型的行为。 B.韦特在夜间展示的行为不在白天展示。

C.放置小光标在韦特的背上不会显著改变韦特正常的夜间捕食习惯。 D.韦特在研究者对其研究的月份中比其他时间更频繁地捕食。

E.研究者不使用其他观察方法补充他们使用小光标来追踪韦特夜间行为的方法。

05.20年前,从全球来看,因心血管疾病死亡人数在发展中国家还不多见,发达国家中的心血管疾病的发病率却一直呈现上升的趋势。可是到了今天发展中国家中因心血管疾病死亡的人数却在逐渐增加,并逐渐超过发达国家。

如果以上是事实,那么下面哪项最能解释这种现象?

A.发展中国家的经济发展速度要远快于发达国家。人民的生活水平也逐渐提高了。 B.发展中国家里死于其他疾病的人数也比发达国家要多。 C.现在医学的发展已经能够治疗心血管疾病。

D发达国家已经注意到了心血管的危害,注意了防治。而发展中国家的医疗条件还不够好,患者不能得到及时治疗。

E.发展中国家应该吸取发达国家的教训,加紧对心血管病的防治。

06.近来最常被演奏的15部歌剧中不包括19世纪德国作曲家瓦格纳的作品。尽管音乐出品人乐于制作观众想要的作品,而瓦格纳的作品却不在被频繁演出之列,但这并不能说明瓦格纳的作品不受欢迎,而是因为瓦格纳的歌剧的演出费用极其昂贵。 下列哪一个,如果正确,最支持上面的结论?

A.最频繁演奏的歌剧不包括由小的业余团体演奏的歌剧。

B.一些歌剧公司背后有愿意负担大量费用来享受奢华作品的顾客。 C.近来被频繁演奏的所有的15个歌剧是至少已流行75年的作品。 D.最近出品的瓦格纳作品的录音制品比其他歌剧作曲家的多。 E.近年来所有种类的歌剧作品的欢迎程度一直在增加。

07.某宿舍住着四个大学生,分别是广州人、武汉人、上海人和北京人。他们分别在机械、化学和管理三个系就学。其中: I.北京籍大学生单独在化学系。 II.上海籍大学生不在机械系。

III.广州籍大学生和另外某个大学生同在一个系。 IV.武汉籍大学生不和广州籍大学生同在一个系。

以上条件可以推出广州籍大学生所在的系为哪个系? A.机械系。 B.化学系。 C.管理系。


D.机械系或管理系。 E.无法确定。

08.哺乳动物类的侏儒个体的身体相对于非侏儒个体的身体的比例较之侏儒个体的牙齿相对于非侏儒个体的牙齿的比例小。一个成年多毛猛犸象的侏儒最近被发现,牙齿是正常多毛非侏儒猛犸象的3/4

上面的话假如正确,最支持下列哪一个结论?

A.多毛侏儒犸象的身体小于一个正常的成年非侏儒猛犸象身体的3/4

B.最近被发现的多毛侏儒猛犸象的牙齿没有一颗如已被发现的非侏儒多毛猛犸象的牙齿一样大。

C.哺乳动物类的大部分成年侏儒的个体的牙齿是相同种类非侏儒成年个体牙齿的3/4 D.多毛侏儒猛犸象与非侏儒有相同数目的牙齿。

E.大多数哺乳动物类的侏儒个体通常不超过那个种类非侏儒个体大小的3/4

09.跨国制药企业85.4%的人员投入在直接的市场营销和销售,10%在生产和另外4.6%在财务、行政和人事上。而国内的制药企业仅有12%从事直接销售、分销和市场营销上,而75%人员在生产。

从以上论述最能推出以下哪个结论? A.国外,市场营销和销售比生产更重要 B.国内,生产比市场营销和销售更重要

C.跨国制药企业与国内的制药企业相比不够重视生产

D.跨国制药企业与国内的制药企业相比更加重视市场营销和销售

E.不管是在国内还是国外的制药企业,财务、行政和人事都得不到重视

10.近年来全球的青蛙数量下降,而到达地球的紫外线照射量上升。由于青蛙蛋暴露在阳光下,受外线照射时其基因物质会受到伤害,并且因为青蛙蛋是胶质的,没有甲壳或坚韧层的保护,青蛙数量的下降——至少部分上——可能是由紫外线照射的增加导致的。 下列哪一个,假如正确,对论证提供最有力的支持?

A.即使在那些紫外线照射并无显著增加的地区,所产的青蛙蛋也只有很少比例可以孵化。 B.在青蛙数量减少比例最小的地区,青蛙捕食的昆虫物种的数量下降。

C.数量下降的青蛙物种的蛋比数量没有下降的青蛙物种的蛋倾向于含有更高浓度的伤害性的杀虫剂。

D.所产的蛋有坚硬韧性外壳的乌龟同青蛙共同生存的许多地区,乌龟的数量也在下降。 E.将它们的蛋藏于岩石下或沙土下的青蛙物种下降的数量大大小于那些不掩藏它们的蛋的青蛙物种下降的数量。

11.为提高生产力,制造公司近来开始重组工作,以使用更少的装配线工人来生产更多的产品,其结果是公司解雇了许多工人。被解雇的工人是那些工龄最短的,通常为较年轻的工人。 上述陈述,假如正确,最有力地支持如下哪一个结论?

A.虽然制造工作被重组,公司制造的产品并未进行设计的改变。

B.当装配线工人提出改善生产过程的建议时,有一些建议被实施,但许多仍没有实施。 C.装配线工人现在需要提高阅读和数学技能来完成工作。

D.一些旨在提高生产力的装配线过程和程序的创新经证明反而削弱了生产力。 E.制造公司在提高了装配线劳工平均年龄的同时仍寻求提高生产力。

12.梦境是记忆中不连贯的画面组成的,在此基础上构成了一些浅显的故事。但是,这些梦并没有什么心理上的价值,因为它们的产生,完全是当大脑对一天的经历进行分类、对比和贮存时,一种化学过程引起的视觉图像再现。 以下哪项,如果为真,有力地驳斥了上述观点?


A.如果梦境有心理上的价值,就会更连贯,构成的故事也会更精彩。 B.除非做梦者醒来,否则别人根本无法得知梦的内容。

C.如果梦境对于充足的睡眠来说很关键,那么做梦一定有生物学上的根据。 D.如果没有这种化学过程,大脑就无法正常工作。

E.对经历进行分类、对比和贮存,以及大脑中故事的形成都具有心理上的价值。

13.19世纪,英国的城市人口上升,而农村人口下降。一位历史学家推理说,工业化并非是产生这种变化的原因,这种变化是由一系列人口向城市地区的迁移而造成的,而这种迁移都是发生在每次农业经济的衰退时期。为证明这种假说,这位历史学家将经济数据同人口普查数据做了对比。

下列哪一个,假如是正确的,最支持该历史学家的假说?

A.工业经济增长最大的时期总伴随着农业人口数目的相对减少。 B.农业经济最衰弱的时期总伴着总人口数目的相对缓慢增长。

C.在农业经济相对强大,工业经济相对衰弱时期,总伴随着农村人口的快速减少。 D.在农业和工业经济都强劲的时期伴随着城市人口尤其快速的增长。 E.在农业经济最强劲的时期伴随城市人口的相对稳定。

14.在美国费城,有个房产主将一套年久失修、久不使用的破旧公寓租给了一家房客,租约上写明租期为20年。不久,这个房产主改变了主意。他雇佣了一帮黑社会分子不断对房客进行骚扰,迫使房客搬走。正当他计划对旧房进行装修,再以高价出租时,事发,他被指控犯罪,法院判决没收该套公寓,并将所有权转归一非营利机构,用于周济流离失所的家庭。但实际上,法院的这种判决很难阻止类似事情的发生。 以下哪项,如果为真,最能加强上述结论?

A.上述被迫搬走的房客,在房屋产权转移后将被允许搬回居住。

B.上述这样的破旧公寓,如果不加装修,其价值对于该房产主来说,小到无所谓了。 C.该房产主出租未加装修的破旧公寓实际上还是有利可图的。 D.费城的黑社会相当猖獗,雇佣黑社会分子相当容易。 E.上述这样的公寓装修后的租价将是装修前的近10倍。 15.按照联合国报告,西非伊卡拉部落的自杀率(每千人的自杀人数)1992年比1962年高8倍。因为伊卡拉部落从1962年到1992年,由完全封闭的社会变成一个开放社会,自杀率的巨大变化一定是这一社会变化的结果。

下列哪项如果为真,则对以上结论提出最严重的质疑?

A.从完全封闭的社会变成开放社会有时会引起重大犯罪率的变化。 B.甚至在完全封闭的社会里,也有相当一部分社会成员选择自杀。 C.根据联合国的记录,1992年伊卡拉部落自杀者大多为男性。

D.1992年,伊卡拉部落人的寿命不比社会发生变化以前的1962年长很多。

E.1992年,大多数发生在伊卡拉部落的自杀被联合国记录报道,而1962年大多数自杀案例并未记录在案。

16.10年前,大学教师的人均月工资收入约180元,现在这个数字已上升到1200元左右,而其他行业的人均月工资收入只增加了500%,这说明大学教师的收入已经比其他行业要高了。

下列哪项都削弱了上述结论?除了:

A大学教师的学历要求越来越高,在同等学历中,大学教师的收入相比之下还是比较低的。 B.大学教师的兼课收入增长很快。

C.其他行业的工资收入在总收入中所占比例已不足50%,而大学教师的工资收入在总收入中的比例一般占到80%


D.有个别名牌大学教师的开始有岗位津贴,但在绝大部分高校里,大学教师的岗位津贴还不如一般行政人员的岗位津贴高。

E10年前,其他行业的月人均工资收入是260元。

17.政治家:每年,小企业比大型老牌公司要创造更多的就业机会。因此,为减少长期的失业率,我们应当鼓励推动兴办小企业而不是扩大老牌的大公司。 下列哪一个,假如是正确的,对政治家的论点提出最大质疑? A.通常,小企业的雇员比大公司的雇员对工作的满意程度要高。

B.在当前失业人员中有许多人具有足够的完成小企业提出的工作要求的技能。 C.提供创办企业的有效刺激通常比为扩大一个大公司提供的有效刺激要小得多。 D.有很大比例的小公司在开办3年内由于他们的所有者缺乏经验而倒闭。 E.一般大公司给社会捐款比一般小企业多。

18.汽车电话越来越受欢迎了。因为有了它,人们就能在开车的时候打电话了,司机们将不会如以前那样寂寞;此外,如果不幸有事故发生,汽车司机可以立即打电话请求帮助。不过,汽车电话还是应该禁止使用,因为在驾车时使用汽车电话,司机容易分心,从而导致危险。 根据以上陈述,可推出作者最支持以下哪种说法?

A.汽车电话之所以受欢迎,主要是因为在事故发生时,司机能够用之呼救。

B.如果使用汽车电话,由于司机在拨号时无法双手握住方向盘,因而会造成危险。 C.与驾车时可能导致的危险相比,汽车电话带来的方便是微不足道的。 D.打电话会分散司机的注意力,这证明是很危险的。 E.开车时能通过电话与他人联系,真是太方便了。

19.在工作场所,流感通常由受感染的个体向他们附近区域内共同工作的人们传播。因此,一种抑制流感症状的新型药物由于使本应躺在家中床上的病人在染病时返回工作,而实际上将增加流感病例数。

下列哪一个,假如正确,是严重质疑这个预测?

A.咳嗽为新药抑制的一种流感症状,是疾病传播时的一个主要方式。

B.一些用来抑制流感症状的药物也被许多人用来治疗不是由流感引起而是由其他疾病引起的症状。

C.许多染上流感呆在家里的工人这样做是因为流感症状使他们不能有效地工作。

D.大多数为避免感染而对流感进行免疫的成年人都已经超过65岁且退休,因此不在外面工作。

E.疾病的症状往往是身体治疗自身疾病的方法,因此抑制症状会延长引起症状的疾病的病程。

20.相比那些不参加登山运动的人,经常参加登山运动的人一般都健康一些。可见,登山运动能锻炼身体,增进身体健康。

以下哪项最有力地对上述论证提出了质疑?

A.无论什么人参加登山运动都是为了锻炼身体,增进健康。

B.身体健康状况不好的人大多都不会参加具有一定危险性的登山运动。

C.锻炼身体、增进健康有许多方式,例如,参加长跑、球类活动、游泳等都能锻炼身体,增进健康。

D.登山是一项十分危险的运动,每年都有一些人在登山运动中受伤,甚至有些人的伤势还很严重。

E.改善健康状况不是一件简单的事,需要从多方面着手进行,而且还需要一个漫长的过程。 热点推荐:

300分奇迹MBA逻辑模拟题[第十二套]


作者:站长 来源:教育联展网 更新:2007-8-18 阅读:47

21.广泛使用的小剂量镇静剂,允许人们在测谎测验中撒谎而不被发现。测谎测验所测试的压力能够被这种药有效地抑制住并且没有显著的副作用。这个事实的一个附加应用是:这种药在减少日常生活的压力时同样有效。 上文基于下列哪一个假设?

A.镇静剂总是对压力的一种有效治疗。 B.压力反应的抑制增加主观的压力。

C.测谎器所测量的压力与日常生活的压力类似。 D.在测谎器测试中撒谎的人总是显示压力信号。 E.减少日常生活的压力是不理想的。

22.甲:恐龙灭绝的原因在于全球性的气候剧变极大地减少了以前丰富的食物来源。 乙:不对,恐龙的灭绝是由于出现了新的动物家族——哺乳动物,它们繁殖迅速,动作敏捷,生存力极强,成为与恐龙争夺食物的致命对手。

丙:曾经发生过行星撞击地球,引起烟云蔽日达数十年之久,大量的作为恐龙食物的植物相继枯亡。

以下哪项最为恰当地概括了上述三人的意见?

A.三人都同意饥饿是引起恐龙灭绝的原因,但对引起食物短缺的原因有不同的意见。 B.三人都同意气候的改变引起了恐龙食物的短缺,但在食物短缺如何造成恐龙灭绝这一问题上有不同的看法。

C.三人都同意气候巨变引起了恐龙的灭绝,但对气候剧变的原因有不同的解释。 D.三人都认为恐龙的生存竞争力极差,由此导致了它的灭绝。 E.三人都肯定恐龙的灭绝是由于外星体作用地球的结果。

23.学生应该从小时候就开始学哲学。不然的话,他们会不假思索地接受某些传统价值观,哲学正是教会他们对这些价值观提出质疑。 上述议论以下面哪项为假设?

A.学生能对传统价值观提出质疑未必是件好事。 B.哲学是学生小时候首要应该学的知识。 C.传统价值观在现代社会已变得没有意义。

D.除非学生从小就学哲学,否则他们就会接受任何观点。

E.即使在很小的年龄,学生也具有理解某些哲学概念和哲学理论的能力。

24.19601970年间,非洲国家津巴布韦境内猎捕了6500多头大象获取象牙,这一时期该国大象总数从35000头下降到30000头以下。1970年该国采取保护大象措施,1970年—1980年期间逮捕并驱逐了800多名狩猎人。但是,1980年该国大象总数还是下降到21000头。 下列哪项如果为真,最有助于解释上述表面上的矛盾现象? A.19601980年间逮捕的狩猎者并未被判处长期徒刑。

B.公众反对滥捕大象呼声高涨,19701980年间象牙的需求下降。 C.该国的一个领国19701980年间大象数量略有回升。 D.1970年以前,该国反对捕杀大象的法律没有得到执行。 E.19701980年间,该国大量砍伐了大象赖以生存的森林。

25.国家的政府官员和公民对于政府在其行动中负有义务遵守规则的理解是相同的。因此,如果一个国家故意无视国际法,该国的政府官员的态度也会变得不支持他们的政府。 上面的论述基于下列哪一个假设? A.人民对政府义务的理解经常改变。

B.上个国家的公民将赞成其政府通过合法的方式发展其国际影响力的努力。


C.极权主义政府的一些官员对国际法中所体现的规则是不敏感的。 D.每个国家的公民相信国际法是政府应当遵守的规则之一。

E.当选的政府官员比任命的政府官员更可能怀疑他们自己政府行动的明智性。 参考答案:

01.E 02.E 03.D 04.C 05.D 06.D 07.C 08.A 09.D 10.E 11.E 12.E 13.E 14.B 15.E 16.B 17.D 18.D 19.A 20.B 21.C 22.A 23.E 24.E 25.D

01. 投资者越是怕投资失败赔钱,就越对他们的投资效益抱高的企求。高风险必须由高效益来补偿。这也恰是决定贷款利率的基本原理。以下事实就说明了这一点。 以下哪项作为上文的后继最为恰当?

A.成功的投资者区别于他人之处在于,他们在作风险投资时不惧怕赔钱。 B.放贷者向不安全的借贷者收高利息,向有抵押的借贷者收低利息。 C.在高通货膨胀期间,银行给存款的利息可能实际上比通货膨胀率低。 D.在同一时间,商业银行对所有的借贷者实行相同的利率。 E.新企业的投资回报一般要低于经营良好的老企业。

02. 科学家相信过去被认为安全的人工髋关节的移植手术,在使用45年后实际上增加接受移植者患癌症的危险。尽管这些移植确实提高了移植接受者的生活质量,但为获得这些提高而增加癌症危险是不可接受的代价。因此,它们应当被禁止。 下列哪一个,假如正确,是对上面论述的一个最好地反对?

A.人工髋关节的移植手术能够导致剧烈的并发症,像感染、慢性热、骨质腐败,并且这些并发症可能致残甚至是致死的。

B.几乎所有的人工髋关节手术的移植接受者在接受移植的时候,已不大可能再活30年。 C.尽管人工髋关节的手术在使用45年后增加癌症的危险,它们导致的一些癌症不是致命的。 D.因为人工髋关节的移植手术并不非常普遍,禁止它们只会有很小的困难。

E.尽管人工髋关节移植手术的好处在过去10年基本保持相同,手术的价格却增长了许多。 03. 在毕业考试的五门课程中,张云和李山只有数学成绩相同,其他科的成绩互有高低,但所有课程的分数都在80分以下。在录取时只能比较他们的总成绩了。 下列哪项如果为真,能够使你判断出张云的总成绩高于李山? A.张云的最低分比李山的最低分高。 B.张云的最高分比李山的最高分要高。 C.张云的最低分比李山的平均成绩高。

D.张云的最低分比李山的两门课分别的成绩高。 E.张云的最低分是数学,而李山的最低分是英语

04. 威廉镇的学校董事会计划继续在威廉高中提供拉丁文课程,尽管这些课参加的学生很少。该董事会认为拉丁文课的高成本是值得的,参加拉丁文课可以给需要提高英文水平的学生带来很多好处。

下列哪一个,假如正确,最反对学校董事会的论述?

A.学校的拉丁文老师可以教西班牙语,可以被重新安排到更受欢迎的西班牙语课中。 B.需要提高英文水平的学生能够从上其他的语言课中获得上拉丁文课一样多的好处。 C.在拉丁课中表现良好的大多数学生在其他课中同样优秀。 D.英语课中表现良好的学生在拉丁文课中表现不佳。

E.因为拉丁语在现代世界中的任何地方都不再被广泛使用,即使学者也不能确定其正确发音。

05. 一份调查报告显示,北京市现有的老年人中,只有1%表示愿意去老年公寓去安度晚年,这个比例实在是太小了,因此现有的老年公寓应该关门了。


以下哪项如果是真的,将最严重地削弱以上结论?

A.在老年公寓,老人们可以享受到良好的医疗服务。

B.有50%以上的老年人虽然自己不愿意去老年公寓,但是对别的老人去老年公寓还是表示理解的。

C.老年公寓的档次越来越高了。

D.按北京现有的老年公寓的容量,只能接受北京市现有老年人的千分之五。 E.老年人的子女不在身边的比例越来越大了。

06. 注意到某一城市1982年的犯罪数目比1981年下降了5.2%,警察局长说:“我们看到这是城市中1982年初开始推行改革性的警察制度的结果。 下列哪一个,假如正确,最反对上面警察局长得到的结论?

A.一些城市最近增加在警察项目上花费,其犯罪数目相对于1981年没有下降。 B.该城市的犯罪数目每年由相同的方法从被报道的犯罪数目中估算出来。 C.该城市周围的郊区犯罪的数目1982年相对于1981年增加了5%并且几乎与该城市中1982年的犯罪数目相同。

D.1982年该城市犯罪的数目比1972年该城市的犯罪数目高了10%

E.该城市中容易犯罪的年龄组1982年比1981年显著地下降,因为出生率下降了。 07. 刘大爷看见邻居丁大爷又在喝酒,忍不住再次苦口婆心地劝说丁大爷。刘大爷:“老丁,老说过你多少次了,你可不能再喝这么多酒了!要知道,酒精确实对你是非常有害的。”丁大爷抬头看了一眼刘大爷:“老刘,你错了。我这样喝了整整30年了,但从来没有喝醉过。 以下哪项,最能加强和支持刘大爷的意见?

A.许多人和丁大爷一样喝酒,却经常喝得酩酊大醉,人事不醒。 B.酒精并非一沾就醉,这取决于各个人的酒量。 C.醉酒并不是喝酒对人有害的惟一原因。

D.如果不断喝白酒,那么酒量将越来越大,越来越不容易醉。 E.白酒的酒精含量最高,喝其他的酒,也许要好得多。

08. 枫糖的价格已经从三年前的每加仑22美元跳涨到今天的每加仑40美元,由此可推导出枫糖的收获者已经人为哄抬了价格,政府的价格管理法规必须控制上升的价格。 下列哪一个,假如正确,最反对上面所得的结论?

A.政府已经要求枫糖收获者将他们的设备提交到市健康部注册。

B.病虫害和干旱已经阻碍了产糖枫树的生长,并且导致了枫糖收获量的减少。 C.枫糖由失业率很高的乡村地区出产。

D.枫糖收获和技术进步已经降低了生产成本。

E.枫糖价格在过去已经增加了许多次了,尽管从未超过最近所观察到的增长率。 09. 企业发展到一定程度,一旦在某外地市场建立了一套广泛的销售网络并取得销售的显著增长,就应该在外地市场上采取与本地类似的营销策略。因此,在开创初期,或在才建立了办事处的外地市场上,需采取与本地不同的营销策略。 以下哪项如果为真,则最能支持文中结论? A.广泛的销售网要优于不发达的销售网。 B.某些地区经济比其他地区发展迅速。

C.外地市场与本地市场上的销售网络可以完全相同。

D.研究市场营销必须充分考虑不同市场上广告的适应能力。

E.比较起来,大型的外地市场比小型的外地市场更能适合采用本地营销策略。 10. 由街头暴力犯罪而被囚禁的人在释放以后经常重复相同类型的犯罪。由诸如受贿和贪污等白领犯罪而被囚禁的人在释放以后不再重复他们的犯罪。可以公平地推导出,囚禁,通常


不能改变街头的暴力犯罪的行为,却能够成功的使那些白领犯罪不再愿意重复他们的犯罪。 下列哪一个,假如正确,最反对上面所陈述的结论? A.统计展示,被控白领犯罪的人很少有监禁的记录。

B.白领犯罪并且因此而被囚禁的人的比率低于因街头暴力犯罪而被囚禁的人。 C.被减刑的白领罪犯比服满刑的白领罪犯重复犯罪的比例高。

D.白领犯罪后被释放的人很少被给予很高的职位或接近其他人的钱。 E.暴力犯罪的人很少白领犯罪,反之亦然。

11. 延长的不合季节的霜冻会在生长的温带的落叶树上产生霜冻年轮。在南极洲发现的落叶树的化石中没有一个有霜冻年轮。因此,在南极洲,当这些已形成化石的树木长长的时候,不大可能发生这种霜冻的现象。 以上论述依据下面哪个假设?

(A)南极洲的一些形成化石的非落叶性树木上有霜冻年轮。 (B)落叶树比其他树种更容易产生霜冻年轮。

(C)形成化石的过程不会使落叶树中的霜冻年轮完全模糊。

(D)现在的落叶树比古代南极洲的落叶树对温度的变化更敏感。

(E)在现已成为化石的树木还生存着的时期,在南极洲发生的延长的不合季节的霜冻不总是在落叶树上产生霜冻年轮。

12. 根据官方的政府记录,1987年格陵兰岛的自杀率(每千人中自杀的数目)1960年高17倍。因为格陵兰岛在1960年至1987年间由一个捕鱼捕猎的社会转向一个工业社会,急剧增加的自杀一定是这种社会改变的结果。

下列哪一个,假如正确,最反对上面得出的结论?

A.从一个捕鱼、捕猎社会向工业社会的转变有时导致社会上犯罪率的增加。 B.即使在捕鱼捕猎社会,一些社会成员仍会选择自杀。

C.根据官方的政府记录,1987年格陵兰岛自杀的大多数人是男性。

D.格陵兰岛居民的寿命在1987年并不比社会变化前的1960年的人的寿命长。 E.1987年,大多数发生在格陵兰岛的自杀被上报到相关的政府部门,1960年不是这样。 13. 我国已故著名逻辑学家金岳霖小时候听到“金钱如粪土”“朋友值千金”这样两句话后,感到这两句话有问题,因为它们会推出“朋友如粪土”的荒唐结论。 根据以上信息,可以从逻辑上推出如下哪项结论? A“金钱如粪土”“朋友值千金”这两句话都不对。 B“金钱如粪土”这一说法是假的。 C“朋友值千金”这一说法是假的。 D“金钱如粪土”“朋友值千金”这两句话都对。

E.如果朋友确实值千金,那“金钱如粪土”这句话不成立。

14. 20世纪50年代的一段时间,摇滚乐把爵士乐永久地赶出音乐舞台。证据来自于那段时间年轻人的行为。在拥挤的夜总会中,他们热情地为摇滚表演鼓掌。但是当爵士乐开始时,他们就出去并且吃一些小吃。只有当爵士表演结束后他们才回来。 下列哪一句话,假如正确,对上面得到的结论做了一个合理的反对? A.爵士乐是美国对于世界文化的最重要的音乐贡献。

B.尽管20世纪50年代参加夜总会的一些年轻人确实试图听爵士乐,他们最终对其厌烦。 C.20世纪50年代,摇滚乐不仅为年轻人提供娱乐,而且也成为社会演说时一个振奋人心的因素。

D.尽管20世纪50年代,爵士表演不那么流行,但是从那里起在中产阶级的职业人员中对爵士乐的兴趣开始复苏。


E.爵士乐在20世纪30年代到20世纪50年代间越来越受欢迎。

15. 甲:世界卫生组织去年的统计数据显示,威尼斯城因癌症死亡的人数比例比世界城市的平均值要高出两倍。而历史上威尼斯城是以癌症特别是肺癌的低发病率而著称的。看来,尼斯城近年来在防治癌症方面出现了失误。 乙:威尼斯城的癌症死亡的人数比例提高的主要原因,是它秀丽的风光吸引了世界各地众多的游客,包括各种危重病人,许多癌症患者在这里度过了他们生命的最后时光。 以下各项如果为真,都能构成对乙的见解的质疑,除了 A.据统计,威尼斯城的癌症死者,一直大多是本地居民。

B.威尼斯城有些居民,改变了以往的饮食习惯,变得喜欢吃烧烤食物。医学已经证明,此类食物含有致癌物质。

C.世界许多著名城市,如日内瓦、苏黎世和夏威夷,同样具有吸引世界各地众多游客的秀丽风光,但癌症死亡的人数比例并不高。

D.据统计,作为人类的头号杀手的心血管病引起死亡的人数比例,在威尼斯并不高。 E.患有危重疾病的患者,尽管喜欢到异国的旅游胜地去疗养,但大都不愿客死他乡,且癌症属于消耗性疾病,患者突发死亡的概率很小。

16. 如果起6级以上大风,就不能施工。现在不能施工,因此,一定是起6级以上大风了。以下哪项与以上论证是相同的结构?

A.如果马拉多纳上场,阿根廷队就一定会赢。阿根廷队输了,所以马拉多纳肯定没上场。 B.所有的证据都可被接受除非它被破坏。这个证据不能接受,因此,它被破坏了。

C.如果某甲犯罪了,他的指印可以在现场找到。某甲的指印在现场被找到了,所以,某甲犯罪了。

D.老葛是我的叔叔,小菲是老葛的侄女。因此,小菲是我的姐姐。

E.阿森将戴太阳眼镜,如果海岸可被清楚地看见。海岸可被清楚地看见,因此,阿森将戴太阳眼镜。

17. 根据最近一次调查,婚姻使人变胖。作为证据的是一项调查结果:在13年的婚姻生活中,女性平均胖了23磅,男性平均胖了18磅。

对下列哪一个问题的回答可能对评价上面调查中所展现的推理最有帮助? A.为什么调查中研究的时间是13年,而不是12年或14年。 B.在结婚的时间里,有一些男性体重增加少于18磅吗?

C.与调查中的有可比较年龄的单身汉在13年中的体重增加或减少了多少? D.调查中的女性和调查中的男性在调查中一样积极吗? E.报道中的体重获得将维持一生吗?

18. 一个最近州内人力资源的调查发现文秘学校毕业生的寿命预计超过其他高中毕业生的寿命四年。一个可能的结论是参加文秘学校有益于一个人的健康。 在评价上述结论时,最重要的是回答下列哪一个问题?

A.高中毕业生的平均年龄和文秘毕业生的平均年龄近年来增加了吗? B.一位文秘学校的毕业生有大学学位吗?

C.女性比男性寿命长,在高中和文秘学校毕业生中男性和女性的相对比例是多少? D.女性比男性寿命长,占多少比例的女性上了文秘学校? E.参加文秘学校的高中毕业生的比例近年来增加了吗?

19. 最近的一项调查显示:某公司的许多工人对他们的工作不满意。调查同时显示:大多数感到不满意的工人认为他们对自己的工作安排没有自主权。因此,为了提高工人对工作的满意程度,公司的管理层仅仅需要集中改变工人对他们工作安排自主权程度的观念。 下列哪一个,假如也在调查中被显示,最能使段落作者所做的结论有疑问?


A.不满意的工人感到他们的工资太低并且工作条件不令人满意。 B.公司中对工作满意的工人的数目比对工作不满意的工人数目多。 C.该公司的工人与其他公司的工人相比,对他们的工作更不满意。

D.公司管理层的大多数人相信工人对他们的工作已经有太多的控制权力。

E.该公司中,对工作满意的工人认为他们对自己的工作安排有很多控制的权力。

20. 一些人对某法官在针对妇女的性别歧视案中的客观性提出疑问。但是有记录表明:在60%的这类案件中,法官的决定对妇女有利,这个记录表明法官在针对妇女的性别歧视案件中并没有歧视妇女。

下列哪一个,假如正确,最反对上面得出的结论? A.该法官的大量案件是针对妇女的性别歧视的控诉。

B.许多法官发现在针对妇女的性别歧视案件中难以保持客观性。 C.该法官在非性别歧视案件中,对女性被告或原告有偏见。

D.大多数送到该法官法庭的针对妇女的性别歧视案件是由一个较低法庭上诉的。

E.有证据表明,妇女应当在该法官处理的70%以上的针对妇女的性别歧视案件中胜诉。 参考答案:

01.B 02.B 03.C 04.B 05.D 06.E 07.C 08.B 09.E 10.D 11.C 12.E 13.E 14.D 15.B 16.C 17.C 18.C 19.A 20.E

1“平反是对处理错误的案件进行纠正”

依据以下哪项能最为确切他说明上述定义的不严格?

A.对案件处理是否错误,应该有明确的标准,否则不能说明什么是平反。 B.应该说明平反的操作程序。

C.应该说明平反的主体,平反的主体应该具备足够的权威性。 D对平反的客体应该具体分析,平反了,不等于没错误。

E.处理错误的案件包括三种:重罪轻判,轻罪重判和无罪而判。

2.某公司财务部共有包括主任在内的8名职员。有关这8名职员,以下三个断定中只有一个是真的:

Ⅰ有人是广东人。 Ⅱ有人不是广东人。 Ⅲ主任不是广东入。 以下哪项为真?

A8名职员都是广东人。 B8名职员都不是广东人。 C.只有一人不是广东人。 D.只有一人是广东人。

E.无法确定该部广东人的人数。

3.按照上帝创世说,上帝在第一天创造了地球,第二天创造了月亮,第三天创造了太阳。因此,地球存在的头三天没有太阳。

以下哪项最为确切地指出了上述断定的逻辑漏洞? A.没有太阳,一片漆黑,上帝如何创造地球?

B.上帝创世说是一种宗教想像,完全没有科学依据。

C.上述断定带着地球中心说的痕迹,在科学史上,地球中心说早被证明是错误的。 D.天体史揭示星球的形成确实有先后,但没有证据说明太阳比地球早形成。 E“一天”的概念是由太阳对于地球的起落周期来定义的。 45题基于以下题干:


某岛上男性公民分为骑士和无赖。骑士只讲真话,无赖只讲假话。骑士又分为贫穷的和富有的两部分。有一个姑娘只喜欢贫穷的骑士,一个骑士只讲一句话,使得这姑娘确信他是一个贫穷的骑士。另外,姑娘问任何一个男性公民一个问题,根据回答就能确定他是否为贫穷的骑士。

4.以下哪项可能是该骑士所讲的话? A.我不是无赖。 B.我是贫穷的骑士。 C.我不是富有的骑士。 D.我很穷但我不说假话。 E.我正是你所喜欢的人。

5.以下哪项可能是姑娘的问话? A.你是富有的骑士吗? B.你是无赖吗?

C.你是贫穷的骑士吗? D.你说真话吗? E.你说假话吗?

6.在某餐馆中,所有的菜点或属于川菜系或属于粤菜系,张先生的菜中有川菜,因此,张先生点的菜中没有粤菜。

以下哪项最能增强上述论证?

A.餐馆规定,点粤菜就不能点川菜,反之亦然。

B.餐馆规定,如果点了川菜,可以不点粤菜,但点了粤菜,一定也要点川菜。 C.张先生是四川人,只喜欢川菜。 D.张先生是广东人,他喜欢粤菜。 E.张先生是四川人,最不喜欢粤菜。

7.对某生产事故原因的民意调查中,70%的人认为是设备故障,30%的人认为是违章操作,25%的人认为原因不清,需要深入调查。

以下哪项最能合理地解释上述看来包含矛盾的陈述? A.被调查的有125个人。

B.有的被调查的人改变了自己的观点。

C.有的被调查者认为事故的发生既有设备故障的原因,也有违章操作的原因。 D.很多认为原因不清的被调查者实际上有自己倾向性的判断,但不愿意透露。 E.调查的操作出现技术性差错。 8“如果货币的储蓄额和销售回笼额都没增长,那么货币的证券投资额一定增长”,以 以下哪项能作为该增加的前提?

A.货币的证券投资额事实上增长了。 B.货币的证券投资额事实上没增长。 C.货币的销售回笼额事实上没增长。

0.货币的销售回笼额和证券投资额事实上都没增长、 E.货币的销售回笼额和证券投资额事实上都增长了。

9某银行失空窃,职员甲涉嫌被询问,保安人员的第一个问题就是:“你以后还敢不敢再偷了?”

上述提问方式和下列哪项最为类似?

A.小明考试粗心,数学只得了90分,爸爸问他:“你以后还粗心吗?” B老张花了大笔钱游玩某地,结果大失所望,老李幸灾乐祸,问老张:“你以后还去吗?”


C.小赵酒后驾车,结果翻车住院,还被罚了款,小赵爱人又气又急,问:“你以后还敢再酒后驾车吗?”

D.某歌舞厅因提供////务被查封,半年后复业,执法人员问老板:“你以后还敢不敢再犯了?” E“文革”中,在一次批斗会上,造反派质问被批斗的老干部:“你以后还敢不敢再走资本主义道路了?”

10.调查表明,最近几年来,成年人中患肺结核的病例逐年减少。但是,以此还不能得出肺结核发病率逐年下降的结论。

以下哪项如果是真的,则最能加强以上的结论?

A.上述调查的重点是在城市,农村中肺结核的发病情况缺乏准确的统计。 B.肺结核早就不是不治之症。

C.和心血管病、肿瘤病等比较起来,近年来对肺结核的防治缺乏足够的重视。 D.近年来未成年人中的肺结核病例有所上升。 E.防治肺结核病的医疗条件近年来有很大的改善。

11.某国20年前居民用于吃的费用占收入的90%,目前降低为40%。 如果上述断定是真的,那么以下哪项最有可能是真的? A.该国居民吃得比20年前要好多了。 B.该国居民吃得不如20年前。 C.该国居民吃得和20年前差不多。

D.扣除物价上涨因素,目前该国居民花在吃上的费用比20年前要少。 E.扣除物价上涨因素,目前该国居民花在吃上的费用和20年前差不多。

12.在经营领域,工商企业尝试向消费者承诺。这种经营承诺能否继续进行下去,取决于消费者是否支持。在经营领域现场调查显示,只有25.49%的消费者认为经营承诺活动“成效大,应该搞”;有6.23%的消费者认为经营承诺活动“没用,流于形式”;有4.59%的消费者认为经营承诺“法约束力不够”;而48.46%的消费者对经营承诺活动持观望态度。 以下哪一项可以构成近半数消费者对经营承诺活动持观望态度的理由? A.经营承诺不符合市场经济运行规则。 B.经营者根本不懂违诺责任。

C.经营承诺活动本身是件好事,关键看如何兑现。 D.经营承诺活动的开展促进了企业改善经营。 E.经营承诺活动缺少约束机制。

13.香港的繁荣是事实。英国对香港的统治也是事实。有人由此得出结论:是英国的统治造成了香港的繁荣。

以下哪项最有力地削弱了上述结论?

A.香港的繁荣仅是近几十年的事,而英国的统治已达百年。 B.英国本土的经济一直处在不景气和衰退之中。 C.作为香港人主体的广东人素以善于经营著称于世。 0.亚洲四小龙的其他三小龙,并非一直受英国的统治。 E.香港的繁荣得益于它的国际金融中心的地位。 14.时下炙手可热的所谓“影星”“歌星”实际上称不上真正的明星,他们没有无愧于受到大众关注、崇拜的艺术成就、文化素养以及个人品格。 上述议论可得出以下所有结论,除了:

A.现在有些不合格的“明星”也受到了大众的关注和崇拜。

B.真正的明星应该具备那些所谓的“明星”尚不具备的艺术成就、文化素养和个人品


C.真正的明星无愧于大众的关注和崇拜。 D.现在的“影星”、歌星”大都有愧于大众的关注和崇拜。 E.目前大众识别明星的能力尚待提高。 15“常在河边走,哪有不湿鞋”。搞财会工作的,都免不了有或多或少的经济问题,特别是在当前商品经济大潮下,更是如此。

以下哪项如果是真的,最有力地削弱了上述断定?

A.以上断定,宣扬的是一种“人不为己,天诛地灭”的剥削阶级世界观。

B随着法制的健全,以及打击经济犯罪的深入,经济犯罪已经受到严厉的追究与打击。 C.由于进行了两个文明建设,广大财会人员的思想觉悟与敬业精神有了明显的提高。 D.万国投资信托公司房产经营部会计胡大全,经营财务30年,分文不差,一丝不苟,并勇于揭发上司的贪污受贿行为,多次受到表彰嘉奖。 E“慎独”是中国的传统美德。这种传统美德,必将发扬光大。 参考答案如下:

1E 2 A 3 E 4 C 5 A 6 A 7 C 8D 9E 10D 11A 12C 13B 14D 15D

01. 一批年龄不同的孩子在听故事。故事中有人做坏事;做坏事的人中,有的是故意的,有的是无意的。当问及该如何惩罚这些做坏事的人时,年龄小的孩子就不同于年龄大的孩子,他们不加区别地对做同类坏事的人设计同样的惩罚,不管他们是故意还是无意。这说明,龄小的孩子在考虑对做坏事的人的惩罚时天然地倾向于持“唯效果论”的观点。作为一种和“唯动机论”极端对立的伦理学观点,“唯效果论”认为,尽管某种行为的结果可能由不同的动机造成,但对这一行为的道德评价,只能依据结果本身,而不能依据不同的动机。 以下哪项如果为真,最能削弱上述结论?

A.小孩子对不同的坏事能设计不同的惩罚。

B.有些大孩子甚至大人也认为惩罚坏事应当只考虑结果而不考虑动机。 C.区别不同的动机需要有较为成熟的理解和分析能力。 D.小孩子和大孩子一样都有可能无意识地干坏事。 E.在上述这些故事中,坏事的危害性表现得非常清楚。 02. 某市一项对读书爱好者的调查表明,那些称自己每周固定去书店一至二次的人近两年来17%增加到24%而对该市大多数书店的调查则显示,近两年来书店的顾客人数明显下降。 以下各项如果是真的,都有助于解释上述看来矛盾的断定,除了: A.调查统计过程出现失误。

B.图书馆阅览室的兴起抢了书店的生意。 C.上书店没什么规律的人在数量上明显减少。

D.书店出于非正常的考虑,往往少报光顾的人数。

E.受调查的逛书店爱好者只占全市逛书店爱好者的10%

03. 根据古代的记录,塞勒地区政府对基本商品征的第一个税是,塞勒地区出售每罐食用油征收两个生丁,税务记录显示:尽管有稳定的人口和严格税法实施方法,食用油税的收入在税法生效的头两年中显著下降了。

下列哪一个,假如正确,最帮助解释塞勒地区的食用油税收的下降? A.在税法实施后的10年,塞勒地区的平均家庭收入稳定增加。

B.在实用油税实行后的两年,塞勒地区政府开始在许多其他基本商品上加税。

C.在塞勒地区,食用油罐传统上被用作结婚礼物,在税法实施后,食用油的礼物增加了。 D.在加税后,塞勒地区的商人开始销售比税前更大罐的食用油。 E.在加税后,很少塞勒地区的家庭开始生产他们自己的食用油。


04. 北京某冷饮店在二月份开展买一赠一的促销活动,即买一盒冰淇淋送一块冰砖。 下列中除哪一项外都对这一活动的有效性提出了质疑?

A.二月份天气比较寒冷,几乎没有人能吃得完一盒冰淇淋和一块冰砖。 B.自从开展这一活动,许多老顾客带上他们的朋友到这家冷饮店。

C.许多顾客都是老主顾,他们选择哪家冷饮店是相对固定的,不会太受其他因素的影响。 D.冷饮店的冰砖是外地产的,不适合北京人的口味。

E.自从开展促销活动以来,冰淇淋的单价明显高于其他店。

05. 暴露于高温之下,房屋建筑材料会发出不同的声音。声音感应器精确地探测这些声音,并用应用声音感应器的火灾报警器能够提供一个房屋火灾的早期警报,让居住者在被烟熏倒前逃脱。因为烟熏是房屋火灾造成死亡的最通常的原因,要求使用基于声音控测器的警报系统而不是烟雾探测器将消除作为主要死亡原因的房屋火灾。 下列哪一个,假如正确,最反对上面的论述?

A.假如基于声音感应器的报警系统广泛使用的话,其高成本将下降。

B.当完全燃烧时,许多用于房屋建筑的材料发出的声音几百码之外也可以听得见。 C.许多火灾开始于座垫和床垫,产生大量的烟雾而不发出声音。 D.在一些较大的房屋中,两个或两个以上的声音探测器基于的报警器被需要以达到足够的保护。

E.在它们普遍使用后,烟雾探测器拯救了许多生命。

06. 为了帮助家长选择最满意的幼儿园,有关部门实施一项评选“最佳幼儿园”的活动。方法是,选择十个指标,包括教师水平、师生比例、教学方式、教学设施、清洁程度、绿化程度、饮食卫生、家长满意度等等,每个方面按实际质量的高低,评比从1分至100分之间的某一分值,然后求得的十个分值的平均数即是这个幼儿园的质量指数。 以下哪项是实施以上活动需要的前提?

.幼儿园的各种指标的质量程度都可以用准确的数字表达。 .幼儿园的各种指标对于居民来说都是同等重要的。 .家长有自由选择幼儿园的权利,并且大都乐于这样做。 A.仅Ⅰ。 B.仅Ⅱ。 C.仅Ⅲ。 D.Ⅰ和Ⅱ。 E.Ⅰ,Ⅱ和Ⅲ。

07. 199212月,底特律的购物中心修理并且改善了购物中心停车场的照明。1993年,停车场的偷车案和试图偷车案比前一年下降了76%。因为小偷通常被好的照明条件所阻止,偷车案的下降可以归功于这些改善。

下列哪一个,假如正确,最支持上述论据? A.1992年和1993年,大多数购物中心停车场的偷车案所盗窃的车是相对新和昂贵的豪华型。

B.大多数购物中心的盗车案发生于上午10点到下午4点间。 C.底特律购物中心是底特律地区全部的三个购物中心之一。

D.在购物中心所在的底特律镇,汽车盗窃案在1993年和1992年一样多。 E.1993年购物中心停车场巡逻的安全警卫的数目在夜间加倍了。

08. 如果杨教授今天来学校演讲,那么全校的人不会都外出。只有校长今天去参加教委的招生工作会议,才能知道今年招生工作的指导思想。只要知道今年招生工作的指导思想,就能很好地完成今年的招生工作。事实上,杨教授今天来学校演讲了。


如果上述断定都是真的,则下列各项都可能是真的,除了: A.校长不知道今年招生工作的指导思想。

B.校长今天去参加了教委的招生工作会议,但不知道今年招生工作的指导思想。 C.知道了今年招生工作的指导思想,但是今年的招生工作仍不能保证很好地完成。 D.全校的人今天都没有外出。 E.校长没有外出。

09. 要想精神健康,人们必须自尊。人们要保持自尊仅仅通过不断赢得他们尊重的其他人的尊重,他们要赢得这种尊重只有通过道德地对待这些人。 下列哪一个结论可以从上文得到?

A. 精神健康的人将被别人道德地对待。

B. 精神健康的人将道德地对待他们尊重的人。

C. 精神健康的人为了被别人道德地对待必须有自尊。

D. 人们能够期望被别人道德地对待只有看他们是否尊重别人。 E. 自尊的人很少道德地对待那些他们尊重的人。

10. 城市居民住房条件较十年前有了很大改善。因为平均每户拥有住房面积(建筑面积) 加了15%

上述论证需要下列哪项假设?

A 房屋质量较十年前有了提高。 B 家庭平均人口数没有增加。

C 随着房地产市场的发展,可居住的房屋总面积增加了。 D 北京、上海等一些大城市的住房困难应该得到解决。

E 商品房的价格必须下调,如果商品房的空置率居高不下,那么有再多的商品房也 不能解决住房难问题。

11. 赵、钱、孙三人一起参加了申请硕士学位的外语和综合两门考试。三个人中,只有一个在考试中发挥正常。考试前, 赵说:“如果我在考试中发挥不正常,我将不能通过外语考试。如果我在考试中发挥正常,我将能通过综合考试。 钱说:“如果我在考试中发挥不正常,我将不能通过综合考试。如果我在考试中发挥正常,我将能通过外语考试 孙说:“如果我在考试中发挥不正常,我将不能通过外语考试。如果我在考试中发挥正常,我将能通过外语考试

考试结束后,证明这三个人说的都是真话,并且,发挥正常的人是三人中惟一通过这两门科目中某门考试的人;发挥正常的人也是三人中惟一没有通过另一门考试的人。 从上述断定能推出以下哪项结论? A. 赵是发挥正常的人。 B. 钱是发挥正常的人。 C. 孙是发挥正常的人。

D. 题干中缺乏足够的条件来确定谁是发挥正常的人。 E. 题干中包含互相矛盾的信息。

12. 一度被广泛持有的“知识分子是基本道德问题的澄清者”的观念现在不再有效了。知识分子不再充当被压迫组织的代言者。他们把辩论留给了政治家,而不是运用他们的洞察力来分析解决这些组织的问题。

上面文章的逻辑结构依赖作者紧密地把基本道德问题的澄清与下列哪一个联系起来? A. 智力的运用


B. 代表被压迫组织辩护 C. 洞察力和分析

D. 对当代实践问题辩论 E. 政治家的角色

13. 宿舍楼的高度为二层到六层不等,如果宿舍在二楼以上,它就有安全通道。如果上面陈述属实,则下面哪项也是正确的? (A)位于第二层的宿舍没有安全通道。 (B)位于第三层的宿舍没有安全通道。

(C)只有位于第二层以上的宿舍有安全通道。 (D)位于第四层的宿舍有安全通道。 (E)有些两层楼的宿舍楼没有安全通道。

14. 人口,人口,其实人不光有张口,首先有双手,即人不光是物质资料的消费者,而且是物质资料的生产者,更重要的是人作为物质资料的生产者所创造的物质资料,要比人作为物质资料消费者消费的物质资料多得多,因此,人是最宝贵的,人多是优势。 以下哪项最有力地减弱了上述论证?

A.人口的膨胀不利于维持人和自然之间的生态平衡,例如,将导致耕地减少,环境污染,等等。

B.膨胀的人口将增加社会矛盾,不利于社会的稳定。

C.人作为物质资料的消费者是无条件的,在任何时候人都要吃饭穿衣,而人作为物质资料的生产者是有条件的,因为他要受到有限的物质资源的制约。

D.地球只有这么大,许多地区如沙漠、戈壁、高源尚不具备开发的条件。 E.现代科学技术还不具备把地球人类社会向外层空间扩展的能力。

15. 由于受自然灾害的影响,湖南今年的橘子产量锐减,这一地区的橘子价格比往年同期上涨了一倍。这同时大大提高了橘子罐头的生产成本,估计今年橘子罐头的价格将会大幅度提高。

以下哪项,如果是真的,最能削弱上述观点? A. 今年其他水果的产量所受的影响不太大。

B. 今年的旱灾、虫害并不如当初估计的那么严重。 C. 近几年,橘子价格持续走低,现在应该反弹了。

D. 近年来,通过技术革新,橘子罐头的生产工艺有了很大改进。

E. 除了湖南之外,其他不少没受灾的省份也盛产橘子,生产橘子罐头。

16. 电脑更新换代的速度之快,几乎是20世纪末的一道风景,通常是一种款式的电脑刚刚上市、为人们所知晓,另一种配置更高、价格更低的产品又问世了。据《中国计算机通讯》杂志编辑分析,正在酝酿上市的三鑫新一代电脑比目前市场上的同类产品配置更高、价格更低,它会很快在市场竞争中脱颖而出。

以下哪项如果为真,将最不能削弱上述论证?

A.三鑫电脑尚未建立起自己的销售网络,目前甚至尚无代销商与之接洽。 B.据报道,市场上很快又将有另外几种高配置、低价位的电脑新品牌问世。 C.三鑫新一代电脑还有进一步降价的余地。

D.目前正值通货紧缩,三鑫电脑所定位的个人客户对电脑的需求不旺。 E.许多客户对三鑫电脑的售后服务还不太满意。

17. 德国轿车80年代在A国十分畅销。进入90年代以来,A国货币对德国马克汇率不断下调,这样会使进入A国市场的德国轿车价格上升。然而,德国轿车在A国的销量却并未因此下降。


以下哪项为真,最能解释德国轿车销量居高不下? A. A国的国货宣传不够。

B. 德国轿车生产厂家把成本的增幅控制在一定范围内。 C. 德国轿车生产厂家成本增幅比A国国产轿车增幅小。

D. 尽管目前德国轿车销量未下降,未来两三年内就会显现下降趋势。 E. A国消费者更加注重德国轿车的优秀质量,弥补了价格上的不利面。 18. 在一些大城市,尽管一些重要的路口已经有红绿灯,并且装了电子眼系统,但仍然安 交通警察值勤。

下列哪些项都有助于解释这种看似矛盾的现象?除了: A 有些突发事件需要警察处理。

B 有些警察在同一个路口已经工作了许多年,有感情,不愿离开岗位。 C 电子眼经常遭到人为破坏。

D 许多司机和行人还没有养成自觉遵守交通秩序的好习惯。 E 电子眼的可靠性还有待进一步观察。

19. 某学校准备选四位同学给予表彰,从学习成绩来看,小周、小吴两位同学比小郑、小王两位同学好。

据此,再加上以下哪项可推出“小赵比小王的学习成绩好”的结论? A. 小周的学习成绩比小吴差。 B. 小吴的学习成绩比小周好。 C. 小赵的学习成绩比小周好。 D. 小赵的学习成绩比小吴差。 E. 小赵的学习成绩比小郑好。

20. 全国最大的零售商声称,在春节前后的6个月里,销售了大量货物,但利润却比平常低。这种情况是很少见的,因为当销售量上升时,通常利润也会上升。 以下哪项,如果为真,能最好地解释上述情况? A.利润很高的化妆品的销售额没有什么增加。

B.女装和童装的销售额增加了20%,而男士服装的销售额则下降了许多。 C.去年,全国独立的零售商增加了大约20%

D.为了吸引更多的消费者,全国最大的零售商大大增加了广告成本。

E.为了利用春节期间消费者扩大消费的时机,许多零售商提高了商品价格。

21. 如果一个家庭的人均收入低于当地15%的家庭的水平,则可称为该地区的贫困户。从统计局的调查得知,近20年来,西部山区经济发展得很快,其贫困户的数量一直在稳步下降。

假定以上情况为真,则以下哪项也必定为真?

A. 20年来,西部山区的贫富分化现象越来越严重。 B. 20年来,西部山区的非贫困户的数量不断下降。

C. 20年来,西部山区富裕户的人均收入一直在稳步增长。

D. 20年来,在西部山区,每一个贫困户的家庭收入都低于整个地区平均家庭收入。 E. 20年来,在西部山区,每一个富裕户的家庭人均收入都超过整个地区的人均收入。 22. 人类学家发现早在旧石器时代,人类就有了死后复生的信念,在发掘的那个时代的古墓中,死者的身边有衣服、饰物和武器的陪葬,这是最早的关于人类具有死后复生信念的证据。 以下哪项,是上述议论所预设的?

A.死者身边的陪葬物是属于死者的。 B.死后复生是大多数宗教的核心信念。


C.宗教信仰是大多数古代文明社会的特征。

D.只有相信死后复生才会在死者身边安置陪葬物。 E.陪葬物的置放是表示对死者的崇敬。

23. 在美国,企业高级主管和董事们买卖他们手里的本公司股票是很普通的。一般来说,某种股票内部卖与买的比率低于21时,股票价格会迅速上升。近些天来,虽然X公司的股票一直下跌,但公司的高级主管和董事们购进的股票却9倍于卖出的股票。 以上事实最能支持以下哪种预测?

A. X股票内部买卖的不平衡今后还将增长。 B. X股票的内部购买会马上停止。 C. X股票的价格会马上上涨。

D. X股票的价格会继续下降,但速度放慢。

E. X股票的大部分仍将由其高级主管和董事们持有。

24. 工业机器人的广泛发展导致失业,因此,智能机器是危险的。 上述论述最像下列哪一个?

A. 墨索里尼是一个邪恶的人,因此他成为一名法西斯。 B. 吃熏肉是有害于健康的,因此高脂肪的食物是不安全的。 C. 一些法官是不诚实的,因为他们受贿。

D. 鲸处于灭绝的危险中,因此我们不应当同允许捕鲸的因家做贸易。 E. 鸟类是危险的因为它们都传播疾病。

25. 为了阻止利益的冲突,政府将规定阻止退休科技专家在离开科研院所后的三年内接受外聘科技顾问的职务。然而一位科技专家断言,这个限制是不幸的,因为它将阻止退休科技专家三年的谋生之道。

这位科技专家的结论逻辑地依靠以下哪条假设? A. 法律不应限制退休科技专家的行为。

B. 外聘科技顾问主要由曾在科研院所担任科技专家的人来担任。 C. 许多科技专家离开科研院所后不经常担任外聘科技顾问。 D. 退休科技专家离开科研院所后只能以外聘科技顾问谋生。 E. 现在退休科技专家离开科研院所只允许做三年外聘科技顾问。 参考答案:

01.C 02.B 03.D 04.B 05.C 06.E 07.D 08.C 09.B 10.B 11.B 12.B 13.D 14.C 15.E 16.C 17.E 18.B 19.C 20.D 21.B 22.D 23.C 24.B 25.D

01. 女性占据了国家人口的大多数,并且许多医生开的镇静剂处方是为女性病人开的。然而这些药的针对效果及推荐剂量的测试仅对男性病人做过,医生用于测试该药物毒性的动物也不是雌性。

上面的话,假如正确,最支持下列哪一个作为结论? A. 一些镇静剂开给女性比开给男性更合适。

B. 女性中,没有处方开出的镇静剂的负面反应的报道。

C. 当医生对他们的诊断没有信心时,镇静剂在许多情况下被开给病人。 D. 药物对女性的毒性比对男性的毒性小。

E. 推荐的镇静剂的剂量对于女性是否理想是未知的。 02. 灰狼在一个大型国家公园中已消失了几十年。公园官员希望重新引入灰狼而又不危害那儿的任何已存在的野生生命。因为公园里有足够的供灰狼捕食的对象,并且因为灰狼避免和人近距离接触,重新引进它们可能达到官员所期望的目的而又不严重危及游览者的安全。 下列各项,假如正确,除了哪一个都支持上面论述?


A. 公园是如此之大以致灰狼不需要冒险进入人类频繁的地区。

B. 狂犬病在灰狼中非常稀少,从有记录以后,没有来自于非狂犬病的野生灰狼造成的人类受伤的案子被证实。

C. 公园附近的牧场主表达了对灰狼的关心,假如重新引进后,灰狼可能有时会捕捉他们的牲畜。

D. 在公园中灰狼对麋鹿中的营养不良者的捕捉可能会改善公园中鹿作为一个整体的健康状况。

E. 灰狼不捕捉目前居住在公园的任何濒于灭绝的种类。

03. 调查表明,最近几年来,中老年人中患高血压的病例逐年减少。但是,以此还不能得出高血压发病率逐年下降的结论。

以下哪项如果是真的,最能加强以上的结论? A. 高血压早就不是不治之症。

B. 近年来青年人中的高血压病例有所上升。

C. 防治高血压病的医疗条件近年来有很大的改善。

D. 和心血管病、肿瘤病等比较起来,近年来对高血压的防治缺乏足够的重视。 E. 上述调查的重点是在城市,农村中高血压的发病情况缺乏准确的统计。

04. 奥特珀若斯是一种减少骨质、导致易碎脆性骨头的疾病。目前对奥特珀若斯的治疗使用爱斯进或卡斯通宁药物帮助阻止更多的骨质损失,但并不增加骨质。因为氟化物据知能增加骨质,因此,把氟化物用于奥特珀若斯的病人可能帮助使他们的骨头不那么易碎。 下列哪一个,假如正确,最反对上面的论述?

A. 大多数患奥特珀若斯的病人未意识到氟化物可以增加骨质。 B. 氟化物在许多情况下被加入饮水中来加强人的牙齿。

C. 患上奥特珀若斯和其他使骨质腐坏的疾病的危险可以由锻炼和摄入足够的钙而降低。 D. 不像氟化物,爱斯进或卡斯通宁据知对许多人导致无法预期的副作用。 E. 由氟化物的使用而增生的新骨质比通常的骨骼组织更易碎和不那么有弹性。 05. 铁矿采掘业发言人提出建议:为了维持国产铁的价格,必须限制较便宜的国外铁的进口,否则,我国的铁矿采掘业将难以生存下去。 炼钢厂发言人对上述建议的反应:我国炼钢厂购买的铁70%都是国产的。如果铁价不是按国际价格支付,那么,由于成本的提高,国产的钢就会卖不出去,这样对国产铁的需求就会下跌。

以下哪项是对炼钢厂发言人的论证的最恰当评价?

A. 它说明即使上述建议被拒绝,铁矿采掘业也会繁荣。 B. 该论证无的放矢,和铁矿采掘业发言人的建议无关。

C. 它说明铁矿采掘业发言人的建议如果实施的话将会对自身产生负面影响。

D. 它没有给出理由说明为什么上述建议的实施并不能减轻铁矿采掘业经营的难度。

E. 该论证是循环论证,它预先假设了为了评论铁矿采掘业发言人的建议而需要证明的东西。 06. 用地面雷达所拍下的流星的最近距离为220万英里,即流星托特被拍下的最近距离。近的流星照片是盖斯,在仅仅1万英里远的距离被拍摄到。 下列哪一个可以从上面语句中正确推导出来? A. 托特比盖斯更可能与地球相撞。 B. 托特,不像盖斯,仅仅最近被发现。

C. 托特仅仅可以通过使用地面雷达拍摄下来。

D. 基于地面的雷达不能拍摄距地球220万英里以外的物体。 E. 盖斯的照片不是由基于地面的雷达拍摄下来的。


07. 目前艾滋病毒感染者的人数在欧美国家趋于稳定或略有下降,但在非洲国家却持续快速增长;估计2000年末全球的艾滋病毒感染者为4000万~1. 1亿人,其中60%将集中在非洲国家;因此,在21世纪初,非洲国家与欧美国家相比,将有更多的人死于艾滋病。 以下哪项如果为真,最能削弱上述论证?

A. 欧美国家艾滋病的传播方式以同性恋和吸毒为主,非洲国家则以异性途径为主。没有证据说明,异性途径比同性恋或吸毒更利于艾滋病的传播。

B. 中医在预防和治疗艾滋病方面的进展越来越受到关注,估计21世纪初会有重大突破。 C. 21世纪初,世界卫生组织将对非洲国家的医疗保健、包括艾滋病的防治投入更多的关注和经济支持。

D. 21世纪初,艾滋病感染者的集中地将从欧美转移到亚洲和非洲。

E. 欧美国家艾滋病感染者从感染到发病的平均时间要大大短于非洲国家,从发病到死亡的时间只有非洲国家的1/2

08. 现代人类是生物中唯一经历青春期的种类,青春期是一个在完全成熟前的身体加速增长的时期,远古人类的种类,现在都已经灭绝并且仅仅通过化石记录得知,是否经历青春期不能知道,因为„„

下列哪一个最合逻辑地完成上面的论述?

A. 根据种类不同,指示青春期的最小的身体生长加速可能不同。 B. 化石记录,尽管在稳定增加,总是不完全。

C. 探测青春期的突然生长需要对在不同年龄的同一个人的测量。 D. 已灭绝的人类的完整骨骼是极其稀少的。

E. 人类可能是第一个获益于由青春期导致的生存优势的种类。

09. 追溯到上一次冰河发生并且最终由极地区散布时期的珊瑚帕玛被发现在比现在帕玛生存地区更深的海洋深处。因此,尽管化石化的帕玛与目前生存的帕玛相比,外表上没有区别,但就其能够居住在深水中可得知它们一定是有所不同的。 上面的论述所基于的假设是:

A. 没有当冰河还未从极地地区散布时期的珊瑚帕玛被发现。

B. 上一次冰河散布时期的地质上的变化不会导致帕玛化石所存在的地方大规模下沉。 C. 生活在浅水中的帕玛和那些被发现在深水中的帕玛的化石大部分在相同的地理区域。 D. 帕玛的化石被发现在追溯冰河从极地地区散布的每个时期。

E. 帕玛在海洋温度低的海域比海洋温度高的海域可以居住在更深的地区。

10. 宏远钢铁厂厂领导经过调查发现,和同行业其他企业相比,该厂产品的成本要高得多,因而在市场上只能以较同类产品偏高的价格出售,导致市场竞争中节节败退。通过慎重研究,厂领导决定采取减员增效措施,30%的工人下岗,全面降低全厂员工工资,力争使成本降下来。

以下哪项,如果为真,最可能使厂领导的决定收效甚微? A. 宏远钢铁厂的设备老化,亟待更新。

B. 宏远钢铁厂的产品质量与其他企业差不多。

C. 宏远钢铁厂员工工资总额只占产品成本的一小部分。 D. 宏远钢铁厂的销售费用高于其他企业。

E. 宏远钢铁厂交货速度比较慢,常常被罚交违约金。

11. 环境保护主义者相信将他们的保护环境的努力集中在类似鸟类这样易于被调查的种类上,他们就可能保存该地区典型的总体的多样种类。这个信念基于一个观点,一个某一种类丰富的地理地区也会在该地区气侯特征下具有其他丰富种类。

下列哪一个在一个温带气侯区,以长宽10公里划分的广泛分散带的发现最挑战环境保护主


义者的假设?

A. 这些地区很少有来自于人类入侵和人类活动的污染所带来的破坏。

B. 在诸如鸟类的某一群丰富的地区,同样有这一种群所捕食的诸如昆虫,或者它所需要的植物,土地和水资源的丰盛。

C. 一个这样的10公里地带的面积通常足够来包含这一地区具有代表性的生物体的样本。 D. 在富有蝴蝶种类的地带与富有鸟类的地带很少重叠。

E. 在总体多样性最大的地区,高度集中的一种稀有种类的个体被发现。

12. 某大学某寝室中住着若干个学生。其中,一个是沈阳人,两个是北方人,一个是浙江人,两个在经管系,三个是进修生。因此,该寝室中恰好有8人。

以下各项关于该寝室的断定,如果是真的,都有可能加强上述论证,除了 A. 题干中的介绍涉及了寝室中所有的人。 B. 沈阳学生在财金系。 C. 浙江学生在经管系。 D. 进修生都是南方人。

E. 该校经管系不招收进修生。

13. 古希腊剧作家尤里披蒂在其晚期的作品中不再像早期作品中那样严格地遵守已确立的诗体构成的常规。最近发现的一部尤里披蒂的剧作中的文字与尤里披蒂早期作品一样严格地遵守那些常规,那么这部作品一定创作于尤里披蒂的早期。 下列哪一个是上文所做的假设?

A. 所有尤里披蒂的剧作都被写成诗体。

B. 尤里披蒂在其创作生涯晚期未写任何模仿其早期风格的作品。

C. 随着其创作的发展,尤里披蒂逐渐意识不到已确立的诗体构成常规。

D. 在其晚期生涯,尤里披蒂是其时代唯一的有意打破已确立的诗体构成常规的剧作家。 E. 古代的剧作家在其创作晚期比早期更倾向于不再愿意打破某种常规。

14. 在美国,新制造的国产汽车的平均油效尽管仍然比新制造的进口汽车的平均油效差,但1983年至1988年间显著提高了。新的国产汽车的平均油效从那以后没有提高,但是新的国产汽车和新的进口汽车平均油效之间的差别已经稳定下降了。 假如上面的语句是正确的,基于此下列哪一个一定也正确?

A. 1988年后制造的国产汽车的平均油效比1988年制造的进口汽车的平均油效高。 B. 新制造的国产汽车的平均油效从1988年后稳定下降了。 C. 新制造的进口汽车的平均油效从1988年后稳定下降了。 D. 新制造的进口汽车的平均油效在1983年稳定提高了。

E. 1983年制造的进口汽车的平均油效比1988年制造的进口汽车的平均油效高。 15. 桌子上有4个杯子,每个杯子上写着一句话:第一个杯子:“所有的杯子中都有啤酒”第二个杯子:“本杯中有可乐”;第三个杯子:“本杯中没有咖啡”;第四个杯子:“有些杯子中没有啤酒”

如果其中只有一句真话,那么以下哪项为真? A. 所有的杯子中都有啤酒。 B. 所有的杯子中都没有啤酒。 C. 所有的杯子中都没有可乐。 D. 第二个杯子里有可乐。 E. 第三个杯子中有咖啡。

16. 长久以来被认为,高水平的睾丸激素荷尔蒙是男性心脏病发作的主要原因。然而,这个观点不可能正确,因为有心脏病的男性一般比没有心脏病的男性有显著低水平的睾丸激素。


上面的论述基于下列哪一个假设?

A. 从未患过心脏病的许多男性通常有低水平的睾丸激素。 B. 患心脏疾病不会显著降低男性中睾丸激素的水平。

C. 除了睾丸激素以外的荷尔蒙的水平显著影响一个人患心脏病的可能性。 D. 男性中心脏病和降低的睾丸激素的水平是一个相同原因的结果。 E. 高水平的睾丸激素不会导致除心脏病外的其他严重疾病。

17. 参与带有呼吸器潜水的人平均比不参与这项运动的人更健康。因此,带有呼吸器潜水有助于提高人的健康。

下列哪一个批评了上面论述?

A. 预先假设进行呼吸器潜水的每个人这样做只是为了健康原因。

B. 导致一个更远并且不实际的结论?不参与呼吸器潜水,没有人能够获得好的健康。 C. 没有指出每年有少量的人在呼吸器潜水事故中严重受伤。

D. 处理提高某人健康的前提条件好像其本身是能够保证健康的因素。 E. 忽视了人们通常不参加潜水器潜水的可能性是他们没有好的健康。

18. 据调查,90年代以来,中学生中出现失眠症状者的比例比80年代高出2倍,有长期从事学生工作经验的人说,这是近年来“升学教育”对学生造成了过大压力的结果。 下列哪项为真,则对上述结论提出质疑? A. 这几年中学生升学比例比较稳定。 B. 同学们反映中学学习生活特别紧张。 C. 为了升学而进行的教育过于死板。

D. 人们以前只是没有太注意这种问题而已。

E. 这几年其他社会群体中出现失眠症状者的比例也明显上升。

19. 美国和前苏联的每英亩粮食产量进行的一次为期10年的对比分析结果表明,当仅以种植面积比较时,前苏联的产量是美国的68%。但当对农业总面积(包括种植面积和休耕面积)进行比较时,前苏联的产量是美国的114%

根据以上信息,关于美国与前苏联在这10年期间的农业情况,下面哪个能最可靠地推断出?

(A)美国农业总面积中休耕地的比例要大于前苏联。 (B)美国休耕地面积多于耕地面积。

(C)前苏联闲置的可用农业面积要比美国少。 (D)前苏联的耕种面积多于休耕地面积。 (E)前苏联出产的粮食要比美国多。

20. 近年来,百舌鸟的数目剧烈下降了。百舌鸟是一种居住在诸如农场和牧场之类的平地上的捕食类鸟类。一些鸟类学家猜测说这次下降可能归因于引入新的、更有效的用以控制百舌鸟所捕食的昆虫的杀虫剂。

对下列哪一个问题的回答与评价鸟类学家的假说不相关? A. 在新的杀虫剂使用前,百舌鸟的数目下降了吗?

B. 在那些新的杀虫剂没有被使用的栖息地,百舌鸟的数目已经显著下降了吗? C. 新的杀虫剂比以前使用的杀虫剂更显著地使百舌鸟所捕食的昆虫数目下降了吗?

D. 吃了新的杀虫剂的百舌鸟所捕食的昆虫比吃了以前使用的杀虫剂的昆虫对百舌鸟更有毒?

E. 大多数人认为新的杀虫剂比老的刹虫剂对环境的害处少吗?

21. 张教授不但学识渊博,而且非常有管理能力。他当上历史系系主任以后,报考历史系的学生的人数有了明显增加,这在当前历史等基础学科受冷落的情况下是非常不易的。


上述推理假设了以下哪项前提?

(1)报考历史系学生人数的增加要归结于张教授的努力。

(2)张教授在历史界享有很高的声誉,在社会上也有很高的知名度。 (3)考生的人数是衡量系主任管理能力的一个重要尺度。 A. 只有(1) C. 只有(3)

B. 只有(1)(2) D. 只有(1)(3) E. (1)(2)(3)

22. 有经验的飞行员在学习驾驶新研制的超轻型飞机时遇到的问题经常比新手还多。因为他们已经习惯了重型飞机,所以在驾驶超轻型飞机时,似乎对风没有引起应有的注意。 文中暗示上面提到的重型飞机: (A)比超轻型飞机更难着陆。

(B)不像超轻型飞机那样在飞行员中受欢迎。 (C)不如超轻型飞机安全。 (D)比超轻型飞机节省燃料。 (E)在风中比轻型飞机更易驾驶。

23. 某种被发现在污水中的极其有害的细菌难以直接探测到。在海水中对一种不易探测并且毒性较小类型的伊列细菌的测试,可能是一个可靠地决定这些更有毒的细菌是否存在的方法,因为除非海水遭到含有有毒细菌的污水污染,海水中才会含有伊列细菌。 下列哪一个,假如正确,最反对上面的论述?

A. 有许多种不同种类的伊列细菌,并且只有这些种类中的一些有毒。

B. 一些被发现在污水中的细菌既不是导致疾病的,也不是难以直接探测的。 C. 一些被发现在污水中与伊列共存的细菌对人无害,除非这些细菌被大量消化。 D. 伊列比污水中更有害的细菌死得更快,并且那时不再容易被探测到。 E. 一些被发现在污水中与伊列共存的细菌复制的速度比伊列慢。

24. 黄总工程师不仅精通业务,而且在企业经营管理方面很有一套。自1997年他兼任北山钢铁厂厂长以来,该厂当年就扭亏为盈,且上缴的利润连年上升,去年更是突破1亿元大关。这在目前国有企业普遍不景气的情况下,确实难能可贵。 以下各项都是上述议论中所暗含的内容,除了 A. 北山钢铁厂是一家国有企业。

B. 企业盈亏、利润上缴额等是衡量厂长经营管理能力的一个重要尺度。 C. 黄厂长兼总工程师的经营管理能力是使北山钢铁厂扭亏为盈的重要原因。

D. 黄厂长兼总工程师之所以在企业经营管理方面能取得成功,主要是因为他精通业务。 E. 北山钢铁厂能够扭亏为盈,且上缴的利润连年上升,去年更是突破1亿元大关,黄厂长兼总工程师应记头功。

25.20年来,在易快配公司所有申请为技师的申请者被要求演示他们可以操作和修理易快配制造业的中心机器,然而现在,那种特殊的机器过时了,并且非常不同的机器替代入中心部分。因此,旧的要求不再是一个有效的评价申请者是否具有在易快配工作的技师所必须掌握的技巧的方法。

下列哪一个,假如正确,最支持上面的论述?

A. 现在过时的机器在它过时之前被许多制造公司使用。

B. 在已经在易快配公司成为技师的人中,那些最熟练地操纵新机器的人是那些操纵旧机器最不熟练的人。


C. 大多数今天申请技师工作的人有比20年前申请技师工作的人有更广泛的技术。

D. 需要用于操纵和修理过时机器的技巧用于操纵和维持许多其他类型的在易快配公司正在使用 的并且没有过时的机器时有用。

E. 易快配公司在制造中使用的许多机器在未来20年非常可能过时。 参考答案:

01.E 02.C 03.B 04.E 05.C 06.E 07.E 08.C 09.B 10.C 11.D 12.C 13.B 14.C 15.E 16.B 17.E 18.E 19.A 20.E 21.D 22.E 23.D 24.D 25.B

1.去年,美国费城由妇女控告的**案率增加了20%。具有讽刺意味的是,这个数字是由女权运动组织在年度报告中以赞许的口气公布的。

以下哪项如果是真的,能逻辑地解释上述女权运动组织看来不合情理的赞许态度?

A.市政府鼓励受害妇女控告**的新法案的实施,极大地减少了受害妇女不敢控告的情况。

B.近三年来,这个城市**案在刑事案中之比例逐年上升。

C.女权组织的领导人一直把预防**案的发生作为优先考虑的问题。

D.这个城市受害妇女控告的**案发生率最高的地区集中在东部的三个邻近街区。 E.这个城市对**犯的法律惩治越来越严厉。

2.近七年来,家庭收入用于娱乐消费的比例一直稳定在10%左右。这个比例不会因为新的娱乐形式的出现而扩大。因此,随着电视录像这种娱乐形式的兴旺,电影院的票房收入会随之减少。

以下哪项如果是真的,最能动摇上述论证? A.录像带的祖金要比电影票便宜。

B.电影制片商依靠出售电影版权从录像业分得一部分利润。 C.录像业虽逐年兴旺,但远不能完全取代电影。

D.由于录像业的兴起,人们用于电影和电视录像以外的娱乐消费明显减少。 E.某些不适合于电影院放映的影片适合于以录像形式放映。 34基于以下题干:

政府用于支持纯理论研究的投入经常被认为是浪费,似乎只有直接的技术应用可以证明科学的价值。但是,如果没有纯理论研究,应用技术终将成为日渐枯竭的无源之水。今天的纯理论研究可能看来没什么用,但是谁也说不准某一天它会产生什么样的应用奇迹。这正如人们完全可以问:“新生的婴儿有什么用?” 3.上述议论基于以下哪项假设?

I 理论发现和它的实际应用之间存在时 间上的差距。 II 纯理论研究比技术应用更费时 间和金钱。

III理论转化为实际应用的时 间越长,这种理论就越有价值。 A.仅仅I B.仅仅II C.仅仅III

D.仅仅IIII EIIIIII

4.上述论证把“新生婴儿”比作以下哪项? A.基于纯理论的一种新技术突破。 B.政府对科学研究的投入。 C.公众对纯理论研究的支持。

D.新技术成果使社会得到的受益。


E.一时看不到直接应用价值的纯理论发现。

5.农场发言人:毗邻我农场的炼铅厂引起的空气污染造成了本农场农作物的大幅度减产。

炼铅厂发言人:“责任不在本厂。我们的研究表明,农场减产应该归咎于有害昆虫和真菌的蔓延。

以下哪项如果是真的,最有力削弱炼铅厂发言人的结论? A.炼铅厂的研究并没有测定该厂释放的有害气体的数量。 B.农场近年来的耕作方式没什么变化。

C.炼铅厂的空气污染破坏了周边的生态平衡,使得有害昆虫和真菌大量滋生。 D.炼铅厂释放的有害气体是无色无嗅的。

E.所说的有害昆虫和真菌在周边地区近百年来都偶有发现。 67基于以下题干:

在南美,因为气候恶劣,同时又有许多原先种植胡椒的农民改种价值更高的可可,所以 过去三年中世界胡椒的产量一直低于销售量,胡椒处于相对短缺状态,价格也飞涨直至与可可相当。

6.由上文可推知:

A.胡椒只有大量种植才有利可图。

B.过去三年中世界胡椒消费量高得非同寻常。 C.气候一旦回转正常,世界胡椒产量又会回升。 D.过去三年中世界的胡椒剩余储备减少了。

E.过去三年种植胡椒的农民获利之大是前所未有的。

7.有人认为,由于胡椒价格上涨,那些三年前改种可可的人并不见得比不改种的好。但是,这个结论是不确定的,因为由上文可以推知:

A.那些改种可可的农民并不能预见到底胡椒价格会涨到多高。 B.从种胡椒转种可可的初始成本巨大。

C.如果他们不转种可可,胡椒的供给不可能如此之少而导致价格上涨。 D.可可和胡椒一样易受气候条件的影响。

E.当越来越多的人种植可可时,可可的供给就会上升,价格就会下降。

8.当前的大学教育在传授基本技能上是失败的。有人对若干大公司人事部门负责人行了一次调查,发现很大一部分新上岗的工作人员中都没有很好掌握基本的写作、数量和逻辑技能。 上述论证是以下列哪项为前提的?

A.现在的大学里没有基本技能方面的课程。 B.新上岗人员中极少有大学生。

C.写作、数量、逻辑方面的基本技能对胜任工作很重要。

D.大公司的新上岗人员基本上代表了当前大学毕业生的水平。 E.过去的大学生比现在的大学生接收了更多的基本技能教育

9据最近统计,在需要同等学历的十个不同职业中,教师的平均工资五年前排列第九,而目前上升到第六;另外,目前教师的平均工资是其他上述职业的平均工资的96%,而五年前只占55%。因此,教师工资偏低的状况有了很大的改善,教师的相对生活水平有了很大的提高。

上述论证基于以下哪项假设?

Ⅰ近五年来的通货膨胀率基本保持稳定。

Ⅱ和其他职业一样,教师中的最高工资与最低工资的差别并不悬殊。 Ⅲ学历是确定工资标准的主要依据。


Ⅳ工资是实际收入的主要部分。 A.仅Ⅰ和Ⅱ。 B.仅Ⅲ和Ⅳ。 C.仅Ⅲ。 D.仅Ⅳ。 E.仅Ⅲ和Ⅳ。

10有一位研究者称,数学方面女性和男性一样有才能。但是她们的才能之所以未被充分发挥出来,是因为社会期望她们在其他更多的方面表现出自己的能力。 以下哪项是该研究者的一个假设?

A数学能力比其他方面的能力更重要。 B数学能力不及其他方面能力重要。 C.妇女在总体上比男性更有才能。 D.妇女在总体上不比男性更有才能。 E.妇女倾向于趋同社会对她们的期望。 11.某国的一份公文如下:“我国应立法限制使用石油制品生产塑料。我国对燃料的需要远远甚于对塑料的需要;而且,我国对进口石油越来越依赖,一旦战争爆发,石油进口被阻断,我国将陷入危机。如果减少制造塑料的石油制品用量,我们就能在独立自主和国家安全上迈进显著的一步。

以下哪项如果为真,最能反驳上述论证?

A.事实上,该国用于制造塑料的石油消耗只占全部石油消耗非常小的比例。 B.某些塑料制品,如飞机、汽车上的部件,对国防有很重要的作用。 C.战争时期,敌国会极力攻占石油供给国地区。 D.新研制的塑料制造方法能有效减少石油用量。

E.利用核能代替石油的研究受到了国际核条约的约束而减慢。

12.重振女排的威风,关键是发扬拼搏精神,如果没有拼搏精神,战术技术的训练发挥再讲究,也不可能在超级强手面前取得突破性的成功。 下列诸项都表达了上述议论的原意,除了:

A.只有发扬拼搏精神,才能取得突破性的成功。 B.除非发扬拼搏精神,否则不能取得突破性的成功。 C.如果取得了突破性的成功,说明一定发扬了拼搏精神。 D.不能设想女排取得了突破性成功但却没有发扬拼搏精神。

E.只要发扬拼搏精神,即使战术技术的发挥较差,也能取得突破性的成功。

131960年~1970年间,非洲国家津巴布韦境内的狩猎者猎捕了6500多头大象以获取象牙,这一时期津国大象总数从35000头下降到30000头以下。1970年津国采取保护大象措施,1970年~1980年期间逮捕并驱逐了800多名狩猎人。但是,到1980年津国大象总数还是下降到21000头。

下列哪项如果为真,最有助于解释上述表面上的矛盾现象? A1960年~1980年问逮捕的狩猎者并未被判处长期徒刑。

B.公众反对滥捕大象呼声高涨,1970年~1980年间象牙的需求下降。 C.津国的一个邻国1970年~1980年间大象数量略有回升。 D1970年以前,津国反对捕杀大象的法律没有得到执行。

E1970年~1980年间,津国大量砍伐了大象赖以生存的森林。 1415基于以下题干:

所有并且只有带轮子在高速公路上运行的交通工具才是交通污染源。


自行车不是交通污染源。

我的汽车在高速公路上行驶的时候,天都下着雨。 现在天正下着雨。

14.如果上述断定都是真的,下面哪项断定也一定是真的? A.自行车并不在高速公路上行驶。

B.只有当天下雨的时候,自行车才在高速公路上行驶。 C.如果我的汽车不在制造污染,则天没有下雨。 D.我现在正在高速公路上驾车。 E.我的汽车不是污染源。

15.从上述断定加上以下哪项条件可逻辑地得出结论“我的汽车没有在制造污染”? A.Ⅱ改为“自行车是交通污染源” B.Ⅱ改为“我的汽车是交通污染源”

C.Ⅲ改为“如果自行车是交通污染源的话,我是一定会在高速公路上驾我的汽车的” D.Ⅳ改为“雨水被污染了” E.Ⅳ改为“天现在没下雨” 参考答案: 1A 2 D 3 A 4 E 5 C 6 D 7 C 8 D 9E 10E 11A 12E 13E 14A 15 E 01. 去年电话机行业的销售额大幅度上升。为了利用这一形势,某电话公司准备扩大本公司型号的电话机生产量,同时继续从事已经家喻户晓的广告宣传工作。 以下哪项为真,则最有可能使得该公司采用以上计划时不能增加销售额? A.虽然去年生产的产品全部售出,但该公司的市场占有率是下降的。 B.该公司准备发运给零售商的电话机的库存数去年有轻微下降。 C.广告使得该公司生产的电话机的品牌广为人知,但很少有人知道这个牌子的电话机是该公司生产的。

D.该公司的电话机是去年销售额增加的三种品牌之一。 E.尽管零售价格有所上升,该公司的销售额去年是下降的。

02. 有一种高效降血脂药物目前只能从一种叫“菠苴”树的树皮中提取。这种树在我国非常稀少,而制取一公斤药物需要剥取1000株“菠苴”树的树皮。因此,这种药物在我国的大批量持续生产,将不可避免地导致“菠苴”树种在我国的灭绝。 以下哪项如果为真,最能削弱上述结论? A.我国已制定法律严格禁止砍伐“菠苴”

B.“菠苴”可以通过剪枝插栽的方式由人工栽培。 C.在我国,“菠苴”大多生长在人迹稀少,交通不便的边远地区。

D.由“菠苴”树皮制取的药物须由中央卫生部门的批准才能交付医生使用。

E.非洲某国有丰富的“菠苴”树资源,我国每年可以从该国购得10000株“菠苴”树皮作为药物原料。

03. 历史证明,民族兴旺、国家发展的关键因素是国民素质的提高。因此,实现我国宏伟发展目标的关键措施是进一步增加教育投入。 上述断定基于以下哪项假设?

(1)教育事业的发展是提高国民素质的主要条件。 (2)增加教育投入是发展教育事业的重要条件。 (3)我国目前的教育投入不能适应发展教育的需要。 A.仅(1) B.仅(2)


C.仅(3)

D.仅(1)(3) E(1)(2)(3)

04. 几年前的一个司法法令的目标是鼓励电话业的竞争,竞争可能导致消费者的节省。现在白天打的长途电话比法令公布前便宜了,但是居民的长途电话的平均花费已经增加了百分之二十五。

下列哪一个,假如正确,最好地解释了居民长途电话费更高了? A.企业打的长途电话比居民打的多。

B.电话公司把他们的服务延伸到计算数据和数据处理领域。 C.大多数居民打电话的时间是夜晚,夜晚的电话费已经增加了。 D.竞争的加剧已经导致电话公司为新技术的发展扩大预算。

E.电话公司在电话费率变化付之实践之前必须得到管理机构的批准。

05. 从一组调查数据中,我们发现这样一个有趣的现象。从1980年至今,我国农民的人均收入和化妆品的销售额几乎是在同步增长。 下述哪项最能合理地解释这个数据结果呢?

A.改革开放以前,农民的温饱问题都解决不了,哪里还有闲钱去买化妆品呢。现在农民的日子好过了,他们把多挣的钱都用来购买化妆品。

B.虽然实行了计划生育政策,但由于我国农民的基数大,农民人口的增长率仍然很快,人口多,用的化妆品自然就多。

C.改革开放以来,我国人民的生活水平有了很大提高,所以农民的收入和化妆品的销售额都有了较大增加。

D.这一时期里,化妆品的广告越来越多,对化妆品这个行业的发展起到了推动作用。 E.化妆品的价格上涨幅度很大,导致化妆品的销售额增加。

06. 需求规律告诉我们:如果某种产品价格上涨,那么对它的需求就会减少,价格下跌,需求就会增加。但是,1988年,当大豆色拉油的价格上涨时,对它的需求却大大增加了。 如果上述情况属实,以下哪项能最好地解释上述理论和实际的差别? A.因为大豆色拉油味道不错,许多人都喜欢它。 B.大豆色拉油便于储存,比较方便。

C.大豆色拉油的制作工艺比较复杂,一般家庭不会做,只得去买。

D.因为制作大豆色拉油的大豆等的零售价格上涨,所以大豆色拉油的价格自然要上涨。 E.因为纯正花生油等其他食用油的零售价比大豆色拉油上涨得更快,所以相对于其他食用油,大豆色拉油的价格还是比较便宜的。

07. 在一次产品评比会上,某彩电生产企业生产的21厘米彩电获得了金奖。该企业的总经理觉得很受鼓舞,当即决定:加大投入,把21厘米彩电的产量扩大一倍。 以下哪项最不能质疑这位总经理的决策?

A.生产21厘米彩电所需的彩色玻壳已经很难买到了。 B21厘米彩电已经面临被淘汰的局面。 C21厘米彩电已经在市场上供大于求了。

D.此次评比会不是权威部门举办的,很多彩电生产企业没有参加。

E.该企业生产的21厘米彩电经过了几次价格大战,出厂价格已经低于成本价。

08. 地理和历史的证据显示美国东部的土也 震与加州相比震中强度上相同,但影响的面积更大。相同震级的土也 震在美国东部打击的区域是加州的100倍。

下列哪一个,假如正确,最帮助解释上面所描述的被影响区域的面积的不同? A.美国东部的建筑物比加州的建筑物更老,因此在大土也 震中易受更大的破坏。


B.加州的地壳,与东部相比较,当土也 震波从震中向外传递时,有更丰富的吸收土也 震波的断层。

C.发生在美国的一些最强并且影响范围最大的土也 震是以美国东部的一些地方为中心发生的。

D.因为发生在美国东部的主要土也 震没有加州那么频繁有规律,难以预测什么时候下一次土也 震可能发生。

E.加州土也 震的原因比东部土也 震的原因更容易理解。

09. 去年6月下旬天气奇热,但京西大学的师生却无法利用学校游泳池消暑,因为京西大学游泳池要到暑期才开放,而暑期则开始于7月上旬。因此,今后为了避免这一问题,京西大学校方应该把游泳池开放的时间定在从6月下旬开始。 上述论证预设了哪项?

A.游泳是消暑的最好形式。 B.京西大学的师生人数太多。

C.6月下旬京西大学游泳池尚未具备开放的条件。 D.京西大学的游泳池总是要开放到暑期结束

E.去年6月下旬的炎热天气对每年同期的气候来说是很典型的。

10. 在过去10年中,登山设备有了一些改进。这些改进使这项运动对于有经验的登山者而言更安全并且更有娱乐性。然而,尽管有这些改进,登山的受伤率在过去10年还是增加了。 假如上面的话都是正确的,下列哪一个,假如正确,最好地解释了他们的明显差别? A.许多登山者,陷入一种虚假的安全感,使用新设备尝试一些他们没有能力做到的高级动作。 B.一些登山受伤的事故是由未预见的天气条件导致的。

C.登山运动,尽管是一种危险的行动,通常不会给有经验的登山者带来伤害。 D.在过去10年,登山技术和登山设备都有一些提高。 E.尽管登山受伤率已经增加了,登山死亡率没有变化。

11. 名牌进入千家万户,家庭电器化、电脑化和“洋货化”已经成为时尚。但同时,人们也发现,这是一个美丽的陷阱:流行是短暂的,掏不完的钱带给消费者的是新的遗憾。聪明的消费者逐渐醒悟:买家电还是要量力而行。

以下哪项,如果以真,最能构成对上述看法的质疑?

A.家电的流行在一定时期内代表了更为先进的技术水平。

B.购买时尚家电所费甚多,财力的消耗提醒我们注意自己的家底。

C30%购买豪华家电的居民并不具备相应经济实力,但却有紧跟消费浪潮的心理。 D.投资电脑获得知识、技能回报,带给消费者的是比遗憾更多的成功和自信。

E.家电价格不断下调,用旧家电换购新家电只需要支付很小的价格补差就可以更新换代。 12. 因为冷冻食品的过程消耗能量,许多人把他们的电冰箱保持半空,仅仅使用它们冷冻已冷冻好的食品,然而半空的电冰箱比全满的电冰箱消耗更多的能量。 下列哪一个,假如正确,最有助于解释上面所描述的明显不同?

A.电冰箱中一定量的空气保持在零度以下比相同量的冷冻食物消耗更多的能量。 B.冰箱的门开的次数越多,就需要更多的能量来保持电冰箱的正常温度。

C.当未冷冻食品被放在电冰箱中,电冰箱内部一定量的空气的平均温度暂时升高。 D.一个通常保持半空电冰箱的人应该使用小型电冰箱来削减能量花费。

E.只有当冷冻空气在电冰箱内部冷冻室中能自由循环时,一个电冰箱才能有效操作。 13. 某国的一份公文如下:“我国应立法限制使用石油制品生产塑料。我国对燃料的需要远远甚于对塑料的需要;而且,我国对进口石油越来越依赖,一旦战争爆发,石油进口被阻断,我国将陷入危机。如果减少制造塑料和石油制品用量,我们就能在独立自主和国家安全上迈


出显著的一步。

以下哪项如果为真,最能反驳上述论证?

A.事实上,该国用于制造塑料的石油消耗只占全部石油消耗非常小的比例。 B.某些塑料制品,如飞机、汽车上的部件,对国防有很重要的作用。 C.战争时期,敌国会极力攻占石油供给国地区。 D.新研制的塑料制造方法能有效减少石油用量。

E.利用核能代替石油的研究受到了国际核条约的约束而减慢。

14. 20年前,任一公司的执行官在重新选择公司总部时主要关心的是土地的成本。今天一个执行官计划重设总部时的主要关心的东西更广泛了,经常包括当地学校和住房的质量。 假如上面的信息是正确的,下列哪一个最好地解释了上面所描述的执行官关心方面的变化? A.20年前高质量的住房和学校像今天一样难以发现。

B.某些地区房地产和学校税停止增加,现允许许多人购买房屋。 C.公司执行官在做决定时总是考虑替换方法将怎样影响公司的利润。

D.一个近来人员缺乏的问题迫使公司找到尽可能多的方法来吸引新的雇员。 E.在过去的20年中,一些地区比其他地区土地的价值变化少。

15. 为了减少天然气使用中的浪费,某区政府将出台一项天然气调价措施;对每个用户,包括民用户和工业用户,分别规定月消费限额;不超过限额的,按平价收费;超过限额的,按累进高价收费。该项调价措施的论证报告估计,实施调价后,全区天然气的月消耗量至少可以节省10%

为了使上述论证报告及其所作的估计成立,以下哪项是必须假设的? (1)天然气价格偏低是造成该区天然气使用中存在浪费现象的重要原因。 (2)该区目前天然气消费量的至少10%是浪费。

(3)该区至少有10%的天然气用户浪费使用天然气。

(4)天然气价格上调的幅度足以对浪费使用天然气的用户产生经济压力。 A(1)(2)(3)(4)

B(1)(2)(3)(4)都不是必须假设的。 C.仅(1)(2)(4) D.仅(1)(2)(3) E.仅(2)(3)(4)

16. 在一次世界范围内的股市剧跌这后的余波中,T国宣称该国经历的相当严重的股市下跌是由于在下跌之前不久,该国许多行业经历了过快的非国有化过程。

以下哪一项,如果能被执行,将最有助于对T国股票市场严重下跌原因的评估? (A)计算在下跌期间T国个人交易商的平均损失。

(B)利用经济学理论预测T国下一次下跌最有可能的时间。

(C)T国下跌过程中最糟糕的那段时间抛出的股票总数同刚刚下跌前T国抛出的股票总数相比。

(D)T国下跌的严重程度同那些其他经济条件与T国相似,但没有经历最近的非国有化过程的国家下跌的严重程度相比。

(E)把这次下跌对T国货币购买力的长期影响下下跌对T国货币购买力的即时的、更严重的短期影相比。 17. 甲:尽管本地区几年来中学招生人数持续下降,但是小学招生人数却在大幅增加。因此,地区校务委员会提议建造一所新的小学。

乙:另一个方案可以是将一些中学教室临时改为小学学生教室。 下列哪一个,假如正确,最有助于支持乙的方案?


A.一些中学教室不能被改造为适合小学学生用的教室。 B.建造一个中学的成本比造一个小学的成本高。

C.虽然出生率未提高,送孩子去本地区中学的家庭数目显著增多。 D.中学气氛可能危及小学学生的安全和自信。

E.即使在该地区中学人数开始下降以前,有几个中学的教室很少被使用。

18. 甲:R岛上的大学理科毕业生比以往任何时候都更愿意接受在其他领域的永久性工作。这表明了R岛上的科研人员的报酬不高。

乙:不,并非如此。这些毕业生不在科学界工作的原因很简单,因为在R岛上没有那么多的科学工作可供这些毕业生去做。

下列哪一项,假如正确,将最反对甲的观点?

A.不在科学领域工作的理科学位的大学毕业生目前所得薪水比做科研人员的薪水低。 B.获得理科学位的大学生比以前少了。

C.在过去十年,科学界工作机会数目稳定增长。

D.相当数量的理工学位大学毕业生在校时做过低酬工作。

E.每年均有些应届理工学位大学毕业生接受非科学领域的永久工作。

19. 如果一个家庭的人均收入超过所在地区75%的家庭的水平,则称为该地区的富裕户。近10年来,闽江地区富裕户的数量一直在稳步增长。 如果以上断定为真,则以下哪项也必定为真?

A.近10年来,闽江地区的贫富分化现象越来越严重。 B.近10年来,闽江地区的非富裕户的数量不断增长。

C.闽江地区每一个富裕户的人均收入都超过整个地区的人均收入。 D.每一个富裕户的家庭收入都大于任一非富裕户的家庭收入。 E.近10年来,闽江地区富裕户的人均收入一直在稳步增长。 20. 顾问:某受欢迎的杂志每年都要公布一个以若干项标准评判的综合统计分数排名的美国大学名录。然而,这些总分通常不应被学生作为决定申请哪所大学的依据。 下列哪一个,假如正确,最有助于证明顾问建议的合理性? A.大多数购买列出排名的杂志的人是不准备进大学的人。 B.杂志中排名最高的学校将这一事实作为广告以吸引学生。 C.该排名逐年很少变化。

D.某具体标准对任何两个学生的意义可能因他们不同的需要而不同。

E.一些对其所在大学满意的大学学生在他们做出选择去哪所大学读书的决定前考虑的该杂志的排名

21. 在某纺织企业的一次干部会议上,关于是否让一部分职工先下岗的问题有两种不同意见。一方认为:让一部分职工下岗可以减轻企业的经济负担,增加企业的竞争力。

另一方认为:企业职工下岗后,企业要承担政府部分分配的下岗职工保险费。结果还是羊毛出在羊身上,不解决问题。

下面哪项最有力地反驳了反对职工下岗者的意见? A.下岗职工再就业是很困难的。

B.很多没有下岗的企业竞争力越来越弱。

C.下岗职工中很少有为了领取下岗补贴而选择下岗的。

D.政府征收下岗职工的保险费对所有企业都是一视同仁的。 E.用于交纳保险的费用在短期内比给下岗职工的工资高。 22. 总统之所以任命乔治为副总统,保守党人之所以对这项任命感到高兴,是因为他们确信,这项任命如果能被上议院通过的话,最高法院中的政治平衡至少在数年内会持续出现有利于


右翼势力的倾斜。乔治之所以遭到自由党人的激烈反对是因为他们确信,保守党人是右翼势力。

从上述的议论中,可以推出以下哪项结论? A.乔治是保守党人。

B.如果保守党人不支持对乔治的提名,自由党人将不反对乔治。 C.目前在最高法院中左翼势力占有利地位。

D.如果自由党人不反对乔治,保守党人将不支持对乔治的提名。 E.自由党人确信乔治将作出有利于保守党人的决策。

23. 计算机程序的特别之处在于,它是唯一受专利权和版权保护的产品。专利权保护的是一种发明的创意,而版权保护的是这种创意的表述,但是为了获得两方面的保护,这种创意和它的表述必须得到严格的划分。

根据以上陈述,可以推出如下哪项结论?

A.计算机程序的创意和它的表述可以区分开来。

B.任何计算机程序的设计者都是这一个程序创意的发明者。

C.受版权保护的大部分产品都是对某种受专利权保护的创意的表述。 D.很少有发明家既是专利权的所有者,又是版权的所有者。 E.一个获得了专利权的计算机程序,很容易就可获得版权。

24. 甲:政府在卡瑟纳省通过鼓励创造工作机会、减少失业率的计划已经失败了,因为在该计划实施一年以后失业率仍没有变化。

乙:但是在计划开始前的三年中,卡瑟纳的失业率是上升的,因此该计划是有帮助的。 下列哪一个,假如正确,最强地反对乙对甲论述的反对? A.政府由一些专门研究失业问题的经济学家提供建议。 B.卡瑟纳省的失业率在历史上比整个国家总体失业率高。

C.当前的政府以很大优势当选的原因是许下了为卡瑟纳省减少失业率的诺言。 D.在政府计划开始的时间里,大量卡瑟纳的居民离开该省去其他地方寻找工作。 E.卡瑟纳省的失业率在现任政府当政以前一直相对稳定。 25. 某先生每天早晨的工作内容和工作程序如下,醒来后立即在煤气灶上烧水(10分钟即开)打开煤气灶后就开始穿衣服(5分钟即完)然后出门去取牛奶(来回共5分钟)回来后就烧牛(10分钟即开),同时开始洗漱(5分钟即毕),最后是喝奶(5分钟喝完)。又知道,他家中只有一个灶头(热源),他又必须在745分离开家,否则将迟到。

如果我们忽略像打开煤气灶等其他占用时间极少的工作,你同意下列哪种观点?

A.该先生上班前有抽一支香烟(5分钟抽完)的习惯,如果他715分醒来并起床,他就没有时间抽香烟了。

B.如果前一天已经取来牛奶,则该先生就可以晚起来5发钟,上班仍不会迟到。 C.如果前一天已经烧好水,则该先生就可以晚起来10分钟,上班仍不会迟到。

D.如果前一天既烧好水,又取来了牛奶,则该先生就可以晚起来15分钟,上班仍不会迟到。 E.只要他每天715分起来,他不但有时间抽香烟(5分钟抽完),而且上班也不会迟到 参考答案隐藏如下:

01.E 02.B 03.E 04.C 05.C 06.E 07.C 08.B 09.E 10.A 11.E 12.A 13.A 14.D 15.C 16.D 17.E 18.A 19.B 20.D 21.D 22.E 23.A 24.D 25.E

01.哲学应当在那些学生年纪很小的时候就教。这样就能慢慢地灌输给他们对传统观念的健康有益的怀疑论。

上面的推论做了以下哪项假设?

I. 学生们如果不是在很小的年纪就接触哲学,他们就能多接受一些观念。


II. 甚至在很小的年纪,学生们就能够懂得一些哲学概念。 III.学生们对传统观念质疑是很好的主意。 A. III B. IIIII C. II D. III E. IIIIII

02-03在历史上,世界经济的繁荣无不是建立在导致新产业诞生的新发明连续不断的浪潮的基础之上的,从汽车、飞机产业到化工、制药、电子等领域。因此,经济持续发展的动力保证应当是产业界增加在科学研究和开发上的投入。 02.上面这段话的作者做了下面哪项假设?

I. 科学研究和开发能够确保产生一系列的新发明。

II. 化工、制药和电子等产业将会在未来几年中提供大量的新技术。

III.目前产业界投入科学研究和开发的资金量还不足以确保一系列新发明的产生。 A. IIII B. IIIIII C. III D. III E. I

03.以下哪项如果为真将最能削弱上面的推论? A. 在目前的这个资金水平上,公司的研究开发部门申请专利的数量比起十年前来要少得多。 B. 大部分产业的研究开发部门关心的只是对现有产品进行的微小改进而不是开发什么新的技术。

C. 历史上,只有一些新的主干行业是直接依赖公司研究开发部门获得的技术突破的。 D. 公司在科学研究和开发上的投入与公司每年新的发明专利的数量直接相关。 E. 政府对科学研究和开发的投入将在未来五年中大大缩减。

04.中国不适宜发展私人汽车,因为中国人口众多,城市人口密度过大,交通设施(包括停车设施)落后,城市道路容量有限,现在的车流量已使城市交通不堪重负,如果再大量发展私人汽车,势必造成难以解决的社会问题。

以下哪项如果为真,则最有力地削弱了上述论证?

A. 随着经济的发展,无论从个人的经济能力还是从国家的经济实力看,都具备了发展私人汽车工业的条件。

B. 日本东京的人口总量和密度不亚于中国任何一个城市,它也曾经存在过交通设施滞后的问题,但它现在是世界上拥有私有汽车最多的城市之一,并没有出现难以解决的社会问题。 C. 各国的经验表明,在发展车和发展路的关系上,都是车的发展促进了路的发展。促进城市道路建设的动力之一就是发展汽车工业,包括私人汽车工业。

D. 衣、食、住、行是人的物质生活的四大要素,没有任何理由不让中国的普通老百姓也享受私人汽车的便利,特别是当他们拥有相应的经济能力之后。

E. 国外的财团纷纷看好中国的私人汽车产业,这方面的外部投资有着光明的前景。

05.各电视台、报纸、杂志经常统计和公布固定播放的各栏目的收视率及观众的性别、年龄和受教育的情况,这对于广告策划人员和想要进行广告宣传的企业都是十分有用的。 下述哪项如果为真,则最能有力地支持上文的推论。

I. 广告策划人员在制作广告时,应预估有多少人可能会看到它。

II. 广告要想引导消费、推介商品,其内容和形式必须注意要有针对性。


III. 电视台和其它宣传媒介采用的抽样调查的办法,其可靠性是一个需要研究的问题。 IV. 教育的程度是能够影响一个人对于广告信息的理解的。 A. IIIIII B. IIIIIIV C. IIIIIV D. IIIIV E. IIIIIIIV

06.航空公司:最新开发的避免撞机系统,尽管没有进行完全的测试以发现潜在的功能失灵,但应该立即安装在客机上。它们的自动警报能使飞行员避免相撞。 飞行员:飞行员不能驾驶装有未经过完全测试的避免撞机系统的飞机,系统功能失灵会误导飞行员,造成撞机。

如果以下哪一项是正确的,将最有力地加强飞行员的反对意见? (A)机械装置总有可能失灵。

(B)喷气发动机,尽管第一次投入使用时没有经过完全测试,但已经取得了优异的表现和安全记录。

(C)尽管避免撞机系统可以使飞行员避免一些撞机事故,但未经完全测试的系统可能的失灵会导致更多的撞机事故发生。

(D)许多飞机相撞事件是部分由于工作负担过重的飞行员的过度疲劳引起的。

(E)发展到这个阶段的避免撞机系统在为期6个月的试验飞行过程中,在客机上比在货机上工作得更好。

07.一个出版商目前开展了这样一项业务,专门为大学中的教授针对各自的课程定制教科书。教授可以删除一本书中他不感兴趣的几个章节,也可以添加一些内容。 这种定制业务的应用,在以下哪种教育目的中最不适合? A.教科书的内容针对某一学生的特殊需要。

B.在高级的选修课中针对某一个问题进行深入的研究。 C.在全国范围内学生的统一必修读物。 D.使大学教材满足某一教师的要求。

E.为了增强学生对这门课程的兴趣,在教材中引用生动、可读性强的指定阅读材料。

08.政府的教育投入不见得真正有利于学生。在七十年代和八十年代,美国政府用于教育面的投入的总量增加了150%,而在此期间,学生在标准考试中的成绩却逐年下降。 上述论证基于下面哪一项假设?

A. 学生在标准考试中的成绩是衡量教育方面投入的有效性的一个主要标准。

B. 在七十、八十年代,越来越多的学生对各种教育项目,包括政府资助的项目都失去兴趣。 C. 在七十、八十年代,美国的教育投入本来是可以较好地用于健康和福利事业的。 D. 在七十、八十年代,学生在美国人口中的比例没有什么大的变化。

E. 在七十、八十年代,美国大学毕业生中选择教师职业的人数有很大的下降。

09.不管广告上如何宣传,所谓的低尼古丁烟不见得比其他烟更安全。因为,所谓的尼古丁含量低,是有关部门用专门的吸烟仪器测定的,而不是依靠对人的测定。但人吸烟与仪器吸烟毕竟是不同的。通过对吸烟者的血样研究,没有在吸不同品牌的吸烟者之间发现什么实质性的差异,也没有发现摄入的尼古丁和吸烟的数量之间的关系,不管吸的是什么品牌的烟。 如果有人要反驳以上的论证,维护“低尼古丁烟比其他烟更安全”的说法,最可能会提出以下哪项?

A. 大多数吸烟的人并不在乎吸烟是否有损健康。 B. 被动吸烟者受到的损害比主动吸烟者还大。


C. 一个吸烟者完全可能使他们的吸烟方式类似于吸烟仪器。 D. 大多数烟草公司生产各种品牌的烟,其中包括低尼古丁烟。

E. 烟草公司必须向商检部门申报产品的尼古丁含量,并在外包装上注明。

10.郭教授不但学术知识渊博,而且非常有行政工作能力,在他当上哲学系系主任之后,报哲学系的学生人数有了非常明显的增加,这在当前哲学等基础理论学科倍受冷落的情况下是非常不容易的。

上述推理假设了以下哪项前提?

I. 报考哲学系学生人数的增加要归结于郭教授的努力。

II. 郭教授在哲学界享有很高的声誉,在社会上也有很高的知名度。 III. 考生的人数是衡量系主任工作能力的一个重要尺度。 A. I B. II C. III D. IIII E. IIIIII

11.目前有人设计了一种安装在汽车中的气袋,它能够直接在撞车事故发生时迅速自胀,减轻乘员在碰撞时的承受压力,从而避免死亡。设计者认为气袋应该象安全带,安装在每一个新汽车里面。有人对此持反对意见,认为,安装气袋设施将增加新汽车的成本,使得国产汽车在竞争时处于不利的地位,因为外国政府并没有要求它们的汽车制造商也给新汽车安装气袋。

下面哪一项如果为真,则最能削弱上述反对意见的论证?

A. 大多数接受调查的汽车司机都认为气袋并没有比安全带增加更多的效能。 B. 在十分之九的汽车事故中,使用安全带都最大程度地减少了伤亡。 C. 根据对气袋的测试,其质量合格率几乎达100% D. 要求安装气袋的规定同样也适用于进口汽车。 E. 气袋的研制已经引起了越来越多的国家的注意。 12.只有坚持社会主义才能救中国。

以下诸项都准确地表达了上述断定的含义,除了: A. 除非坚持社会主义,否则不能救中国。 B. 如果不坚持社会主义,那么就不能救中国。 C. 如果救了中国,说明一定坚持了社会主义。 D. 坚持社会主义,是救中国的必不可缺的条件。 E. 只要坚持社会主义,就一定能救中国。 13.小贾说:“我们班的每一个同学都是体育达标的。 如果他说的话事实上是错误的,则下面哪一项是真的呢? A. 他们班没有一个同学体育达标。 B. 他们班至少有一个同学体育达标。 C. 小贾自己体育是达标的。

D. 他们班至少有一名同学的体育没有达标。 E. 小贾自己的体育就没有达标。

14.妈妈和儿子点点在为整理房间的事争论着。 妈妈:你不把房间整理好,就别想去看电影。

点点:我早已整理好了,这下我可以去看电影了吧。

妈妈:不行,我只是说,如果你不整理好房间,你就不能去看电影。


根据上文,判断下面哪一项是错误的?

A. 妈妈 的意思是点点只有整理好房间才能去看电影。 B. 妈妈 的意思是点点没有整理好房间就不能去看电影。

C. 点点认为妈妈 的意思是如果他整理好房间就可以去看电影。 D. 点点认为妈妈 的意思是只有他整理好房间才能去看电影。 E. 妈妈 的意思是点点即使整理好了房间也不一定能去看电影。

15.研究人员发现,软底运动鞋会吸收能量,使运动员难以创造好的成绩。他们认为,高弹性的鞋底比软底更能将能量反送给运动员,从而提高他们的成绩。很显然,未来最佳的运动鞋必定是高弹性底的运动鞋。

下面哪一项如果为真,最能推翻上述推断?

A.许多人购买运动鞋是为了日常使用,而不是为了参赛。 B.今天的软底运动鞋比起三十年前的皮底运动鞋要优越得多。 C.高弹性底的运动鞋对足部关节压力过大,容易使运动员受伤。 D.随着训练技术的提高,运动员对器材、装备的要求也越来越高。 E.高弹性底的运动鞋生产成本低于软底的运动鞋。

16.一项对于黄金时间电视节目收视情况的调查结果表明,对于收视率相近的节目的质量,观众给予的评价却有很大的分歧。这一结果对广告公司可能很有价值,因为他们可能会因此而把广告费投在观众觉得质量高的节目上。 下面哪一项如果为真,则最能支持上述关于“观众对收视率接近的电视节目质量的看法的信息对广告公司可能会很有价值”的主张?

A.被观众普遍认为高质量的节目在全部被评节目中占有较大的比重。 B.被观众普遍认为高质量的节目通常是在经济台和教育台播出的。 C.电视观众比较经常地记得他们所喜欢的节目的赞助商的名字,而不怎么记得那些他们认为没有什么特色的节目的赞助商的名字。

D.电视观众通常只有在新节目是伴随他们所熟悉的老节目出现时才会看新节目。 E.电视观众反映说电视广告的质量对于他们的购物习惯没有什么影响。

17.通过计算机辅助设计量身定做的假肢虽然要比普通假肢多花近两倍的钱,但还是非常值得的。因为不仅手术时间和康复时间都缩短了,而且假肢使用寿命也长了,从而减少了今后再进医院的可能性。

为证实上述论断,应研究以下哪个问题?

A. 病人动手术的时间与手术后的康复时间之比。 B. 由于采用新技术制作假肢带来的成本降低的幅度。

C. 与使用普通假肢相比,采用量身定做的假肢减少再次手术的程度。

D. 采用新技术生产假肢比生产普通假肢在加工过程上的精细程度的比较。 E. 当生产实现标准化或达到大规模生产时,采用新技术可降低成本的幅度。

18.只有社会主义才能救中国,我们坚持了社会主义,所以我们一定能够救中国。 以下哪个推理最有力地说明了上述推理的不成立?

A. 只有学习好,才有资格当三好学生,我学习好,所以我一定有资格当三好学生。 B. 只有学习好,才有资格当三好学生,我有资格当三好学生,所以,我一定学习好。 C. 只有学习好,才有资格当三好学生,我没有资格当三好学生,说明我学习不好。 D. 只有学习好,才有资格当三好学生,我学习不好,因此我没有资格当三好学生。

E. 只有学习好,才有资格当三好学生,因此没有资格当三好学生的,不见得学习成绩一定不好。

19.向一群不同年龄的孩子朗读有关事故肇事者的故事,故事中的当事人有的是故意的,有


的是无意的。当被问到对这些肇事者应采取什么样的惩罚措施时,年龄小的孩子不象年龄大的孩子那

样对故意伤害和过失伤害进行了区别对待。因此,年龄小的孩子不认为惩罚时应考虑肇事者的动机。

下面哪一项如果为真,则最能削弱以上论证?

A. 在这些故事中,受伤害的严重程度都有明确的描述。

B. 在就这些故事进行判断时,听故事的人只有具备相对成熟的人类心理才能判定伤害是出于有意还是无意。

C. 年龄小的孩子和年龄大的孩子一样都可能造成无意的伤害。

D. 年龄大的孩子在处理惩罚的问题上,已经和成年人在类似情况下采用的方法很类似。 E. 年龄小的孩子根据故事里所描述的伤害程度来决定采取何种惩罚方式。

20.一项针对超过五万名瑞典士兵所做的十五年的调查显示,大量吸食大麻的人得精神分裂症的可能性是不吸食大麻的人的六倍。研究报告的作者说,精神分裂症与大麻之间的相关并不一定意味着毒品会引起精神分裂症。相反,研究可能说明,使用大麻是由精神分裂症的发作引起的。

下面哪一项在结构上与上文最为相似?

A. 法官发现被告犯了盗窃财产和企图出售盗窃赃物罪,并各判他五年监禁,同时执行。 B. 这个园丁总是在土豆旁边种上金盏花,几年来我们一直以为这只是为了使花园更漂亮,但园丁向我们解释说金盏花有助于保护土豆免受线虫的侵害。

C. 最近该国街头犯罪案件(如抢劫)上升可归于两个因素:使用毒品的增加(如可卡因和海洛)以及高的失业率。

D. 据原始的意识观点,行为和动作是由灵魂引起的。例如,树摇摆并不是由于风吹的缘故,而是因为树的灵魂对风的灵魂的运动做出的和谐反应。

E. 医生向我解释,造成我脚踝疼痛的,可能不是软底鞋一侧的过度磨损,相反,由于脚踝疼痛,我可能下意识地把重量移向脚外侧,造成了鞋子磨损。 参考答案

01.B 02.A 03.B 04.B 05.D 06.C 07.C 08.A 09.C 10.D 11.D 12.E 13.D 14.D 15.C 16.C 17.C 18.A 19.B 20.E

01.最近,新闻报道说成百只的海豹因为吃了受到化学毒物污染的黄花鱼而死去。但是,尽管许多人也吃黄花鱼,但却没有一则人吃黄花鱼中毒的报道。 以下哪项为真则最能正确地解释这两种不同的结局?

A.化学毒物污染的黄花鱼本身并不会受这种毒物的影响。 B.比煮鱼所用的温度还高的温度也不会影响化学毒物的毒效。 C.最近我们发现了一些人身体中有化学毒品的存在。

D.被海豹吃下的受污染的黄花鱼只占海豹总食量的一小部分。

E.海豹吃的是受化学毒物污染的黄花鱼,而人们吃的是未受污染的黄花鱼。

02.如果公司的亏损进一步加大,那么是总经理不称职;如果没有丝毫撤换总经理的意向,那么总经理就是称职的;如果公司的领导班子不能团结一心,那么是总经理不称职。 假定上述逻辑关系确实严格存在,如果总经理不称职,那么下列哪项一定是事实? A.公司的亏损进一步加大了。 B.出现撤换总经理的意向。

C.公司的领导班子仍不能团结一心。

D.公司的亏损进一步加大,并且出现撤换总经理的意向。

E.公司的领导班子仍不能团结一心,并且出现撤换总经理的意向。


03.赵、钱、孙、李星期五要参加公司的职位晋升业务资格考试。他们希望考前在一起复习,取长补短,争取好成绩。但是因为各自工作时间的限制,每个人可能的复习时间如下: 赵只能在周一晚、周二晚、周三晚和周四的下午和晚上复习; 钱只能在周一晚、周三晚、周四晚和周二的下午和晚上复习; 孙只能在周三晚、周四晚,周一、二这两天的下午和晚上复习; 李只能在周二、周三、周四这三天的下午和晚上以及周一下午复习。 如果赵决定只是每晚参加复习,那么,以下哪个判断是真的? A.他每晚都有可能与孙一起学习。 B.他没有机会与李一起学习。 C.他没有机会与孙一起学习。 D.周一晚上他只能独自学习。 E.周四晚上他只能与钱一起学习。 04.甲和乙之间有以下一段对话:

甲:根据人口统计资料可以发现这样一条规律,在新生婴儿中,男婴的比例总是在22/43下波动,而不是1/2

乙:不对吧,根据我掌握的许多资料,多数国家和地区,如前苏联、日本、德国以及我国的台湾省都是女人比男人多。可见,认为男婴出生的比例总在22/43上下波动是不成立的。 请指出下列各选项中哪一项根据上述对话判断是可以确认的。 A.乙的资料是不完整的,因此不可信。 B.乙混淆了讨论的概念。

C.甲所说的统计规律确实是不存在的。 D.甲所依据的统计调查是不科学的。 E.甲本身的陈述中就存在着矛盾。

05.同是一种化妆品,在包装简陋的情况下价格低而且滞销,而在换用精美包装后,则价格高而且畅销。如果其它因素都未发生变化,就可以断定包装的好坏与售价、销售情况有因果关系。

以上推理过程与下面所列各项中的哪个最为相似?

A.将几种不同品种的畅销名牌商品进行比较(如海尔冰箱、长虹彩电、小天鹅洗衣机等),虽然它们的构造、用途等都各不相同,但它们的一个共同特点就是质量都比同类商品好。由此可见,商品的质量好是这些商品畅销的重要原因。 B.松下公司生产的耐火板,开始取名为“松下耐火板”,结果销售额很低。后来他们把名称改为“厨房守护神”,结果成了畅销品。由此看出,商品的名称与销售情况有因果关系。 C.LQ-1600K作为EPSON公司的拳头产品畅销不衰,国产的打印机难以企及。然而国产打印机的保修能力是绝不亚于EPSON公司的,可见,保修能力的强弱并不是影响打印机市场销售状况的关键因素。

D.这两年引进的美国电影在中国受到异乎寻常的欢迎,而几乎每部影片的制作成本都远高于中国影片。可见影片的制作成本的高低对于影片的受欢迎度起着关键性的作用。

E.这个四川餐厅比起隔壁的海鲜馆生意可差多了,看来在北京,还是粤菜、潮州菜更有前途哇。

06.长安大街上有两个邻近的影剧院:长虹影院和紫光剧场。长虹影院的每日电影的放映场数要少于紫光剧场,而票价又基本相同,所以长虹影院的收入要比紫光剧场少。 项目哪项如果为真,就指出了上述论证中逻辑推理的缺陷?

I. 长虹影院在电影选择上比紫光剧场要严格,所以不能投合一般市民的兴趣。 II. 长虹影院的座位要比紫光剧场多一倍。


III.上演话剧的成本要比上演电影的成本大得多。 A. I B. II C. III D. III E. IIIII

07.社会科学家们研究发现:对某种社会现象影响最大的要素往往在量上只占少数。比如,社会财富的80%以上集中在不到20%的人手中。 以下现象中哪项不符合上述断定?

A.国家人口的大部分主要集中在少量的国土上。

B.生产中产品质量的控制往往只受某些关键环节的影响。 C.学生学的最多的是大量的基础知识。

D.大多传媒上报道的核心内容只集中在几篇报道。 E.家庭支出大部分只花在少数几个方面。

08.在美国, 大约有5000名犯有非暴力行为的罪犯, 被处以////务的刑罚, 而不去蹲监狱。 这些犯人所//务的项目是和他们的经历或他们所掌握的技术相适应的,从擦地板到为国家搞科研, 样样都有。判刑为////, 1979年实行以来, 越来越广泛地被采纳。 个月前, 对酒后开车处罚所颁布的法规也使用这种刑罚。

1979年所采纳的为////务的处罚, 最有可能是受下列七十年代所发生的哪件事情的敦促而产生的?

A.国内暴力犯罪的减少。 B.国家雇员犯罪数量的增加。 C.国内中年法官的逐渐减少。 D.国内监狱的拥挤。

E.其它州对酒后开车的法规的通过。 09.甲和乙任何一人都比丙、丁高。

如果上述为真,再加上下述哪项,则可做出“戊比丁高”的结论? A.戊比甲矮。 B.乙比甲高。 C.乙比甲矮。 D.戊比丙高。 E.戊比乙高。

10.由于邮资上涨,主要通过邮购的《家庭装饰》杂志计划把月刊改为双月刊来增加利润,杂志的质量、发行量、年定价都不改变。市场调查表明,如果计划得以实施,承销商和广告商都不会有任何损失。

以下哪项如果为真,则能够提出最有力的证据证明该计划实施后杂志的收入将有所减少? A.在新的邮资下,每邮寄一份杂志,成本将增加三分之一。

B.杂志主要的承销商不关心杂志的质量是否会下降,而是关心在目前杂志的高质量下,杂志的发行是否会出现亏损。

C.即使年定价提高,许多长期订购此杂志的订户仍会订购。

D.绝大多数在该杂志上做广告的广告商还会象过去一样每登一期广告支付与原来同样的广告费。

E.杂志的制作成本会保持不变。

11.一项对婚姻关系的调查表明,在一对配偶中,如果一方的作息时间和另一方有严重冲突,


两个人发生争执的比例比作息时间相合的夫妇中要高。看来,这种不和谐的作息时间会破坏一个婚姻。

以下哪个选项如果为真,能够最有效地削弱上述观点?

A.那些作息时间比较一致的夫妇,偶尔也会发生一些争执,这些争执也会导致婚姻的破裂。 B.一个人的作息时间是会随季节的变化而变化的。

C.那些作息时间不一致的夫妇在工作中很少与同事发生争执。

D.人们发现那些不幸婚姻中的夫妇经常采取与对方不同的作息时间来表示其敌意。 E.最近一项调查表明,人们的作息时间是很容易被控制和更改的。

12.有许多公司现在免费向员工提供健身课程,帮助他们锻炼身体、减轻压力,甚至学习怎样戒烟。这些课程提高了员工的生产力,降低了他们的缺勤,并且可以使公司减少了保险支出,因此,这一课程既对公司有益,又对员工有益。 以下哪项如果为真,能够有效地支持上述结论?

A.健身课程是许多公司向员工提供的很普及的服/务。 B.研究表明,在压力管理下的练习对很多人都没有效果。

C.常规性的锻炼能够减少人们患心脏病的可能,并使他们精力充沛。 D.过快地适应高负荷的健身课程,容易造成伤病。 E.公司需要专门雇佣一些员工来指导各种锻炼课程。

13.在一项有关颜色和劳动生产率的关系的研究中,原来有100名员工在一间颜色单调的车间中工作,其中50人现在搬到了颜色明亮、丰富的车间中工作,其余50人仍留在原车间工作。调查结果显示,新车间和旧车间的劳动生产率都有所提高。 以下哪个选项如果为真,能够解释上述现象?

A.搬到颜色明亮的车间人员完成的工作与留在旧车间的人员完成的工作类似。 B.颜色单调的旧车间只适合容纳65名员工工作。

C.颜色明亮的新车间的50名工人在年龄和工作水平上与留在旧车间的员工的基本相同。 D.几乎所有的人都愿意搬到新车间去。

E.很多搬到新车间的工人表示他们宁愿留在旧车间。

14.某公司财务处失窃,职员小丁涉嫌被讯问,保安人员问他的第一个问题是:“你以后还敢不敢再偷?”

上述提问方式,与以下哪项最为类似?

A.小凡考试粗心,数学只得了90分,爸爸斥问他:“你以后还敢不敢粗心啦?” B.老郑耗尽积蓄去某地游玩,结果大失所望,邻居老陈幸灾乐祸,问老郑:“你以后还去玩吗?”

C.小胡酒后驾车,结果翻车住院,还被罚了款。小胡的女友又气又急,责问他:“你以后还敢酒后开车吗?”

D.某酒吧因提供/////被公安局查封,停业整顿半年后才复业。执法人员问酒吧经理:“你们酒吧以后还敢不敢再犯了?”

E.文化大革命中的一次批斗会上,造反派质问一位死不认罪的老干部:“你以后还敢不敢再走资本主义道路了?”

15.如果甲和乙都没有考试及格的话,那么丙就一定及格了。 上述前提再增加以下哪项,就可以推出“甲考试及格了”? A.丙及格了。 B.丙没有及格。 C.乙没有及格。

D.乙和丙都没有及格。


E.乙和丙都及格了。

16.现在是投资宴席行业的好时机。《婚礼》杂志的一项调查发现杂志的读者中有70%在婚礼中设宴。对于宴席行业的调查表明,现有的宴席公司只能为每年55%的进行婚礼的新人提供服务。

以下哪个选项,如果为真,可以揭示上文的错误? A.宴席行业是一个劳动密集型的行业。 B.宴席业在全国并不均匀分布。

C.设宴的婚礼数目在过去的五年中每年都有所增长。

D.《婚礼》杂志的读者比绝大多数人都更希望在婚礼中设宴。

E.《婚礼》杂志中既有介绍宴席公司办设宴婚礼的文章,又有怎样自己为婚宴准备食物的文章。

17.以下哪个选项,如果为真,会降低上题中有关投资建议的有用性? A.每个婚宴的平均参加人数是50100人。 B.大约有1/4的婚礼不设婚宴。

C.几乎所有的婚礼及婚宴都是由新婚夫妇来付款的。 D.只有一半的婚礼中包括正式的宴席。

E.希望设婚宴的夫妇中只有一半真正这样做了。

18.一切有利于生产力发展的方针政策都是符合人民根本利益的,改革开放有利于生产力的发展,所以改革开放是符合人民根本利益的。 以下哪种推理方式与上面的这段叙述最为相似?

A.一切行动听指挥是一支队伍能够战无不胜的纪律保证。所以,一个企业、一个地区的发展壮大,也必须从大局出发,把人民利益放在首位。

B.经过对最近六个月销出的“命之星”健身器的质量跟踪调查,发现没有一台因质量问题而退货或返修,所以说,这批健身器的质量是合格的。 C.如果某种产品超过了市场需求量,就会出现滞销现象,“大富翁”领带的供应量目前超过了市场需求,因此,就一定会出现滞销现象。

D.凡超越代理人权限所签合同是无效的,这份房地产建设合同是超越代理权限所签合同,以是无效的。

E.我们对一部分实行产权明晰化的企业进行调查,发现通过产权明晰都提高了经济效益,没有发现反例,因此我们认为,凡是实行产权明晰化的企业都能提高经济效益。 19.甲、乙、丙三人有如下一段对话。

甲:恐龙灭绝的原因是由于全球性的气候剧变极大地减少了以前丰富的食物来源。 乙:不对,恐龙的灭绝是由于出现了新的动物家族哺乳动物。哺乳动物繁殖迅速,动作敏捷,生存力极强,成为与恐龙争夺食物的致命对手。

丙:研究表明曾经发生过行星撞击地球,引起烟云遮日达数月乃至数年,大量的作为恐龙食物的植物相继枯亡。

以下哪项最为恰当地表述了三人的不同意见?

A.三人都同意饥饿是引起恐龙灭绝的原因,但对引起食物短缺的原因有不同意见。

B.三人都同意气候的改变引起了恐龙食物的短缺,但并不一致认为这是造成恐龙灭绝的原因。

C.甲和丙同意巨大的气候变化引起了恐龙的灭绝,但对气候剧变的原因有不同的解释。 D.乙和丙同意饥饿引起了恐龙的灭绝,并且同意这是由于地球上的植物总量减少引起的。 甲和丙同意饥饿引起恐龙的灭绝,但不同意丙关于行星撞击地球引起气候变化而减少恐龙食物来源的看法。


20.目前,临海市主要干道上自行车道的标准宽度为单侧3米。很长一段时期以来,很多骑自行车的人们经常在机动车道上抢道骑行。在对自行车违章执法还比较困难的现阶段,这种情况的存在严重地影响了交通,助长了人们对交通法规的漠视。有人向市政府提出,应当将自行车道拓宽为3.5米,这样,给骑自行车的人一个更宽松的车道而能够消除自行车抢道的违章现象。

下列哪项如果为真,最能削弱上述论点?

A.拓宽自行车道的费用太高,此项建议的可行性太差。

B.自行车道宽了,机动车走起来不方便,许多乘坐公共交通的人会很有意见。 C.拓宽自行车道的办法对于机动车的违章问题没有什么作用。

D.当自行车道拓宽到3.5米以后,人们仍会在机动车道上抢道违章。 E.自行车道拓宽,自行车速加快,自行车撞车增多,违章事件会更多。

21.如果公司的财务部门没有人上班,我们的支票就不能入帐;我们的支票不能入帐,因此,公司的财务部门没有人上班。

请在下列各项中选择出来与上句推理结构最为相似的一句。

A.如果太阳神队主场是在雨中与对手激战,就一定会赢。现在太阳神队主场输了,看来一定不是在雨中进行的比赛。

B.如果太阳晒得厉害,黎明就不会去游泳。今天太阳晒得果然厉害,因此可以断定,黎明一定没有去游泳。

C.所有的学生都可以报名参加这一次的赛决赛除非没有通过资格赛的测试。这个学生不能参加决赛,因此他一定是没有通过资格赛的测试。

D.倘若是妈妈做的菜,菜里面就一定会放红辣椒。菜里面果然有红辣椒,看来,是妈妈做的菜。

E.如果没有特别的原因,公司一般不批准职员们请事假。公司批准了职员陈刚的事假,看来一定是有一些什么特别的原因。

22.经理:会计部和出纳部在公司中相邻,而且工作内容相似,但是办公用品的支出上,出纳部门总是比会计部门高很多,你们出纳部门是否存在浪费现象呢? 出纳部门负责人:不是这样的。

以下选项如果为真,能够最有效地支持出纳部门负责人的观点: A.会计部门比出纳部门的职员多

B.两年前会计部门和出纳部门在办公用品的消耗上相近。 C.出纳部门现在与过去相比,有很多新品种的办公用品的需求

D.两个部门都发现,一些原来用纸笔完成的工作,现在可以被计算机取代。

E.出纳部门的办公用品柜离会计部门近,他们更方便从出纳部门的柜子中取办公用品。 23.最近举行的一项调查表明,师大附中的学生对滚轴溜冰的着迷程度远远超过其它任何游戏,同时调查发现经常玩滚轴溜冰的学生的平均学习成绩相对其它学生更好一些,看来,滚轴溜冰可以提高学生的学习成绩。

下面哪一项如果为真,则最能削弱上面的推论? A.玩滚轴溜冰有助于智力开发,从而提高学习成绩。 B.玩滚轴溜冰能够锻炼身体,保证学习效率的提高。 C.只有聪明的孩子才能玩好滚轴溜冰。

D.师大附中与学生家长订了协议,如果孩子的学习成绩的名次没有排在前二十名,双方共同禁止学生玩滚轴溜冰。

E.玩滚轴溜冰很难,能够锻炼学生克服困难做好一件事情的毅力,这对学习是有帮助的。 24.在对某生产事故原因的民意调查中,70%的被访问者认为是设备故障,30%的被访问者认


为是违章操作,25%的被访问者认为原因不清,需要深入调查。 以下哪项如果为真,则最能合理地解释上述看来矛盾的陈述? A. 被调查的人一共有125名。

B. 有的被调查者改变了自己的原有观点。 C. 调查操作出现技术性差错。

D. 有的被访问者认为事故的发生既有设备故障的原因,也有违章操作的原因。 E. 很多的认为原因不清的被访问者实际上有自己倾向性的判断,但是不愿意透露。

25.年轻的朋友总是梦想作家的生活是一种充满魔力、很富有、有名望的生活,但是很快就发现不仅他们象工匠那么辛苦,而且长时间不被社会承认、贫困孤独地生活。当有人问道:“难道不正是那些许许多多的编辑使作家失败的吗?”时,艾略特评价道:“是的,但是因此作家更多了”

艾略特的回答传达了什么样的观点?

A. 少数的作家能够非常幸运地在他们的行业获得真正的成功,成为真正的作家。 B. 编辑工作象写作一样充满创造力和挑战性。

C. 对于一个作家来说,成功是用影响力而不是他们掌握的财富来衡量的。 D. 许多作家发现编辑的限制会对他们长期的学徒生涯会很有益处。

E. 对作家来说,成功与失败没有明显的标准,但是对编辑来说,却有一定之规。 参考答案:

01.E 02.B 03.A 04.B 05.B 06.B 07.C 08.D 09.E 10.D 11.D 12.C 13.B 14.E 15.D 16.D 17.E 18.D 19.A 20.D 21.D 22.E 23.D 24.D 25.A

01.以两面针、芳草、中华等为代表的国产品牌的牙膏在质量、品种、保健和价格上都相当不错,又在各种渠道展开了强大的广告攻势,占据了国内市场消费量的七成以上,使国外品牌的进口牙膏难以称雄,这与其它许多日用品的市场形成鲜明的对照,值得其它国产日用品厂商认真反思。

以下哪项不适合作为以上论述的理由?

A. 国产品牌牙膏洁齿效果好,品质优良,使用方便。

B. 国产品牌牙膏的价格合理,经济实惠,适合中国普通老百姓的消费水平。

C. 国产品牌牙膏充分利用了中药特有的保健作用,洁齿护齿有效,吸引了国内众多的消费者。

D. 电视中的精彩广告使国产牙膏的品牌老少皆知,广为传颂。 E. 我国有超过12亿的人口,构成了一个巨大的消费市场。

02.希望工程突然收到一大笔没有署名的捐款,经过多方查找,可以断定是赵、钱、孙、李中的某一个人捐的。经询问,赵说:“不是我捐的”钱说:“是李捐的”孙说:“是钱捐的”李说,“我肯定没有捐”。最后经过详细调查证实四个人中只有一个人说的是真话。 根据已知条件,请你判断下列哪项为真? A.赵说的是真话,是孙捐的。 B.赵说的是假话,是赵捐的。 C.钱说的是真话,是李捐的。 D.孙说的是真话,是钱捐的。 E.李说的是假话,是李捐的。 03.小赵比小钱个子高; 小孙比小李个子高; 小李个子不如小周高;

小钱和小周个子正好一样高。


如果上述这些陈述都是真实的,那么,下列的哪项也必定是真的? A.小孙比小周个子高。 B.小孙比小赵个子高。 C.小钱比小孙个子矮。 D.小赵比小李个子高。 E.小钱比小李个子矮。

04.现在准备从赵、钱、孙、李、周、吴六个工程技术人员中选出三位组成一个特别攻关小组,集中力量研制开发公司下一步准备推出的高技术拳头产品。为了使工作更有成效,我们了解到以下情况:(1)赵、孙两个人中至少要选上一位;(2)钱、周两个人中至少选上一位;(3)孙、周两个人中的每一个都绝对不要与钱共同入选。

根据以上条件,若周未被选上,则下列中哪两位必同时入选? A.赵吴 B.钱李 C.钱吴 D.赵李 E.赵钱

05.有研究者认为,国际互联网Internet并不会导致色情、暴力等内容信息的泛滥。因为Internet蓬勃发展的这两年中,以色情和暴力内容为主的信息节点在整个网络信息节点中的比例是在降低而不是增加。

以下哪项如果为真,则最能反驳上述观点?

A.色情、暴力信息大多为图象信息,传递、阅读时比较耗费时间。

B.色情和暴力节点的访问人次数超过一般网络节点平均访问人次数六倍以上。 C.色情、暴力内容的信息想要减少直至消灭,必须靠各国信息界的共同努力。 D.色情和暴力的内容这两年在通过Internet和各种其它媒体广泛传播着。 E.色情和暴力的内容如果被儿童看到,会产生比成人恶劣百倍的坏作用。 06.“并非所有的干部都是懂法的” 此句与下列哪句意义完全相同? A.有些干部是懂法的。 B.有些干部不是懂法的。

C.并非所有的干部都不是懂法的。 D.干部不懂法。

E.干部不懂法,一点不稀奇。

07.禁止使用香烟的品牌作为职业体育比赛的冠名并不会减少吸烟的人数,观众们知道从哪里能弄到香烟,而不需要广告为他们提供信息。 如果下述哪项为真,则最能反驳上述观点?

A.禁止使用香烟的品牌作为职业体育比赛的冠名会使职业体育比赛的经费发生很大的困难。 B.人们从广告中获得的并不只是供货信息,更多的是一种消费时尚和需求欲望的唤醒。 C.香烟的危害使得把它作为职业体育比赛的冠名会引起体育迷们的愤怒。

D.女子职业体育比赛越来越普及的今天,使用香烟品牌冠名是不尊重她们的表现。

E.禁止使用香烟的品牌作为职业体育比赛的冠名会使通过其它媒介的香烟广告更吸引人。 08.这栋楼中有的住户家中发现了蟑螂。

如果以上这一断定为真,则在下述三个断定中不能确定真假的是: .这栋楼中没有住户的家中不发现蟑螂。 .这栋楼中有的住户家中没有发现蟑螂。 .这栋楼中所有的住户家中都未发现蟑螂。 A.III B.IIIII C.IIIIII D.I E.II


09.中周公司准备在全市范围内展开一次证券投资竞赛。在竞赛报名事宜里规定有“没有证券投资实际经验的人不能参加本次比赛”这一条。张金力曾经在很多大的投资公司实际从事过证券买卖操作。

那么,关于张金力,以下哪项是根据上文能够推出的结论: A. 他一定可以参加本次比赛。

B. 他参加比赛的资格将取决于他证券投资经验的丰富程度。 C. 他一定不能参加本次比赛。 D. 他可能可以参加本次比赛。

E. 他参加比赛的资格将取决于他以往证券投资的业绩。

10.199771日以后,每一台由香港本地生产的机器上,都会在机壳的显著位置表明Made in China

下面哪项可以从上述结论中正确推论出来?

A. 199771日以后,机壳显著位置标有“Made in China”字样的机器,有一部分是由香港本地生产的。

B. 香港199771日以前不能够由本地自行生产机器。

C. 有些香港本地生产的机器在199771日以前就标有“Made in China”的字样

D. 199771日后没有标有“Made in China”字样的机器,不得由香港进入大陆销售。 E. 199771日前香港本地生产的机器是都是在机壳显著位置,标有Made in Hong Kong的字样的。

11.公司需要招聘一位有经验的有价证券组合管理人,最终入围最后面试的是三个人:赵、钱和孙。三个人中,笔试成绩最差的,是赵;面试成绩最好的,是钱。最后总成绩最好的,不是孙。已知,笔试、面试和总成绩中,三个人的排列顺序没有一次完全相同。三个成绩排序,有一个人的顺序始终未变。

请问三个人的总成绩由低到高的排列顺序。 A.已知信息不完全,无法作出准确判断。 B.钱、孙、赵。 C.钱、赵、孙。

D.以上条件矛盾,没有符合条件的顺序。 E.赵、孙、钱、

12.两位教授在讨论人造宇宙飞船的未来。

罗教授:人造宇宙飞船没有什么前途,因为在实现空间飞行的手段中,人造宇宙飞船在成本上最为昂贵。 周教授:没有哪一种人类的飞行器有过人造宇宙飞船这样好的安全记录:二十五年来只发生过二次事故。因此,人造宇宙飞船确实会很有前途。

以下哪项是对周教授针对罗教授观点所进行的论辩在逻辑上最好的评价?

A. 如果周教授的论辩给出的证据是真实的话,他是试图证明罗教授的论据是错误的。 B. 周教授的论辩提出了一个比罗教授的结论更有价值的观点。

C. 周教授的论辩没有能够针对罗教授的观点,因为他没有能够揭示是否空间飞行可以先于宇宙飞行资金的筹集。

D. 周教授的论辩没有针对罗教授的观点,因为他假定用人造宇宙飞船进行空间飞行没有任其它的困难,而困难问题正是罗教授的观点。

E. 周教授的论辩指出了罗教授的论据与结论之间的很大的分歧。

13.达尔文的生物进化学说,揭开了人类进化研究的世纪性的新篇章。然而至今,人类仍然无法找到从猿向人进化的关键性证据。所以,人们只得又在达尔文学说的前面重新加上“假


说”这两个字。

以下各项中,有哪一项是上文推理中隐含的假设? A. 达尔文的学说还不能算是经过科学验证的真理。

B. 达尔文的生物进化学说的关键性证据是世纪性的悬案。 C. 缺少科学上关键性证据的学说只能称为假说。

D. 认为达尔文的生物进化学说千真万确的观点是伪科学的。

E. 达尔文如果在世,从猿向人进化的关键性证据就不会还是个谜了。

14.在生产塑料制品时使用石油应当用法律进行限制和规范。我们国家对石油在能源供应上的需求比塑料要重大得多。此外,我们对进口石油日益增长的依赖可能会导致一个严重的后果,比如说,如果战争爆发,进口的渠道就失去了。通过减少我们在塑料生产中的石油用量,我们就向能源供应的自力更生迈了一大步,也更增强了我们国家的安全。 以下哪项为真,则最能削弱上述推论?

A. 塑料生产中的新工艺能够降低生产中所需的石油量。 B. 原子能的发展,作为以石油为基础的能源的替代因为公众对安全的担心而变的缓慢起来。 C. 这个国家消耗的石油中只有一小部分是用于塑料生产的。 D. 在战时,参战国会严格地控制非石油生产国与外国的贸易。

E. 有些塑料制品,例如飞机和摩托上的部件,在保卫国家上作用很大。 15.每个人都可以随时赞美某些人。

以下哪项与上面的这句话吻合,不矛盾?

.张三和李四随时都可能被人赞美,也随时都可以赞美你。 .你随时都可以赞美某些人。 .你只能在某些时候赞美人。 A. I B. III C. IIIIII D. II

E. IIIII

16.调查表明,最近几年来,成年人中患肺结核的病例逐年减少。但是,以此还不能得出肺结核发病率逐年下降的结论。

以下哪项如果为真则最能加强上述推论?

A. 上述调查的重点是在城市,农村中肺结核的发病情况缺乏准确的统计。 B. 肺结核早就不是不治之症。

C. 和心血管病、肿瘤等比较,近年来对肺结核的防治缺乏足够的重视。 D. 近年来未成年人中的肺结核病例有明显地增多。 E. 防治肺结核病的医疗条件近年来有较大的改善。 17.并非小张既高又胖。

如果上述断定为真,那么下述哪项断定必定为真? A. 小张高但不胖。 B. 小张胖但不高。 C. 小张既不高也不胖。

D. 如果小张高,小张一定不胖。 E. 如果小张不高,那么他一定胖。

18.新民生住宅小区扩建后新搬入的住户们纷纷向房产承销公司投诉附近机场噪声太大令人难以忍受。然而,老住户们并没有声援说他们同样感到噪声巨大。尽管房产承销公司宣称不


会置住户的健康于不顾,但还是决定对投诉不准备采取措施。他们认为机场的噪声并不大,因为老住户并没有投诉。

下列哪项如果为真,则表明房产承销公司对投诉不采取措施解决的做法是错误的? A. 房产承销商们的住宅并不在该小区,所以不能体会噪声的巨大危害。 B. 老住户自己都配备了耳塞来解决这个问题,他们觉得挺有效果的。 C. 老住户觉得自己并没有与房产承销商有什么联系,也没有太大的矛盾。

D. 老住户认为噪声并不巨大没有声援投诉是因为他们的听觉长期受噪音影响已经迟钝失灵。

E. 房产承销公司从来没有隐瞒过小区位于飞机场旁边这一事实。

19.在美国,政府保障所有个人的银行储蓄,以防银行倒闭。一位经济学家认为,正是这一保障措施,使得美国的银行倒闭率非常高。由于个人储蓄得到保障,民众不再注意银行本身的风险,如果储户对银行倒闭的问题更敏感,各家银行将为竞争而更注意防范风险。 这位经济学家是以以下哪个选项为假设的? A.巨额贷款不能回收是银行倒闭的主要原因。 B.相当多的储户都会把钱分别存在不同的银行中。 C.储户手中钱越多,选择银行越审慎。

D.不同的银行提供不同的利率不是银行倒闭的主要原因。 E.储户有能力判断哪家银行的风险更小。 20.本题题干与上题相同。

以下哪个选项最有效地削弱该经济学家的观点?

A.在政府提供个人储蓄保障之前,银行的倒闭率比现在低。

B.如果政府不向个人储户提供这种保障,更多的银行会因为储户不敢储蓄而倒闭。 C.调查表明,绝大多数的储户都知道他们的储蓄是得到政府的保障的。

D.政府对个人储蓄的保障是有上限的,但很少有储户的储蓄能超过这一界限。 E.银行的风险,取决于他们资产中贷款所占的比率和每一笔贷款的风险。 21.住在临海市的每一个人都要付税;

没有一个人能够不理睬通货膨胀的影响,除非他住在临海市; 每一个付税的人都发牢骚。

根据上面的这些句子,判断下列各项哪项一定是真的? I. 每一个不理睬通货膨胀影响的人都要付税。

II. 不发牢骚的人中没有一个能够不理睬通货膨胀的影响。 III.每一个发牢骚的人都能够不理睬通货膨胀的影响。 A. I B. III C. II D. IIIII E. IIIIII

22.在我国高科技产业竞争如此激烈的今天,我们不难预料未来的竞争更加残酷。在数学物理等基础科学方面具有一定知识和技能的工人越来越需要。然而在中学数学物理等方面的教师越来越缺乏、而且没有什么改变的趋势的情况底下,我们很怀疑未来是否有这么多的技术工人可用。产业界可以帮助来解决这个问题:建立奖学金来资助那些在数理专业的大学学生,希望他们将来有可能从事中学教师职业。

以下各项如果为真,有哪项最有可能妨碍上面这种办法达到它的目标? A.许多高中已经开始强行降低他们聘用数理方面教师的标准。


B.人口统计数字表明未来十年中高中学生的人数有望进一步增加。 C.给数理专业学生提供的奖学金比起给人文科学的来要少。

D.当他们大学毕业后,大部分数理专业的学生仍向产业界求职而非选择教师职业。 E.许多专家说未来的商业需要许多经过初步训练的、有技术功底的雇员。

23.名牌进入千家万户,家庭电器化、电脑化和“洋货化”已经成为时尚。但同时人们也发现,这是一个美丽的陷阱,流行是短暂的,掏不完的钱带给消费者的是新的遗憾,聪明的消费者渐渐醒悟:买家电还是要量力而行。

以下哪项如果为真,最能构成对上述看法的怀疑和动摇? A.电器的流行在一定时期内代表了更为先进的技术水平。

B.购买时尚的家电,就要求装修相配合,财力的消耗提醒我们注意自己的家底。 C.30%购买豪华家电的居民经济能力并不具备,但从众心理促使他们紧跟消费浪潮。 D.投资电脑能获得知识、技能的回报,带给消费者的是比遗憾更多的成功和自信。 E.家电价格不断下调,用旧过时的家电只需支付很少的价格补差就可以更新换代。 24.如果钟明入选了奥运圣火火炬接力,那么他一定是18岁以上了。 上面这个判断中隐含了什么假设?

A. 只有钟明能够入选奥运圣火火炬接力。

B. 只有18岁以上的人才能入选奥运圣火火炬接力。 C. 有一些18岁以上的人入选了奥运圣火火炬接力。

D. 有一些18岁以上的人没有能够入选奥运圣火火炬接力。 E. 钟明没有拒绝参加奥运圣火火炬接力。 25.检察部门对黎明公司的总经理张军说,“你不把公司的账解释清楚,你就别想离开本市”两天后,张军找到检察部门把账解释清楚后说,“这下我可以离开本市了吧?”检察部门却说:“不行,我们只是说,如果你的账解释不清,你就不能离开本市。 根据上面的情况,请判断下列哪句的含义与上文不符?

A. 张军认为检察部门的意思是只有他解释清楚了账,才可以离开本市。 B. 检察部门的意思是张军解释清楚账只是他离开本市的必要条件之一。 C. 检察部门的意思是即使张军解释清楚了账,也不一定就能离开本市。 D. 张军认为检察部门的意思是只要他解释清楚了账,就可以离开本市。 E. 检察部门的意思是张军解释清楚账不是他离开本市的一个充分条件。 参考答案:

01.E 02.B 03.D 04.E 05.B 06.B 07.B 08.A 09.D 10.A 11.D 12.D 13.C 14.C 15.B 16.D 17.D 18.D 19.E 20.B 21.B 22.D 23.E 24.B 25.A

1.一家珠宝店的珠宝被盗,经查可以肯定是甲、乙、丙、丁中的某一个人所为。审讯中,甲说:“我不是罪犯。”乙说:“丁是罪犯。”丙说:“乙是罪犯。”丁说:“我不是罪犯。”经调查证实四人中只有一个人说的是真话。 根据已知条件,下列哪个判断为真? A.甲说的是假话,因此,甲是罪犯。 B.乙说的是真话,丁是罪犯。 C.丙说的是真话,乙是罪犯。 D.丁说的是假话,丁是罪犯。

E.四个人说的全是假话,丙才是罪犯。

2.有人说,彻底的无私包含两个含义:第一,无条件地实行为他人服务;第二,拒绝 何他人的服务。

下述哪项是上述观点的逻辑推论?


A.没有人是彻底无私的。

B.不可能所有的人都是彻底无私的。

C.如果有人接受了他人的服务,那么一定存在彻底无私的人。 D.如果有人拒绝了他人的服务,那么一定存在彻底无私的人。 E.彻底无私的人要靠教育来造就。

3.某单位为了提高干部的业务素质和管理能力,实现管理的现代化、科学化,决定举办计算机应用知识培训班,号召干部们积极参加培训。小张约小李一道去报名参加培训,李回答说:“我又不是从事计算机专业工作的,有什么必要一定要去参加计算机知识培训,我的工作业绩和管理能力是有目共睹的。 上文中小李的回答包含了什么错误前提? A.人们学习计算机应用知识是没有必要的。 B.计算机的普及是相当遥远的事。

C.计算机知识的学习应当在学生中进行。 D.计算机不可能代替人脑的思维。

E.只有从事计算机专业工作的,才应该学习和掌握计算机知识。

4.语言不能生产物质财富,如果语言能够生产物质财富,那么夸夸其谈的人就会成为世界上的富翁。

下面哪项论证在方式上与上述论证最类似?

A.人在自己的生活中不能不尊重规律,如果违背规律,就会受到规律的无情惩罚。 B.加强税法宣传十分重要,这样做可以普及税法知识,增加人们的纳税意识,增加国家财政收入。

C.有些近体诗是要求对仗的,因为有些近体诗是律诗,而所有律诗都要求对仗。 D.风水先生惯说空,指南指北指西东,倘若真有龙虎地,何不当年葬乃翁。

E.金属都具有导电的性质,因为,我们研究了金、银、铜、铁、铅这些金属,发现它们都能导电。

5.并非小张既高又胖。如果上述断定是真的,那么,下述哪项一定是真的? A.小张高但不胖。 B.小张胖但不高。 C.小张既不高也不胖。

D.如果小张高,那么他一定不胖。 E.如果小张不高,那么他一定胖。

6.在国际贸易中,当小国和穷国与大国和富国发生贸易纠纷时,拿到世界贸易组织wTO)去仲裁,肯定会比双边谈判更能得到一个好的结果。 支持上述论点的主要论据是以下哪项?

A.处理贸易纠纷的双边谈判是以本国法制为依据的,这对法制不健全的弱小国家十分不利。

B.世贸组织(wTO)有关调解纠纷的多边规则是完全公平合理的。 C.调解贸易纠纷的多边规则是偏袒弱小国家的。

D.解决国际贸易纠纷应以世界贸易组织(wTO)中的有关规则为裁定标准。 EJ国家要尽快建立起自己完善的法律体系才不至于在国际贸易纠纷中处于不利 位。

7.地球和月球相比,有许多共同属性,如它们都属太阳系星体,都是球形的,都有自转和公转等。既然地球上有生物存在,因此,月球上也很可能有生物存在。 以下哪项如果为真,则最能削弱上述推论的可靠性?


A.地球和月球大小不同。

B.月球上同一地点温度变化极大,白天可以上升到100℃,晚上又降至零下160℃。 C.月球距地球很远,不可能有生物存在。 D.地球和月球生成时 间不同。 E.地球和月球旋转速度不同。

8.随着中国经济的发展,私人轿车进入家庭,对中国人来说已不再是遥远的梦想。然而,随着私家车的日益增多,将使本来就十分严重的交通紧张状况面临雪上霜的困境,因此,中国的交通问题最终必然限制私人轿车的发展。 以下哪项如果为真,则最能驳斥上述观点?

A.私人轿车的增多不是交通紧张状况加剧的主要原因。 B.应先发展交通,再发展轿车工业。

C.随着轿车进入家庭,必将带动中国交通的大发展,从来就是先有车,然后才有路。 D.在人口众多、交通不发达的中国,发展私人轿车是愚蠢的。 E.在中国只有发展公共交通和自行车才不会加剧交通紧张状况。

9.有人认为,我国人口多,生产力还很落后,失业问题严重,特别是近年来,随着大量农村剩余劳动力向城市的转移,给就业问题形成了新的更大的压力。因此,首要的问题是考虑如何更多地增加就业,吸纳安排剩余劳动力,而不是提高劳动生产率。 以下哪项如果为真,则最能驳斥上述观点?

A.应加快发展劳动密集型产业,创造更多的就业机会。

B.在中国不适宜发展技术密集型产业,否则会使日趋严重的失业问题,雪上加霜。 C.每个成年人都有自我保护和生存的能力,大量个体户的发家致富说明中国失业问题是被人为地夸大了。

D.只有加速发展,才能解决日益严重的失业问题。

E.从全社会的经济发展来说,由于劳动生产率的提高会形成更多的积累,促使生产规模不断扩大,从而为更多的人创造就业机会。因此,提高劳动生产率和安排就业两者并不矛盾。

10.美国有一种叫做“小兔读书”的学习软件,通过动画教孩子认字念书,很受孩子和家长的欢迎,目前已售出200多万套。但儿童教育专家通过对49名学龄前儿童的对比研究发现,儿童使用“小兔读书”及类似软件6个月,创造力下降了50%,而阅读能力并没有明显提高。

从以上介绍中可以得出下述哪种结论? A电脑不能取代蜡笔和纸这样最原始的学习工具,因为后者能任儿童的思维自由驰骋。 B.电脑对儿童智力的其他方面的发展有积极影响,如反应速度。 C.电脑对学龄前的儿童的作用不如对已入学儿童大。 D“小兔读书”受欢迎程度足以说明其积极作用。 E.儿童使用电脑会妨碍其社会交往能力的发展。 11“万物生长靠太阳”,这是多少年来人们从实际生活中总结出来的一个公认的事实,然而,近年来科学家们研究发现:月球对地球的影响远远大于太阳;孕育地球生命的力量,来自月球而非太阳。

以下哪项不能作为上述论断的证据?

A.在月照下,植物生长快且长得好,月照特别是对几厘米高、发芽不久的植物如向日葵、玉米等最有利。

B.当花枝因损伤出现严重伤口时,月光能清除伤口中那些不能再生长的纤维组织,加快新陈代谢,使伤口愈合


C.植物只有靠了太阳光才能进行光合作用,动物也只有在阳光下才能茁壮成长。 D.月球在地球形成之初,影响地球产生了一个巨大磁场,屏蔽来自太空的宇宙射线对地球的侵袭。

E科学家在太平洋加拉帕戈期群岛附近的深海海底,发现并采集了红色的蠕虫、张着壳的蛤、白色的蟹等,这可能与月照有关。

12.目前,国内彩电市场竞争十分激烈。进口名牌彩电款新质优,但价格比较昂贵;而国产彩电虽以质量价格比优于进口彩电而日受欢迎,但也将面临严峻的考验。据统计, 1994年全国国产彩电销量为1100万台,而目前全国的彩电生产能力已达2000万台以上。 以上论断中得不出以下哪项推论?

A.彩电生产厂家之间的竞争将不可避免。 B.彩电生产厂家应通过规模经济降低成本。 C.国产彩电目前是买方市场。 D.中国如加入世界贸易组织(WTO,国内家电业将会受到进一步冲击。 E.国产彩电受欢迎程度不亚于进口彩电。

13.有一家电力公司,靠着建造发电量较大、效率较高的电厂,以及刺激该地区用电量这两种方法,已经使得利润大为增加,并能够向消费者提供价格低廉的电力。为了维持这种兴旺局面,该公司计划以一座新电厂来取代一座旧电厂,而新电厂的发电量是该公司原有电厂的3倍。

下面哪一点不在该公司的上述计划考虑之内? A.该公司供电地区的电力供应量未来将会增加。

B.新建电厂的开支额度不能超过提高经营效率或扩大规模带来的收益水平。 C.拟议中的新电厂完全能够提高整个公司的效益。 D.新电厂的安全措施设计将与原来的老电厂一样。 E.发电量增加两倍不会有什么技术上的障碍。 14.某外国航空公司经理:“新开发的避撞系统,虽然还未经全面测试以发现潜在的问题,但也必须马上在客机上安装,因为这个系统的机械报警装置可以使飞行员避免撞机事故。该公司飞行员:“飞行员不能驾驶一架避撞系统未经全面测试的飞机,因为有故障的避撞系统将会误导飞行员,造成撞机。

以下哪项如果为真,则最能加强飞行员的反对意见? A.机械设备总是有可能出现故障。

B.喷气式发动机在第一次投入使用之前也未经彻底测试,但是其性能与安全记录却是有目共睹的。

C.虽然避撞系统能使飞行员避免一些相撞事故,但是未经测试的避撞系统的潜在问题可能会造成更多的撞机事故。

D.许多撞机事故是由于飞行员过度疲劳造成的。

E.处于目前开发阶段的避撞系统,在6个月的试用期间,在客机上的工作效果比在货机上好。

15.许多消费者并没有充分利用他们所购买的运动器材。例如:据调查,美国有17的成年人都有跑鞋,但其中只有45%的人一个月跑一两次, 17%的人一周跑一次以上。 以下哪种说法如果为真,则最能构成对于“许多消费者没有充分利用他们所购买的运动器材”这一结论的疑问?

A.跑步者在刚开始跑步的6个月里,很容易使肌肉拉伤。 B.在有关的调查中,跑步者经常夸大跑步的次数。 C.许多消费者买跑鞋是为参加其他活动,而不是跑步。


D.喜欢跑步的消费者通常都买运动鞋。

E.每周都跑步一次以上的人可能是其他运动的积极分子。 参考答案:

1A 2A 3E 4D 5D 6A 7B 8C 9E 10A 11C 12B 13D 14C 15C 01. 香港的繁荣是事实。英国对香港的殖民统治也是事实。有人因此得出结论:是英国的统治造就了香港的繁荣。

以下哪项,如果为真,则最有力地削弱了上述推论?

A. 香港的繁荣仅是近几十年的事,而英国对香港的殖民统治已经达到百年。 B. 英国本土的经济一直处在不景气与衰退之中。 C. 绝大多数英国殖民地都已经获得了独立。 D. 亚洲“四小龙”中的其它“三小龙”,并不是英国的殖民地。 E. 香港的繁荣得益于它的国际金融中心的地位。

02. 据世界卫生组织1998年年会“Smoking or Health”公布的调查报告显示,70%的肺癌患者都有吸烟史。这充分说明,吸烟将极大地增加一个人患肺癌的可能性。 以下哪项,如果为真,则将最严重地削弱上述结论?

A.进入20世纪后半叶以来,未成年吸烟者的比例有惊人的增长。 B.有吸烟史的人在1990年到1998年一般都超过世界总人口的66% C.1998年全世界吸烟的人数比1997年增加了70% D.被动吸烟者发现同样有导致肺癌的危险。

E.没有吸烟史的人数在九十年代超过世界总人口的50% 03-04 在某校的家属区中: 所有的小保姆都加入了工会。 有些清洁工是湖北人。 有些小保姆是湖北人。

所有的工会会员都入了医疗保险。 没有清洁工入医疗保险。

03. 以下各项都能依据上述前提推出,除了: 在该校家属区中:

A. 所有的小保姆都入了医疗保险。 B. 有些湖北人入了医疗保险。 C. 有些湖北人没有入医疗保险。 D. 有些小保姆兼当清洁工。 E. 没有清洁工加入工会。

04. 以下哪项所描述的那个人是上述前提所做断定的一个反例? A. 一个女清洁工。

B. 一个入了工会的清洁工。 C. 一个没入医疗保险的湖北人。

D. 一个人入了医疗保险,但并非小保姆。 E. 一个人入了医疗保险,但并非清洁工。 05. 去年,全美国由受害妇女报告的强奸案件的数量较前年增加了20%具有讽刺意味的是,这个统计数字是美国女权组织在其年度业绩报告中作为成绩公布的。 以下哪项,如果为真,则最能合乎逻辑地解释上述看起来是矛盾的现象? A. 近三年来,美国的强奸犯罪率在逐年提高。

B. 近三年来,美国的强奸犯罪率在刑事犯罪中的比例是逐年下降的。


C. 美国女权组织一直把预防强奸犯罪作为自己的一项重要任务。

D. 美国国会通过的鼓励受害妇女控告强奸犯的法规大大地减少了被施暴后不敢报告的受害妇女的人数。

E. 美国法院增加了打击强奸犯罪的力度。

06. 随着市场经济体制的不断建立,我国的一些城市出现了这样一种现象:许多工种由外来人口在做,而本地却有大量的待业人员存在。各城市的就业条件都是一样的,我们假设并无其他限制。

以下各项都可能是造成这种现象的原因,除了: A. 外来的劳动力大多数是其他地区的待业人员。 B. 本地人对工种过于挑剔。

C. 外地的劳动力的价格比较低廉。 D. 外来的劳动力比较能吃苦耐劳。 E. 本地人对劳动报酬要求比较高。 07. 为了测试今后的消费趋势,《消费者》杂志对读者作了一次消费意向调查。60%的被调查者声称计划在三个月内购买一台空调或至少一件家电大件。《消费者》杂志因此得出结论,下个季度的社会消费额将可能很高。

以下哪项,如果为真,则最能削弱以上结论?

A.家电中,各种不同品牌的产品在价格上有很大的差异。

B.某些抢手的家电产品是进口货,但并不会形成对国内市场的冲击。 C.并非所有《消费者》的读者都接受了调查。 D.从定价等方面看,《消费者》的读者比普通消费者要更为富裕。 E.空调的价格有可能下调。 08. 人们总是这样质问律师:“你明知罪犯有罪,你为什么还要真诚地为他辩护?” 律师回答说:“我这样做,是为了维护法律赋予被告的合法权利,这对于实施法律的公正是必不可少的。

从律师的回答中,我们能得出以下哪项结论?

I. 被告即使是真的罪犯,也拥有法律赋予的合法权利。

II. 只要维护被告(包括真正的罪犯)的合法权利,就能保证实施法律的公正。

III. 如果剥夺那些明显是罪犯的被告的一切权利,那么就无法保证实施法律的公正。 A. IIIIII B. I C. III D. IIII E. IIIII

09. 有一位从事因特网(Internet)教学工作的专家一次在一个城市公开演讲,说到Internet世界上最大的相互连接起来的计算机网络,可以完成小型局部的计算机网络永远也办不到的事情。一位听众听到这里,站起来问道:“昨天晚上我在电视里看到了世界上最大的一个西瓜,可是,我并不觉得它有什么特殊之处呀”

根据上文情景,这位听众的问话隐含了以下哪项假设?

A. 比较大小对能力、性质的决定,西瓜可能与计算机网络不同。 B. 比较大小对能力、性质的决定,计算机网络与西瓜并无不同。

C. 大的西瓜也不过就是让人吃而已,当然还可以出名,让大家都知道。 D. 通过电视,我们就可以看到一些没有网络以前不可能看到的新鲜事。 E. 小型局部的计算机网络确实不能具备象Internet那么大的功能。


10. 报社的编辑收到如下来信: 亲爱的编辑:

在这个假期里我认为我们应该认真地去体会一下真正的给予。我们每一个人都应该决心给别人礼物而不期望任何回报。如果别人给我们礼物,我们应该拒绝他的好意,并劝他把礼物送给其他人。通过这种方式我们就都可以体会到真正的给予。 下列哪个选项指出了上文的逻辑错误:

A.假期并不是一年中人们唯一的接受礼物和送礼物的时间。 B.如果没有人接受任何礼物,则任何人都不可能送出一份礼物。

C.人们收到的礼物往往并没有用处,所以拒收礼物并不会有多大的牺牲。 D.一些人可能会送出礼物,但希望在一段时间以后有所回报。 E.送礼有上千年的传统,而且存在于各种人的群体中。 11. 明天不必然土也 震。

以下哪项断定与上述断定的意思最为相近? A.明天必然不土也 震。 B.明天可能土也 震。 C.明天可能不土也 震。 D.明天不可能土也 震。 E.明天不可能不土也 震。

12. 小郭和小万在讨论这次学校的“艺翔助学金”发放的一些情况。 小郭:这次没有女生获得“艺翔助学金”的资助。

小万:那就是说这次全校的“艺翔助学金”的名额都空缺了。 小郭:不,事实上这次咱们学校有几位男生获得了“艺翔助学金” 小万的回答里面可能假设了以下所有的断定,除了: A. “艺翔助学金”只发给女生。

B. 只有女生才有资格申请“艺翔助学金”

C. “艺翔助学金”的申请者中,所有的女生都比男生更够条件。

D. “艺翔助学金”的申请者中,大部分的女生比大部分的男生更够条件。 E. 按规定,男生和女生必须获得相等数量的“艺翔助学金”名额。

13. 一天晚上有一个人在一条街上丢了他的钥匙,便在路灯下寻找。旁边有位路人奇怪地问他,“你的钥匙是掉在路灯下的吗?”;他回答说“我也不知道,可是现在只有路灯下能够看得见啊”

以下哪项假设最能反映了这位路人的思路? A. 丢了钥匙不一定要找,可以另配一把。

B. 丢了钥匙一定要在丢掉的地方寻找,否则不可能找到。 C. 丢了钥匙要在能够看得见的地方寻找。

D. 丢了钥匙在路灯下寻找,找到的可能性不一定比在别处来得大。 E. 晚上丢了钥匙,可以等天亮了在找,更容易找到。

14. 我国著名教育家、清华大学前校长梅贻琦先生在其就职演说时曾说过“所谓大学者,非谓有大楼之谓也,有大师之谓也”,成为后来许多大学纷纷效仿的教育方针。 根据梅先生的话,可以符合逻辑地推出以下哪项结论?

A. 目前国内各个大学的设备比起国外来都还有很大差距,所以,很难延聘到大师。 B. 一个大学之所以为大学,全在于有没有好教授,罗织人才是当今大学第一要务。 C. 大学的发展,资金是基础,有了充足资金的保证,不愁吸引不到一流的人才。 D. 青年学生在选择大学就读时,要综合考虑大学的基础设施和师资力量两个方面。


E. 一些大学出现师资大量外流的情况,反映出大学领导者不重视培养大师的态度。

15. 现在不少青年男女没有经过法律上登记结婚的手续,就共同生活在一起,结果出现了财产等民事纠纷时,不能得到法律上的承认和保护。因此,登记结婚在其法律上的严肃性之外,还有着避免婚姻关系变化后导致的无谓纠纷的作用,当前要加强这一工作。 以下哪项为真,则最能加强上述论点?

A.这一代年轻人对男女之间的关系比以前要看得淡,他们追求一种相互之间的投合。 B.现代社会婚姻的变化比起以前要多变,不少婚姻的结局是离异和分手。 C.不尊重婚姻的人,也得不到婚姻的尊重。

D.法律上登记结婚的手续,可能会防止重婚罪的发生。 E.并非每一对非婚同居者都会发生财产上的纠纷。 16-17

有些新雇员一进厂就当了机关干部。在该厂的整顿改造中,所有的上海籍员工都支持孙阳当选厂长,所有的机关干部都反对孙阳当选厂长。

16. 如果上述断定为真,以下哪项关于该厂的断定是真的? A.所有的新雇员都是上海籍。 B.有些上海籍员工是机关干部。 C.有些新雇员不是上海籍。

D.并非所有上海籍员工都是新雇员。 E.某些机关干部是上海籍。

17. 如果在上述断定中再增加以下断定:“所有的新雇员一进厂就都当了机关干部”,并肯定这些断定均为真,那么,以下哪项必定为假? A. 某些新雇员不是上海籍。

B. 并非所有机关干部都是新雇员。 C. 某些上海籍员工是新雇员。

D. 所有的新雇员都反对孙阳当选。 E. 某些机关干部不是上海籍。

18. 那些在1968年当教师,后来改变职业的人中有30%现在的年收入在35 000元以上,而那些在1968年当教师,后来仍从事这一职业的人中只有15%现在年收入高于35 000元。 以下哪个选项是上述论述的前提:

A.没有改变职业的教师中至少有三分之一如果改变他们的职业,现在年收入会在35 000以上。

B.离开教师岗位的人和没有离开的人,他们的收入因素上是能比较的。 C.绝大多数离开教师岗位的人完全是因为教师的收入太低了。

D.那些留在教师岗位的人比那些离开教师岗位的人更有能力,贡献更大。

E.那些离开教师岗位,现在年收入高于35 000元的人比留在教师岗位的人更有能力。 19. 最近的一项报告表明,虽然在玛丽兰州只有3%在高速公路上驾驶的人在他们的车上装了雷达探测器,33%的超速罚单是开给这些人的。很明显,在车上装雷达探测器的人比没有装的更容易超速。

上述结论是以以下哪个选项为前提的:

A.在车上装雷达探测器的司机比没有安装的司机容易受到超速罚单。 B.因超速被罚的司机比那些不被罚的司机更多地超速驾驶。 C.因超速被罚的车的数目大于装雷达探测器的车的数目。 D.在调查阶段,许多车不止一次因超速被罚。

E.在玛丽兰州的高速公路上驾驶的司机比在其它州驾驶超速现象更严重。


20. 所有的零部件都检查过了。

如果上述断定为真,则在下述三个断定中可确定为假的是: .没有零部件被检查过。 .有的零部件被检查过。 .有的零部件没有被检查过。 A.I B.II C.III D.IIII E.IIIII

21. 药品制造商:尽管我们公司要求使用我们新药的病人同时购买一次性的用于每周血液测试的工具,那些工具的花费是完全需要的;每周必须做血液测试以监视药的潜在的可能非常危险的副作用。

下列哪一个,假如正确,最反对制造商品的论述?

A.购买血液测试工具的花费没有阻止任何病人获得药和工具。

B.医学实验室能够做血液测试,对病人或他们的保险商的要价低于制造商对工具的要价。 C.一年的药物和每周的血液测试工具使病人或他们的保险商花费超过1万美元。

D.大多数政府和其他健康保险项目不补偿病人为药品和血液测试工具所付的全部费用。 E.遭受该药一个或一个以上危险的副作用的病人会花费很多钱治疗。

22. 1987年,鼻窦炎是某国最普遍的慢性病,以下依次是关节炎和高血压。关节炎和高血压的发病率随年龄增长而增大,但鼻窦炎的发病率在所有年龄段都是相同的。该国人口的平均年龄在19872000年间将有所增加。

以上信息,可以对该慢性病状况作出的结论哪个最恰当? A.2000年,关节炎和高血压将比鼻窦炎更普遍。 B.2000年,关节炎将成为最普遍的慢性病。

C.19872000年间,鼻窦炎患者的平均年龄将增加。 D.2000年,患鼻窦炎的人数比1987年减少。

E.2000年,相当一大部分人口都将患以上所提到的慢性病中的一种。

23. 佛吉尼和弟弟威廉在他们的父亲的出生日期上有分歧,佛吉尼认为父亲生于1935年,而威廉认为其生于1933年。他们父亲出生的医院没有1933年的记录,但是有1935年的完整记录,但是其中不包括其父亲的记录。因此,他们的父亲一定出生于1933年。 上面的论述依赖下列哪一个假设?

A.佛吉尼和威廉的观点中有一个是正确的。 B.其父亲出生的医院的记录可追溯到1933年。 C.佛吉尼和威廉知道其父亲的出生月份和日期。

D.佛吉尼和威廉为什么必须知道他们父亲出生的日期这件事有着紧迫的实际的原因。 E.他们的其他亲属中没有一个人知道佛吉尼和威廉的父亲出生的年代。

24. 12月上旬,某城市的气候已相当寒冷,湖面的冰层已经非常坚实。溜冰爱好者都希望到湖面溜冰场去溜冰。但溜冰场要等到12月中旬才开放。为此,溜冰爱好者颇有意见。 以下哪项最不可能是溜冰场管理人员所作出的解释:

A.在12月上旬,由于时间仓促,溜冰场的开放工作还没有准备就绪。

B由于门票收费过低,上级领导又不同意提高门票收费,多开一场,溜冰场的亏损就越大。 C.溜冰场每年都到12月中旬开放,这已形成惯例。

D.溜冰场要到12月中旬开放的根本原因是为了杜绝冰塌落水的事故。


E12月上旬溜冰场安排有赛事。

25.桑洛镇最近通过一项禁止在全镇范围内所有餐厅吸烟的法律,因为通常在桑洛餐厅吃饭的许多人不愿意在吃饭时控制吸烟,桑洛的餐厅将毫无疑问地失去许多顾客和相当多的收入。

下列哪一个,假如正确,最帮助加强上面的论述? A.大多数在餐厅吃饭的桑洛的居民是不吸烟者。

B.大多数与不吸烟者吃饭的吸烟者愿意在吃饭时控制住吸烟。 C.假如在餐厅中限制吸烟的法律没有被制定,可能将制定更严格的在桑洛所有公共场合限制吸烟的法律。

D.在桑洛禁止吸烟的法律通过前,小镇有一项要求大多数餐厅有无烟区的法令。

E.与桑洛相邻的其他区域,有许多与桑洛的餐厅差不多的餐厅,没有制定或执行任何反吸烟者的法律。 参考答案:

01.B 02.B 03.D 04.B 05.D 06.A 07.D 08.D 09.B 10.B 11.C 12.D 13.B 14.B 15.B 16.C 17.C 18.B 19.B 20.D 21.B 22.C 23.A 24.D 25.E 1. 第一个事实:

电视广告的效果越来越差。一项跟踪调查显示,在电视广告所推出的各种商品中,观众能够记住其品牌名称的商品的百分比逐年降低。 第二个事实:

在一段连续插播的电视广告中,观众印象较深的是第一个和最后一个,而中间播出的广告留给观众的印象,一般地说要浅得多。

以下哪项,如果为真,最能使得第二个事实成为对第一个事实的一个合理解释? A在从电视广告里见过的商品中,一般电视观众能记住其品牌名称的大约还不到一半。 B.近年来,被允许在电视节目中连续插播广告的平均时间逐渐缩短。 C.近年来,人们花在看电视上的平均时间逐渐缩短。

D.近年来,一段连续播出的电视广告所占用的平均时间逐渐增加。

E.近年来,一段连续播出的电视广告中所出现的广告的平均数量逐渐增加。

2. 一个美国议员提出,必须对本州不断上升的监狱费用采取措施。他的理由是,现在,一个关在单人牢房里的犯人所需的费用,平均每天高达132美元。即使在世界上开销最昂贵的城市里,也不难在最好的饭店里找到每晚的租金低于125美元的房间。 以下哪项能构成对上述美国议员的观点及其论证的恰当驳斥? Ⅰ据州司法部公布的数字,一个关在单人牢房里的犯人所需的费用,平均每天125美元。 Ⅱ在世界上开销最昂贵的城市里,很难在最好的饭店里找到每晚的租金低于125 的房间。

Ⅲ监狱用于犯人的费用,和饭店用于客人的费用,几乎用于完全不同的开支项目。 A.只有Ⅰ。 B.只有Ⅱ。

C.只有Ⅲ。 D.只有Ⅰ和Ⅱ。 E.Ⅰ、Ⅱ和Ⅲ。

3. 过去的20年里,科幻类小说占全部小说的销售比例从1%提高到了10%。其间,对这种小说的评论也有明显的增加。一些书商认为,科幻小说销售量的上升主要得益于有促销作用的评论。

以下哪项如果为真,最能削弱题干中书商的看法? A.科幻小说的评论,几乎没有读者。

B.科幻小说的读者中,几乎没有人读科幻小说的评论。


C.科幻小说评论文章的读者,几乎都不购买科幻小说。 D.科幻小说评论文章的作者中,包括著名的科学家。

E.科幻小说的评论文章的作者中,包括因鼓吹伪科学而臭了名声的作家。

4. 在司法审判中,所谓肯定性误判是指把无罪者判为有罪,否定性误判是指把有罪者判为无罪。肯定性误判就是所谓的错判,否定性误判就是所谓的错放。而司法公正的根本原则是“不放过一个坏人,不冤枉一个好人”

法学家认为,目前,衡量一个法院在办案中是否对司法公正的原则贯彻得足够的好,就看它的肯定性误判率是否足够低。

以下哪项,如果为真,能最有力地支持上述法学家的观点?

A.错放,只是放过了坏人;错判,则是既放过了坏人,又冤枉了好人。 B.宁可错判,不可错放,是“左”的思想在司法界的反映。

C.错放造成的损失,大多是可弥补的;错判对被害人造成的伤害,是不可弥补的。 D.各个法院的办案正确率普遍有明显的提高。 E.各个法院的否定性误判率基本相同。

5.一位海关检查员认为,他在特殊工作经历中培养了一种特殊的技能,即能够准确地判定一个人是否在欺骗他。他的根据是,在海关通道执行公务时,短短的几句对话就能使他确定对方是否可疑;而在他认为可疑的人身上,无一例外地都查出了违禁物品。 以下哪项如果为真,能削弱上述海关检查员的论证?

在他认为不可疑而未经检查的入关人员中,有人无意地携带了违禁物品。 在他认为不可疑而未经检查的入关人员中,有人有意地携带了违禁物品。 在他认为可疑并查出违禁物品的入关人员中,有人是无意地携带的违禁物品。 A.只有Ⅰ。 B.只有Ⅱ。 C.只有Ⅲ。 D.只有Ⅱ和Ⅲ。 E.Ⅰ、Ⅱ和Ⅲ。

6. 美国法律规定,不论是驾驶员还是乘客,坐在行驶的小汽车中必须系好安全带。有人对此持反对意见。其理由是,每个人都有权冒自己愿意承担的风险,只要这种风险不会给别人带来损害。因此,坐在汽车里系不系安全带,纯粹是个人的私事,正如有人愿意承担风险去炒股,有人愿意承担风险去攀岩纯属他个人的私事一样。以下哪项,如果为真,最能对上述反对意见提出质疑?

A.尽管确实为了保护每个乘客自己,而并非为了防备伤害他人,但所有航空公司仍然要求每个乘客在飞机起飞和降落时系好安全带。

B.汽车保险费近年来连续上涨,原因之一,是由于不系安全带造成的伤亡使得汽车保险赔偿费连年上涨。

C.在实施了强制要求系安全带的法律以后,美国的汽车交通事故死亡率明显下降。 D.法律的实施带有强制性,不管它的反对意见看来多么有理。

E.炒股或攀岩之类的风险是有价值的风险,不系安全带的风险是无谓的风险。

7. 据世界卫生组织95年调查报告显示,70%的肺癌患者都有吸烟史。这说明,吸烟将极大增加患肺癌的危险。

以下哪项,如果是真的,将严重削弱上述结论? A. 有吸烟史的人在95年超过世界总人口的65% B. 95年世界吸烟的人数比94年增加了70% C. 被动吸烟被发现同样有致肺癌的危险。

D. 没有吸烟史的人数在95年超过世界总人口的40% E. 95年未成年吸烟者的人数有惊人的增长。 8. 广告:


本厨师培训班有着其它同类培训班所没有的特点,就是除了传授高超的烹饪技艺外,负责向毕业生提供确实有效的就业咨询。去年进行咨询的本培训班毕业生中,100%都找到了工作。为了在烹饪业找到一份理想的工作,欢迎您加入我们的行列。 为了确定该广告的可信性,以下哪个相关问题是必须询问清楚的? 去年有多少毕业生?

去年有多少毕业生进行就业咨询?

上述就业咨询在咨询者找到工作的过程中,究竟起到了多少作用? 咨询者找到的工作,是否都属于烹饪行业? A.Ⅰ、Ⅱ、Ⅲ、Ⅳ。 B.只有Ⅰ、Ⅱ和Ⅲ。

C.只有Ⅱ、Ⅲ和Ⅳ。 D.只有Ⅲ和Ⅳ。 E.只有Ⅰ和Ⅱ。

9.清朝雍正年间,市面流通的铸币,其金属构成是铜六铅四,即六成为铜,四成为铅。不少商人出以利计,纷纷融币取铜,使得市面的铸币严重匮乏,不少地方出现以物易物。但朝廷征于市民的赋税,须以铸币缴纳,不得代以实物或银子。市民只得以银子向官吏购兑铸币用以纳税,不少官吏因此大发了一笔。这种情况,明清两朝以来从未出现过。 从以上陈述,可推出以下哪项结论?

Ⅰ上述铸币中所含铜的价值要高于该铸币的面值。

Ⅱ上述用银子购兑铸币的交易中,不少并不按朝廷规定的比价成交。 Ⅲ雍正以前明清诸朝,铸币的铜含量,均在六成以下。 A.只有Ⅰ。 B.只有Ⅱ。

C.只有Ⅲ。 D.只有Ⅰ和Ⅱ。 E.Ⅰ、Ⅱ和Ⅲ。

10. 有一种观点认为,到21纪初,和发达国家相比,发展中国家将有更多的人死于艾滋病。其根据是:据统计,艾滋病毒感染者人数在发达国家趋于稳定或略有下降,在发展中国家却持续快速上升;到21世纪初,估计全球的艾滋病毒感染者将达到4000万至1亿1千万人,其中,60%将集中在发展中国家。这一观点缺乏充分的说服力。因为,同样权威的统计数据表明,发达国家的艾滋病感染者从感染到发病的平均时间要大大短于发展中国家,而从发病到死亡的平均时间只有发展中国家的二分之一。 以下哪项最为恰当地概括了上述反驳所使用的方法? A.对“论敌”的立论动机提出质疑。

B.指出“论敌”把两个相近的概念当作同一概念来使用。 C.对“论敌”的论据的真实性和准确性提出质疑。 D.提出一个反例来否定“论敌”的一般性结论。 E.指出“论敌”在论证中没有明确具体的时间范围。 11.许多孕妇都出现了维生素缺乏的症状,但这通常不是由于孕妇的饮食中缺乏维生素,而是由于腹内婴儿的生长使她们比其他人对维生素有更高的需求。 为了评价上述结论的确切程度,以下哪项操作最为重要?

A.对某个缺乏维生素的孕妇的日常饮食进行检测,确定其中维生素的含量。 B.对某个不缺乏维生素的孕妇的日常饮食进行检测,确定其中维生素的含量。 C.对孕妇的科学食谱进行研究,以确定有利于孕妇摄入足量维生素的最佳食谱。 D.对日常饮食中维生素足量的一个孕妇和一个非孕妇进行检测,并分别确定她们是否缺乏维生素。

E.对日常饮食中维生素不足量的一个孕妇和另一个非孕妇进行检测,并分别确定她们是否缺乏维生素。

12. 赞扬一个历史学家对于具体历史事件阐述的准确性,就如同是在赞扬一个建筑师在


本文来源:https://www.dywdw.cn/b3189656a16925c52cc58bd63186bceb19e8ed82.html

相关推荐
推荐阅读